Введение
Про ведение математической олимпиады и подведение ее итогов
Примерные тексты математических олимпиад для муниципального этапа
5 класс
Вариант 1
Вариант 2
Вариант 3
Вариант 4
Вариант 5
Варнант 6
Вариант 7
Вариант 8
Вариант 9
Вариант 10
6 класс
Вариант 1
Вариант 2
Вариант 3
Вариант 4
Вариант 5
Вариант 6
Вариант 7
Вариант 8
Варнант 9
Вариант 10
7 класс
Вариант 1
Вариант 2
Вариант 3
Вариант 4
Вариант 5
Вариант 6
Вариант 7
Вариант 8
Вариант 9
Вариант 10.
8 класс
Вариант 1
Вариант 2
Вариант 3
Вариант 4
Вариант 5
Вариант 6
Вариант 7
Вариант 8
Вариант 9
Вариант 10
9 класс
Вариант 1
Вариант 2
Вариант 3
Вариант 4
Вариант 5
Вариант 6
Вариант 7
Вариант 8
Вариант 9
Вариант 10
10 класс
Вариант 1
Вариант 2
Вариант 3
Варнант 4
Вариант 5
Вариант 6
Вариант 7
Вариант 8
Вариант 9
Варнант 10
11 класс
Вариант 1
Варнант 2
Вариант 3
Вариант 4
Вариант 5
Вариант 6
Вариант 7
Вариант 8
ВариантТ 9
Вариант 10
Решения, указания, ответы
5 класс
Вариант 1
Варнант 2
Варнант 3
Вариант 4
Вариант 5
Варнант 6
Вариант 7
Вариант 8
Вариант 9
Вариант 10
6 класс
Вариант 1
Вариант 2
Вариант 3
Вариант 4
Варнант 5
Вариант 6
Вариант 7
Вариант 8
Вариант 9
Вариант 10
7 класс
Вариант 1
Вариант 2
Вариант 3
Вариант 4
Вариант 5
Вариант 6
Вариант 7
Вариант 8
Варнант 9
Вариант 10
8 класс
Вариант 1
Вариант 2
Вариант 3
Вариант 4
Вариант 5
Вариант 6
Вариант 7
Вариант 8
Вариант 9
Вариант 10
9 класс
Вариант 1
Вариант 2
Вариант 3
Вариант 4
Вариант 5
Вариант 6
Вариант 7
Вариант 8
Вариант 9
Вариант 10
10 класс
Вариант 1
Вариант 2
Вариант 3
Вариант 4
Вариант 5
Вариант 6
Вариант 7
Вариант 8
Вариант 9
Вариант 10.
11 класс
Варнант 1
Вариант 2
Вариант 3
Вариант 4
Вариант 5
Варнант 6
Вариант 7
Вариант 8
Вариант 9
Вариант 10
Текст
                    А. В. Фарков
МАТЕМАТИЧЕСКИЕ
олимпиады

Александр Фарков МАТЕМАТИЧЕСКИЕ ОЛИМПИАДНЫЕ РАБОТЫ 5-11 классы ПИТЕР Москва - Санкт-Петербург - Нижний Новгород - Воронеж Ростов-на-Дону - Екатеринбург - Самара - Новосибирск Киев - Харьков - Минск 2010
ББК 22.1я7 УДК 51(075) Ф24 Рецензент: доктор физико-математических наук, профессор В. Н. Попов Александр Фарков Ф24 Математические олимпиадные работы. 5-11 классы. — СПб,: Питер, 2010. —-192 с.: ил. ISBN 978-5-49807-725-3 Александр Фарков — педагог с огромным опытом работы в школе и вузе. Он орга- низовывал проведение городских олимпиад по математике, составлял тексты для проведе- ния районных олимпиад, занимался организацией работы с одаренными учащимися, вел занятия по методике преподавания математики. В пособии содержатся примерные тексты математических олимпиад для проведения второго (муниципального) 'этапа Всероссийской математической олимпиады. Пособие предназначено для учащихся 5-11 классов и их родителей для подготовки к участию в математических олимпиадах и других математических соревнованиях, а также для учителей математики, методистов отделов образования, преподавателей вузов, состави- телей текстов математических олимпиад ББК 22.1я7 УДК 51(075) Все права защищены Никакая часть данной книги не можех быть воспроизведена в какой бы то ни было форме без письменного разрешения владельцев авторских прав ISBN 978-5-49807-725-3 © ООО Издательство «Питер», 2010
Содержание Введение.................................................. 8 Проведение математической олимпиады и подведение ее итогов..19 Примерные тексты математических олимпиад для муниципального этапа...................................34 5 класс.................................................34 Вариант 1............................................34 Вариант 2............................................35 Вариант 3 ......................................... 36 Вариант 4............................................37 Вариант 5.......................................... 38 Вариант 6.......................................... 38 Вариант 7.......................................... 39 Вариант 8.......................................... 39 Вариант 9.......................................... 40 Вариант 10...........................................41 6 класс............................................... 42 Вариант 1............................................42 Вариант 2.......................................... 43 Вариант 3.......................................... 44
4 Математические олимпиадные работы. 5-11 классы Вариант 4.............................................44 Вариант 5........................................... 45 Вариант 6.............................................46 Вариант 7.............................................47 Вариант 8.............................................48 Вариант 9.............................................49 Вариант 10.......................................... 49 7 класс.................................................50 Вариант 1.............................................50 Вариант 2.............................................51 Вариант 3.............................................51 Вариант 4.............................................52 Вариант 5.............................................53 Вариант 6.............................................53 Вариант 7........................................... 54 Вариант 8.............................................55 Вариант 9.............................................55 Вариант 10............................................56 8 класс.................................................57 Вариант 1.............................................57 Вариант 2.............................................58 Вариант 3.............................................58 Вариант 4.............................................58 Вариант 5.............................................59 Вариант 6.............................................60 Вариант 7.............................................60 Вариант 8....................................... 61 Вариант 9.............................................62 Вариант 10............................................62 9 класс.................................................63 Вариант 1.............................................63 Вариант 2.............................................64 Вариант 3.............................................65 Вариант 4.............................................65 Вариант 5.............................................66 Вариант 6.............................................67 Вариант 7.............................................67
Содержание 5 Вариант 8............................................67 Вариант 9............................................68 Вариант 10...........................................69 10 класс...............................................70 Вариант 1............................................70 Вариант 2............................................71 Вариант 3............................................71 Вариант 4.......................................... 72 Вариант 5............................................72 Вариант 6............................................73 Вариант 7.......................................... 74 Вариант 8............................................74 Вариант 9............................................75 Вариант 10...........................................76 11 класс...............................................77 Вариант 1............................................77 Вариант 2............................................77 Вариант 3............................................78 Вариант 4............................................79 Вариант 5............................................79 Вариант 6............................................80 Вариант 7............................................80 Вариант 8............................................81 Вариант 9............................................81 Вариант 10...........................................82 Решения, указания, ответы..................................83 5 класс.................................................83 Вариант 1............................................83 Вариант 2............................................85 Вариант 3............................................86 Вариант 4............................................87 Вариант 5 ......................................... 87 Вариант 6.......................................... 89 Вариант 7............................................89 Вариант 8............................................90 Вариант 9.......................................... 92 Вариант 10...........................................92
6 Математические олимпиадные работы. 5-11 классы 6 класс..................................................94 Вариант 1............................................ 94 Вариант 2.......................................... 95 Вариант 3............................................ 96 Вариант 4........................................ 97 Вариант 5.......................................... 98 Вариант 6.......................................... 99 Вариант 7.............................................100 Вариант 8........................................... 102 Вариант 9............................ ...............103 Вариант 10.......................................... 105 7 класс.............................................. 106 Вариант 1......................................... 106 Вариант 2.............................................107 Вариант 3 .......................................... 109 Вариант 4........................................... 110 Вариант 5........................................... 111 Вариант 6..............................................113 Вариант 7......................................... 114 Вариант 8........................................... 115 Вариант 9........................................... 116 Вариант 10............................................117 8 класс................................................119 Вариант 1..............................................119 Вариант 2........................................... 121 Вариант 3........................................... 122 Вариант 4........................................... 124 Вариант 5......................................... 126 Вариант 6........................................... 127 Вариант 7......................................... 129 Вариант 8........................................... 130 Вариант 9........................................... 131 Вариант 10.......................................... 133 9 класс........................ ......................135 Вариант 1..................................... 135 Вариант 2........................................... 136 Вариант 3........................................... 138
Содержание 7 Вариант 4 .................................... 139 Вариант 5................................... 141 Вариант 6..................................... 142 Вариант 7..................................... 144 Вариант 8 .............................. 146 Вариант 9....................................... 148 Вариант 10........................................ 150 10 класс............................................ 152 Вариант 1....................................... 152 Вариант 2...........................................153 Вариант 3 ..........................................155 Вариант 4....................................... 157 Вариант 5...........................................158 Вариант 6......................................... 160 Вариант 7................................... 162 Вариант 8....................................... 164 Вариант 9......................................... 166 Вариант 10.................................... 168 И класс............................................. 169 Вариант 1................................... 169 Вариант 2....................................... 170 Вариант 3......................................... 173 Вариант 4....................................... 176 Вариант 5...........................................178 Вариант 6....................................... 181 Вариант 7......................................... 183 Вариант 8....................................... 185 Вариант 9...................................... 187 Вариант 10.................................... 189
Введение В последние годы в России проводится много различных матема- тических соревнований. Тем не менее наибольшей популярностью из них в большинстве регионов пользуются олимпиады, к которым отно- сятся традиционные всероссийские олимпиады, олимпиады по лигам, дистанционные олимпиады, устные олимпиады, заочные олимпиады, нестандартные олимпиады, олимпиады-конкурсы «Кенгуру» и т. п. Всероссийские математические олимпиады до последнего време- ни проходили в пять этапов, включая школьный, муниципальный (районный, городской), региональный (областной, краевой, респуб- ликанский), федеральный окружной и заключительный. С 2008 г. со- гласно «Положению о Всероссийской олимпиаде школьников» феде- ральный окружной этап не проводится, а победители региональных этапов олимпиад сразу становятся участниками заключительного этапа Всероссийской математической олимпиады. Данный вид олимпиад остается на сегодня самым массовым и по- пулярным как среди учащихся, так и среди учителей. И хотя популярность традиционных математических олимпиад и сегодня высока, по сравнению с восьмидесятыми годами в боль- шинстве регионов все меньше стало проводиться олимпиад для уча- щихся 5-8 классов, даже несмотря на то, что именно учащиеся это- го возраста наиболее любознательны и с удовольствием участвуют в различных соревнованиях.
Введение 9 К сожалению, перестали проводиться соросовские олимпиады, в которых принимало участие много школьников. В связи с введени- ем ЕГЭ практически прекратили свое существование во многих ре- гионах олимпиады для абитуриентов, проводящиеся в вузах. Основными целями проведения математических олимпиад явля- ются: • повышение интереса к изучению математики; • выявление наиболее интеллектуально одаренных учащихся по математике; • содействие целенаправленному выбору профессии; • воспитание организованности, дисциплинированности, воли; • привитие навыков к систематическим занятиям внеклассной и внешкольной работой; • пробуждение желания учащихся самостоятельно приобретать знания и применять их на практике. Так как одной из основных целей муниципальной олимпиады остается поиск и отбор наиболее одаренных учащихся по математи- ке, которые в дальнейшем могли бы защищать честь района (города) на региональной олимпиаде, необходим специальный инструмент для достижения данной цели, равно как и остальных целей проведе- ния олимпиад. Сегодня этим инструментом является текст олимпиадной работы. Л чтобы с помощью данного инструмента можно было действительно выявлять наиболее одаренных и обучаемых учащихся по математике, данный инструмент должен соответствовать определенным требова- ниям. Тексты для второго этапа чаще всего разрабатываются методиче- скими комиссиями субъекта Российской Федерации. Однако соста- вителями текстов могут быть также учителя ряда школ и препода- ватели вузов, которые не всегда являются специалистами в области диагностики. Особенно часто это происходит, когда олимпиада про- водится для учащихся 5-6 классов, причем региональный этап для них не проводится. Ведь согласно «Положению о Всероссийской олимпиаде» второй этап проводится лишь для учащихся 7-11 клас- сов, а третий — лишь для учащихся 9-11 классов. Анализ литературы по проблемам олимпиадного движения, а так- же собственный опыт участия в математических олимпиадах на раз- личных этапах и составления текстов математических олимпиад
10 Математические олимпиадные работы. 5-11 классы позволяют выдвинуть следующие основные требования к тексту ма- тематической олимпиады, служащей инструментом для выявления наиболее интеллектуально одаренных учащихся в области математи- ки на втором этапе. 1. Число заданий в тексте олимпиадной работы должно быть от 4 до 6 (чаще 5). Меньшее число задач может не позволить достичь планиру- емых целей проведения олимпиады. А если ученикам пред- ложить 7 и более задач (некоторые составители текстов для олимпиадных работ включают 10 и даже больше задач), то вряд ли кто-то из них успеет все их решить. И тогда победите- лем может оказаться не самый одаренный учащийся. 2. Все задания в тексте олимпиадной работы должны быть рас- положены в порядке возрастания трудности, или сложности. Хотя в последние годы данные понятия довольно часто встре- чаются в методической литературе, все же напомним их определения. Сложность — это объективная характеристика задачи, определяемая ее структурой. Сложность задачи зави- сит от: — объема информации (числа понятий, суждений), необходи- мого для ее решения; — числа данных в задаче; — числа связей между ними; — количества возможных выводов из условия задачи; — количества непосредственных выводов, необходимых для решения задачи; — количества взаимопроникновений при решении задачи; — длины рассуждений при решении задачи; — общего числа шагов решения, привлеченных аргументов и т. д. Рассчитать сложность задачи не очень просто, поэтому чаще всего составители распределяют задачи по сложности интуи- тивно. Однако в текст олимпиадной работы задания включа- ются из разных разделов, причем некоторые из них нестан- дартные, поэтому лучше всего вместо понятия «сложность» применять понятие «трудность» задания.
Введение 11 Трудность — это субъективная характеристика задачи, опреде- ляемая взаимоотношениями между задачей и решающим ее учеником. Трудность задачи зависит от: — сложности задачи (сложная задача, как правило, является более трудной для учащихся); — времени, прошедшего после изучения материала, упоми- наемого в тексте задачи (задачи на материал, изученный 1-2 года назад, используемые факты, которые уже забылись, более трудны для учащихся); — практики в решении подобного рода задач (учащиеся, кото- рые тренировались на кружковых и факультативных заня- тиях в решении определенного типа задач, как правило, без проблем решают их и на районной олимпиаде); — уровня развития учеников (задача, трудная для среднего ученика общеобразовательного класса, может быть легкой для обычного ученика физико-математического класса); — возраста учащихся (задача, трудная для пятиклассника, мо- жет быть легкой для восьмиклассника) и т. д. Трудность определяется процентом учеников, решивших зада- чу из числа ее решавших. Существуют различные формулы для расчета трудности зада- чи. Рассмотрим, на наш взгляд, наиболее простую из них: п ”400%, где КТ — коэффициент трудности, измеряемый в процентах; п — число учащихся, не решивших задачу; р — число учащихся, решавших задачу, в том числе не приступивших к ней (общее число участников олимпиады). Пример представлен в следующей таблице: Номер задачи 1 2 3 4 5 6 п 2 6 10 12 16 19 Р 20 20 20 20 20 20 10% 30% 50% 60% 80% 95%
12 Математические олимпиадные работы. 5-11 классы Из данной таблицы следует, что 6-я задача — наиболее труд- ная, так как ее решил всего 1 ученик, а 1-я — наиболее легкая, ее решили 18 учеников. 3. Первые одна-две задачи должны быть доступны большинству участников олимпиады (их должны решить 60-90 % учащихся). В числе этих первых задач могут быть как и наиболее лег- кие олимпиадные задачи, так и задачи, аналогичные наиболее сложным задачам школьных учебников и контрольных работ. Условия задач можно немного изменить. Желательно, чтобы такие задачи содержали «изюминку», заметив которую, сооб- разительный ученик решил бы их быстрее и красивее. Примеры: т. 7 8 9 10 1) Расставьте числа —; —; —; -— в порядке убывания 8 9 10 И (6 класс). 2) Найдите все такие числа а, при которых дробь ----- явля- а -4 ется целым числом (8 класс). 3) Сколько может быть шестизначных чисел с суммой цифр, равной 2 (5-9 классы)? Включение в текст работы посильной для большинства уча- щихся задачи вселяет в учащихся веру в свои силы, возбуждает энтузиазм, пробуждает желание лучше учиться и добиваться дальнейших успехов на следующих олимпиадах. Кроме того, это способствует гуманности олимпиады. Сложно объяснить ученику — победителю школьной олимпиады, который впер- вые приехал на районную олимпиаду и не смог решить ни одной задачи из предлагаемых пяти, почему так случилось. Здесь можно было бы провести Параллель с соревнованиями по баскетболу, в которых участникам обеих команд ни разу не удалось попасть в баскетбольное кольцо из-за того, что диаметр мяча был чуть больше диаметра кольца. В случае же олимпиа- ды предлагаемая составителями трудность задач оказалась не под силу участникам олимпиады. 4. Следующие две-три задачи должны быть более трудными, такими, чтобы справиться с ними смогла примерно половина участников.
Введение 13 Это могут быть задачи, которые не рассматривались на уроках, но с идеями решения которых ученики встречались во вне- классной работе или при самостоятельном знакомстве с раз- личными пособиями. Примеры: 1) Как набрать из озера 8 литров воды, имея девятилитровое и пятилитровое ведра (5-6 классы)? 2) Расшифруйте следующую запись примера па сложение, в котором разным буквам соответствуют разные цифры, а одинаковым — одинаковые (5-6 классы): . СПОРТ СПОРТ КРОСС 3) Постройте график уравнения (10-11 классы): 1Ы + Ы - з| = 2. 4) Решите уравнение (10-11 классы): г1 - 7х3 + 14л'2 - 7х + 1 = 0. 5. Последние одна-две задачи могут содержать материал, не изу- чаемый в школе. В данном случае используются задания, аналогичные задачам регионального этапа олимпиад. Эти задания под силу только некоторым из участников. Примеры: 1) В комнате собрались 8 человек. Некоторые из них постоян- но лгут, остальные всегда говорят правду. Один из собрав- шихся сказал: «Здесь нет ни одного честного человека». Второй сказал: «Здесь не больше одного честного челове- ка». Третий сказал: «Здесь не более двух честных людей» и т. д., вплоть до восьмого, который сказал: «Здесь не более семи честных людей». Сколько в комнате честных людей? Ответ обоснуйте. 2) Решите в натуральных числах уравнение: 19m + 98/7 = 1998.
14 Математические олимпиадные работы. 5-11 классы 3) В каждую клетку квадратной таблицы 25 х 25 вписано про- извольным образом одно из чисел: 1 или -1. Под каждым столбцом пишется произведение всех чисел, стоящих в этом столбце. Справа от каждой строки пишется произведение всех чисел, стоящих в этой строке. Докажите, что сумма 50 написанных произведений не может быть равной нулю. 6. В качестве предложенных задач желательно иметь задачу, со- держащую год проведения олимпиады. Примеры: 1) Запишите подряд 22 пятерки: 5555...5. Поставьте между не- которыми цифрами знаки арифметических действий так, чтобы в результате получилось 2002 (5-6 классы). 2) Сколькими нулями оканчивается произведение: 1 х 2 х 3 х х 4 х ... х 2006 х 2007 х 2008 х 2009 (7-9 классы). 3) Сравните числа: V2004 + V2002 и 2 V2003 (8 класс). 4) На какую цифру оканчиваются числа 19992010 и 19982005 (8 класс)? 5) Решите в натуральных числах уравнение (9-11 классы): X1 — у2 — 2002. 6) Можно ли разбить равносторонний треугольник на 2008 равносторонних треугольников (возможно, не все из них равны между собой)? Если можно, то как? Если нет, то объясните, почему (9-11 классы). 7. В 5-6 классах в текст олимпиадной работы необходимо вклю- чать одну-две занимательные задачи. Примеры'. 1) До царя Гороха дошла молва, что кто-то из троих богатырей убил Змея Горыныча. Царь приказал всем троим явиться ко двору, и молвили они: Илья Муромец: «Змея убил Добрыня Никитич». Добрыня Никитич: «Змея убил Алеша Попович». Алеша Попович: «Я убил Змея». При этом оказалось, что один из них сказал правду, а двое слукавили. Кто убил Змея?
Введение 15 2) Илья Муромец, Добрыня Никитич и Алеша Попович всту- пили в бой с великанами. Получив по три удара богатырски- ми палицами, великаны обратились в бегство. Больше всего ударов (7) нанес Илья Муромец, меньше всех (3) — Алеша Попович. Сколько всего было великанов? 3) Дядя Федор, кот Матроскин, Шарик и почтальон Печкин сидят на скамейке. Если Шарик, сидящий справа от всех, сядет между дядей Федором и котом, то кот окажется край- ним слева. В каком порядке они сидят? 4) Буратино отпил полчашки черного кофе и долил ее моло- ком. Потом он отпил — чашки и долил ее молоком. Потом 1 3 он отпил — чашки и долил ее молоком. Наконец, Буратино 6 допил содержимое чашки до конца. Чего Буратино выпил больше: кофе или молока? 8. Включаемые в текст задачи должны быть как из разных раз- делов школьного курса математики (не должно быть двух тек- стовых задач, двух уравнений и т. п.), так и «чисто олимпиад- ными» (иметь специальные методы решения), при их решении должны применяться разные приемы, идеи. При включении в текст задач на базе программного материала необходимо быть особенно осторожными. Ведь один и тот же материал может изучаться по различным учебникам нс только в разное время учебного года, но и в разных классах. Учебников сейчас много (в различных классах от 3 до 8, а еще есть и экс- периментальные учебники). 9. В текстах олимпиадных работ для различных классов могут быть как одинаковые задачи, так и задачи, использующие одну и ту же идею, но постепенно усложняющуюся от класса к классу. Примеры. 1) Задача Ньютона. Трава по всему лугу растет одинаково бы- стро и густо. Известно, что 70 коров съели бы ее за 24 дня, а 30 коров — за 60 дней. Сколько коров съели бы всю тра- ву за 96 дней? Предполагается, что коровы съедают траву равномерно (9—11 классы). 2) Разделите прямоугольник размером 3x4 на две равные части. Найдите как можно больше способов. Резать можно
16 Математические олимпиадные работы. 5-11 классы лишь по стороне квадрата 1 х 1, и способы считаются разны- ми, если получаемые фигуры в каждом случае оказываются неравными (5-8 классы). 3) Петя, Вася, Коля и Миша играли в футбол. Один из них раз- бил мячом стекло. На вопрос «Кто это сделал?» Петя, Вася и Коля ответили: «Не я», а Миша — «Не знаю». Потом ока- залось, что двое из них сказали правду, а двое солгали. Знает ли Миша, кто разбил стекло? Ответ объясните. 4) Петя, Вася, Коля и Миша играли в футбол. Кто-то разбил мячом стекло. На вопрос: «Кто это сделал?» — пятеро сви- детелей ответили так: Первый: «То ли Петя, то ли Вася». Второй: «То ли Петя, то ли Коля». Третий: «То ли Коля, то ли Миша». Четвертый: «То ли Миша, то ли Вася». Пятый: «Не знаю». Потом оказалось, что трое свидетелей сказали правду, а двое солгали. Знал ли пятый свидетель, кто разбил стекло? 5) Петя, Коля, Вася, Саша и Вова играли в футбол. Кто-то из ребят разбил мячом стекло. На вопрос «Кто это сделал?» они ответили так: Коля: «Это не я и не Петя». Петя: «Это не я и не Саша». Вася: «Это не я и не Петя». Саша: «Это Коля или Вова». Вова: «Не знаю». Потом оказалось, что двое ребят сказали правду, а трое сол- гали. Знал ли Вова, кто разбил стекло? (Задачи 3-5 можно предложить соответственно в 5, 6, 7 клас- сах.) 10. В тексты олимпиадных работ все время необходимо включать новые задачи, тематику задач из года в год необходимо менять. И. Задачи, требующие формального применения формул, громозд- ких вычислений или использования труднозапоминающихся формул, в тексты олимпиадных работ лучше не включать.
Введение 17 12. Желательно включать в текст такие задачи, которые позволя- ли бы оценивать их решение разным числом баллов (к числу таких задач можно отнести логические задачи, задачи на по- иск различных вариантов разрезания, геометрические и дру- гие). 13. Так как различные районы (города) одной области (республи- ки) могут сильно отличаться по уровню обученности и об- учаемости, причем некоторые темы могут вообще остаться не изученными, то жюри городской (районной) олимпиады должно иметь право вносить изменения в рекомендованные вышестоящими органами управления тексты олимпиадных работ. Это может быть как упрощение, так и усложнение тек- ста олимпиады. Лучше, если в регионе составителями текстов муниципальных олимпиад будут одни и те же люди, поскольку им легче устра- нить ошибки, промахи, совершенные в прошлом году. Основным содержанием данного пособия являются тексты мате- матических олимпиад для проведения второго этапа всероссийских олимпиад. Большая часть предлагаемых текстов составлена автором, по некоторым из них проводились городские олимпиады в г. Коряж- ма Архангельской области. В основу других текстов положены тек- сты математических олимпиад, разработанных другими авторами — известными в России математиками и методистами, являющимися специалистами в данном вопросе. Необходимо отметить, что все эти тексты олимпиадных работ автором переработаны, в них включен ряд других задач с целью сохранения единых требований к текстам, кроме того, в некоторых случаях приведены собственные решения автора. Пособие адресовано в первую очередь руководителям городских и районных методических объединений учителей математики, мето- дистам отделов образования, преподавателям вузов, принимающим участие в разработке текстов олимпиадных работ, п, конечно же, учителям математики общеобразовательных учреждений. Оно будет полезно и руководителям математических кружков внешкольных об- разовательных учреждений, студентам математических факультетов педвузов. Как показывает опыт, многие учащиеся пятых-одиннадцатых классов используют аналогичные пособия для самостоятельной под- готовки к школьным и районным математическим олимпиадам раз-
18 Математические олимпиадные работы. 5-11 классы личного уровня и добиваются определенных успехов. Поэтому дан- ное пособие рекомендовано также для учащихся. Практически ни одна книга, а особенно посвященная олимпиа- дам, не может быть идеальной. Что-то в ней удачно, что-то не совсем устраивает читателей. Поэтому автор будет благодарен за все крити- ческие замечания, пожелания, которые он постарается учесть в сле- дующих изданиях. Частично в данном пособии использованы материалы книги авто- ра «Математические олимпиады: Методическое пособие», вышедшей в 2004 г. в издательстве «Гуманитарный издательский центр «ВЛА- дос». Книга является логическим продолжением книги «Математиче- ские олимпиады в школе. 5-11 классы» (издательство «Айрис»), вы- державшей уже 8 изданий. Все замечания по улучшению нового пособия можно высылать в издательство, а также лично автору: 164500. Архангельская область, г. Северодвинск, пр. Труда, д. 30, кв. 54. Фаркову А. В., электронный адрес: a.farkov@mail.ru.
Проведение математической олимпиады и подведение ее итогов Второй (муниципальный) этап Всероссийской математической олимпиады проводится в ноябре-декабре городскими (районными) отделами управления образованием примерно по следующему плану: 1. Создание оргкомитета. 2. Составление текстов олимпиадной работы. 3. Создание жюри. 4. Проведение олимпиады. 5. Проверка решений. 6. Подведение итогов и награждение победителей и призеров. Данный этап олимпиад являются массовым соревнованием, охва- тывающим лучших учащихся всех школ данного района (города). Проводятся такие олимпиады один раз в год. В них участвуют по- бедители или призеры школьных олимпиад (согласно положению об олимпиаде), но могут участвовать и все желающие. В этом случае при определении командного первенства между школами (чаще всего не- официального) их результаты не учитываются. Для проведения олимпиады заведующим отделом образования издается приказ, а также утверждается положение об олимпиаде,
20 Математические олимпиадные работы. 5-11 классы в котором прописываются цели, сроки, место, порядок проведения олимпиады, порядок подведения итогов олимпиады, а также указы- вается продолжительность олимпиады для различных классов. Кро- ме того, в положении перечисляются фамилии членов оргкомитета и указывается, как будет проходить формирование Жюри олимпиады. Районная (городская) олимпиада по математике проводится под непосредственным руководством районного (городского) отдела об- разования. Ответственным за осуществление всех организационных мероприятий обычно назначается районный (городской) методиче- ский кабинет. Для подготовки и проведения олимпиады создается оргкомитет, который чаще всего возглавляет руководитель районно- го (городского) методического объединения учителей математики. В оргкомитет включают наиболее опытных, авторитетных учите- лей математики ряда школ, а также преподавателей вузов, научно- исследовательских институтов, техникумов, училищ, колледжей, ко- торые имеются на территории района (города). Количество членов оргкомитета обычно составляет 3-7 человек. Оргкомитет осуществляет всю подготовительную работу по проведению олимпиады. Он определяет место и время проведения олимпиады, готовит для нее тексты (если тексты не подготовлены предметно-методической комиссией регионального этапа олимпиа- ды), комплектует состав жюри для непосредственного проведения олимпиады и проверки заданий, организует торжественное открытие и закрытие олимпиады. Жюри же проводит олимпиаду, проверяет работы участников олимпиады, определяет победителей. В случае, если региональный оргкомитет разрабатывал единый для всего региона текст заданий олимпиады, районный (городской) оргкомитет имеет право незначи- тельно изменить этот текст. Критерием здесь является общий уровень математической подготовки учащихся данного района. Сделать это можно следующим образом. Представителям школ предлагается под- готовить по 1-2 задания для проведения олимпиад. В день проведе- ния олимпиады, за 1-2 часа до ее начала, один из членов оргкомитета собирает представителей школ и дает им решить предложенные зада- ния. Если сами учителя затрудняются решить некоторые задания или считают, что часть заданий нецелесообразно включать в текст олим- пиады, то такие задания нужно заменить. Решать задания заранее по- лезно еще и потому, что в них могут быть ошибки или опечатки. Кро- ме того, хотя в текст включаются задачи на школьный материал, ко времени проведения олимпиады этот материал по некоторым учебни-
Проведение математической олимпиады и подведение ее итогов 21 кам может оказаться неизученным, и подобную ситуацию могут выя- вить лишь сами учителя. Такое уже случалось в практике проведения олимпиад в некоторых регионах. Тогда вызвавшую затруднение за- дачу заменяют одной из задач, предложенных представителями школ (главное, чтобы остальные задачи вместе с вновь включенной соот- ветствовали требованиям к тексту олимпиады). В случае разногла- сий вопрос решается голосованием. Хорошо, если составители текста олимпиады заранее предусмотрят возможные варианты усложнения или упрощения текста. Например, указать, что в случае, если жюри посчитает данный вариант легким, можно заменить задание 2 задани- ем 5* и изменить порядок предлагаемых заданий: 1,3,4, 5,5*. Если же необходимо усложнить текст, то можно заменить задание 5 заданием 5**, тогда порядок заданий в тексте будет следующим: 1, 2, 3, 4, 5**. В случае, если региональные органы управления образованием не предложили готовых текстов олимпиад, то их могут составить: • методисты городского (районного) отделов образования с при- влечением опытных учителей математики, знающих средний уровень умственного развития учащихся города (района) и тре- бования к текстам олимпиад; • преподаватели кафедр методики преподавания математики и математики вузов, которые имеются в данном регионе; • работники областных органов управления; • различные энтузиасты-специалисты, представители Центров дополнительного образования одаренных школьников, работ- ники клубов «Эврика» и т. и., но обязательно под эгидой отдела образования. Главное, чтобы эта работа была проведена заранее и тексты не ста- ли известны ученикам какой-то из школ. Особенно если составите- лем является учитель данной школы. Иногда сложно совместить проведение математической олимпиа- ды для учащихся 5-11 классов, в таком случае можно определить сле- дующие примерные сроки проведения районных олимпиад для уча- щихся разных классов: • 7-11 классы — ноябрь-декабрь; • 5-6 классы — февраль-март. В этом случае школьные олимпиады необходимо провести соот- ветственно в октябре-ноябре и январе-феврале. А время между про-
22 Математические олимпиадные работы. 5-11 классы ведением школьной и районной олимпиад используется для подго- товки учащихся к районной олимпиаде. Всей подготовкой к участию в районной олимпиаде занимается оргкомитет школьной математи- ческой олимпиады, учителя математики школы. Кроме оргкомитета, в проведении олимпиад принимает участие жюри. В жюри городской (районной) олимпиады наряду с некото- рыми членами оргкомитета и школьными учителями включают- ся преподаватели вузов данного региона, студенты (особенно для 5-8 классов). Для каждой параллели классов создается свое жюри. Число членов жюри зависит от числа участников олимпиады. При- мерный состав его, как уже отмечалось, 3-7 человек. Обязательное требование: председатель жюри не должен иметь среди участников своих учеников. Лучше всего на эту роль подходят преподаватели ву- зов, техникумов, колледжей, училищ. Все это надо хорошо продумать оргкомитету олимпиады. Лучший вариант — включать в состав жюри лишь тех учителей, которые не имеют своих учеников (ведь даже при шифровке работ учителя узнают работы своих учеников по почерку). Ясно, что состав жюри определяется накануне или перед началом проведения олимпиады. А вот председателя жюри, как всей олимпиа- ды, так и по параллелям классов, лучше определять заранее, ведь за- явки от каждой из участвующих в олимпиаде школ приходят заранее. Примерная продолжительность выполнения олимпиадной ра- боты: 9-11 классы — 4 часа; 7-8 классы — 3-4 часа; 5-6 классы -- 2-3 часа. Время выполнения олимпиадной работы будет зависеть от труд- ности предлагаемых заданий, их числа, традиций в регионе. Когда проводить олимпиаду — вопрос не такой простой. Несмо- тря на то что многие математики и методисты, авторы пособий по олимпиадам рекомендуют проводить олимпиады в выходной день, думаю, вряд ли это всегда целесообразно. В некоторых школах одни и те же учащиеся могут быть победителями школьных олимпиад по нескольким предметам, поэтому в случае их участия в нескольких районных олимпиадах они фактически лишаются выходного дня. Поэтому целесообразнее ряд олимпиад проводить в обычные дни (лучше со вторника по четверг), хотя каждый район решает это для себя сам. Возможен вариант проведения олимпиады и в субботу, но это — последний день после напряженной недели, и если в пятницу участникам олимпиады не дать отдохнуть, вряд ли они покажут свой лучший результат. А в итоге ученики пропускают два учебных дня при шестидневной рабочей неделе.
Проведение математической олимпиады и подведение ее итогов 23 Начинать олимпиаду лучше с 9 (10) часов после торжественного открытия и небольшого завтрака для приехавших участников (в слу- чае районной олимпиады). На торжественном открытии олимпиады учащихся поздравляют с участием в олимпиаде, знакомят с регла- ментом, правилами поведения. Особо надо подчеркнуть, что олим- пиада — это соревнование, а поэтому будут как победители, так и по- бежденные. Сами соревнования проходят в больших аудиториях, в которых представители школы, организующей олимпиаду все подготавлива- ют для ее проведения, включая бумагу, запасные ручки, карандаши. При хорошем финансировании олимпиады можно оформить специ- альные подарочные папки для каждого участника. Участников олимпиады желательно рассадить за столы по одно- му проследив, чтобы рядом не оказалось учеников из одной школы. На каждый стол можно положить текст олимпиадной работы, если эти тексты заранее приготовлены. Вариант написания текста на до- ске, на взгляд автора, в современных условиях явно устарел. К тому же это вызовет лишние вопросы, особенно у учащихся с плохим зре- нием, которых в школах становится все больше. В случае большого числа учащихся и нехватки кабинетов возможен вариант проведе- ния олимпиады для двух классов в одной аудитории. В этом случае учащиеся из разных классов и школ садятся за один стол. При проведении олимпиады в аудитории не нужно присутство- вать всем членам жюри, достаточно двух (в крайнем случае трех) че- ловек: председателя жюри и представителя оргкомитета. Остальные члены жюри в это время могут находиться в другой аудитории и про- должать решать предложенные участникам олимпиады задания, на- ходить другие варианты решения того или иного задания, обсуждать возможные варианты выставления баллов за решение заданий. При проведении олимпиады учителям запрещается подходить к «своим» участникам олимпиады. Поэтому лучше подходить к уче- никам (если это очень нужно) председателю жюри олимпиады по данной параллели. Выходить из аудитории участникам олимпиады можно разрешить лишь один раз и то в присутствии члена жюри. По окончании олимпиады желательно сразу или после неболь- шого перерыва (учащиеся должны немного отдохнуть, восстановить силы) провести разбор заданий олимпиады. Разбор проводит один из членов жюри в то время, пока работы участников олимпиады шиф- руются представителем оргкомитета. Хотя на практике встречается
24 Математические олимпиадные работы. 5-11 классы и другой вариант: тексты олимпиад разбираются в каждой школе на уроке или на занятии кружка (факультатива). Проверяют и оценивают решения заданий районной олимпиады члены жюри. Желательно, чтобы в каждой параллели их было не ме- нее 3 человек. Одному учителю оценивать результаты олимпиады не- легко, к тому же он может быть субъективным. После окончания олимпиады члены жюри приступают к проверке заданий олимпиады. Возможны два варианта проверки: 1. Каждый член жюри проверяет только 1--2 задания из текста олимпиады и карандашом оценивает каждое задание, выстав- ляя рядом с заданием определенное число баллов. 2. Каждый член жюри проверяет несколько работ участников, оценивая все задания. Оба варианта проверки имеют как плюсы, так и минусы. Поэтому после проверки всех работ надо снова всем членам жюри еще раз об- судить количество баллов, выставленное за каждое задание. Особен- но это касается работ участников, претендующих на призовые места. При количестве участников 15-25 и числе членов жюри 3-5 человек вся работа по проверке может занять не более 2 часов. Задания всех этапов Всероссийской олимпиады последние годы оце- ниваются по единым нормам исходя из 7 баллов за каждое задание. При этом 7 баллов ставится за верное решение, 6 баллов — за вер- ное решение с недочетами, 4-5 баллов — за верное в целом, но непол- ное или содержащее непринципиальные ошибки решение, 1 -3 балла рекомендуется ставить за неверное в целом решение, но с более или менее существенным продвижением в верном направлении, и 0 бал- лов необходимо ставить за неверное решение или его отсутствие. Например, решение геометрической задачи, в которой требуется до- казать, что данный треугольник является равнобедренным, оценива- ется в 0 баллов, если ученик начинает решение со слов «Пусть тре- угольник АВС — равнобедренный...». ВНИМАНИЕ Жюри должно знать, что задание не может оцениваться дробным числом бал- лов: 0,8; 4,5 и т. п. Начать проверку необходимо с выяснения принципиального во- проса: верно решена задача (тогда ставится 4-7 баллов) или неверно (тогда ее решение оценивается от 0 до 3 баллов).
Проведение математической олимпиады и подведение ее итогов 25 Решение считается неполным в следующих случаях: • если оно содержит основные идеи, но не доведено до конца; • если оно при верной общей схеме рассуждений содержит про- белы, то есть явно или скрыто опирается на недоказанные утверждения, которые нельзя счесть известными или очевид- ными; • если ученик нашел большинство способов, но не все, рассмо- трел не все возможные варианты решения. Исправления, помарки в решениях не учитываются, но учитыва- ется оригинальность решения. Вычислительные ошибки в невычис- лптельных задачах не считаются принципиальными ошибками. При оценке заданий учитывается только их правильность, полнота, обосно- ванность, идейность и оригинальность. За нерациональность решения, как правило, оценка не снижается. Умение догадаться на олимпиаде должно цениться выше, чем умение хорошо изложить решение. Ответ, найденный логическим путем, обычно оценивается выше, чем найден- ный подбором. При этом первоначальная оценка работ участников олимпиады членом жюри может проводиться по системе «плюс-минус», то есть рядом с решением каждой задачи член жюри ставит условные значки, которые «превращаются» в баллы лишь после обсуждения с осталь- ными членами жюри. Жюри желательно также смотреть черновики. Причем при не- большом количестве участников олимпиады работы могут прове- ряться и в присутствии авторов. Недостатки, обнаруженные в чер- новых записях, не учитываются, в то же время учитывается все, что может улучшить чистовик. Иногда составители текстов олимпиад, облегчая работу членам жюри, разрабатывают специальные методические рекомендации, где дают дополнительные указания по оценке того или иного задания. В случае расхождения между общими и дополнительными указа- ниями приоритет имеют дополнительные указания. Однако в случае противоречия между дополнительными указаниями и реально сло- жившейся ситуацией на олимпиаде жюри имеет право вносить изме- нения, касающиеся оценок решения заданий, как в общие, так и в до- полнительные указания. Рассмотрим конкретные рекомендации по оценке заданий олим- пиады на примерах.
26 к Математические олимпиадные работы. 5-11 классы Пример 1. Произведение цифр трехзначного числа равно 4. Най- дите все такие числа. Решение. Произведение 3 цифр может быть равно 4 в следующих двух случаях: • одна из цифр равна 4, а две остальные — единицы; • две из цифр равны 2, а одна -- единице. В итоге получаются числа 411, 141, 114; 122, 212, 221, всего 6 чи- сел. Тогда 7 баллов ставится за верное решение задачи. Неполным решение будет, если рассмотрены оба варианта, причем в каждом из вариантов указано не менее половины случаев. А это означает, что в 5 баллов оценивается решение, когда предложено 5 вариан- тов, в 4 балла — 4 варианта, причем из каждого случая рассмотрено по 2 варианта. Если предложено 4 варианта (причем 3 варианта из одного случая) или менее, то решение в целом считается неверным. Оценить в баллах найденное число вариантов решения можно так: 3 балла - 4 варианта (причем 3 варианта из одного случая), 2 бал- ла -- 2 или 3 варианта решения; 1 балл — 1 вариант решения. Чуть сложнее будет оцениваться следующая задача. Пример 7*. Произведение цифр четырехзначного числа равно 4. Найдите все такие числа. Решение. Произведение 4 цифр может быть равно 4 в следующих двух случаях: ® одна из цифр равна 4, а все остальные - единицы; • две цифры равны 2, а две — единице. В итоге получаются числа: 4111, 1411,1141, 1114, 1122, 1212, 1221, 2112, 2121, 2211, всего 10 чисел. Тогда 7 баллов можно поставить за верное решение задачи, а 6 баллов — за предложенные 9 вариантов (один вариант ученик, по невнимательности, может упустить). Не- полным решение будет, если рассмотрены оба случая, причем в каж- дом из случаев указано не менее половины вариантов. А это означает, что в 5 баллов оценивается решение, когда предложено 8 вариантов; в 4 балла — 6 или 7 вариантов, причем из второго случая рассмотрено не менее четырех, Если же предложено пять или менее вариантов, то решение в целом считается неверным. Оценить в баллах найденное число вариантов решения можно так. Если 4~5 вариантов, но из обо- их случаев, 7 о 3 балла; в 3 балла можно оценить и найденные 6 вари- антов (причем из второго случая предложено 3 варианта). 2 балла —
Проведение математической олимпиады и подведение ее итогов 27 от 3 до 5 вариантов (при этом если 4-5 вариантов из одного случая); 1 балл — 1 или 2 варианта решения. Пример 2. Решите уравнение: х1-7х3 + 14х2 - lx + 1 =0. Решение: Разделим обе части уравнения на х2 (х = 0 не является корнем урав- 7 1 нения), тогда получим уравнение: х2 - 1х + 14-+ -у = 0, которое х х 1 1 преобразуем к уравнению: -1{х + — ) + (х2 + -у) + 14 = 0. Введем х х 1 новую переменную у = х + —, получим уравнение: у2 - 7у + 12 = 0, х г- 3± V5 которое имеет корни г/] = 4, у2 = 3. Тогда у 2 = 2 ± л/3 , у 4 —-— • Если ученик при решении данного уравнения догадался, что необ- ходимо разделить обе части уравнения нах2, но дальше продвинуться в решении не смог, то можно дать ему 1 балл; а если догадался о вве- дении новой переменной, ввел ее, а дальше продвинуться не смог, то можно оценить его решение уже в 4 балла. В 5 баллов можно оценить решение данного уравнения, если ученик остановился на решении уравнения у2 - 7 у +12 = 0, а корни его обозначил по невниматель- ности как у, у. 6 баллов можно дать за решение, в котором допущена вычислительная ошибка в нахождении корней уравнения. Абсолют- но правильное решение оценивается в 7 баллов. Пример 3. В каждую клетку квадратной таблицы 25 х 25 вписано произвольным образом одно из чисел: 1 или -1. Под каждым столб- цом пишется произведение всех чисел в этом столбце. Справа от каждой строки пишется произведение всех чисел в этой строке. До- кажите, что сумма 50 написанных произведений не может оказаться равной нулю. Решение: Перемножая все 50 произведений, мы получим 1, так как в каждое произведение любое из чисел войдет 2 раза. Тогда в 50 сомножите- лей будет входить четное число произведений с «-1», а поэтому сум- ма четного числа произведений с «1» и четного числа произведений с «-1» не будет равна 0 (25 — число нечетное, значит, одинакового числа слагаемых не будет).
28 Математические олимпиадные работы. 5-11 классы В 7 баллов оценивается решение, где есть все обоснования. За правильный ответ без какого бы то ни было обоснования предлага- ется ставить 0 баллов. Решение задачи, в котором есть основная идея о том, что надо перемножить все 50 произведений, можно оценить в 2 балла. Если же ученик дополнительно использовал понятия чет- ности и нечетности, но обосновать полностью ответ не смог, то такое решение можно оценить в 4 балла. ПРИМЕЧАНИЕ Данная задача относится к числу таких, решение которых трудно оценить други- ми баллами. Как показали итоги ее решения, чаще всего ученики получали 0 или 7 (что случалось крайне редко) баллов. Пример 4. На окружности выбраны диаметрально противополож- ные точки Л и В и отличная от них точка С. Касательная к окруж- ности в точке А и прямая ВС пересекаются в точке D (рис. 1). Дока- жите, что прямая, касающаяся окружности в точке С, делит пополам отрезок AD. Рис. 1 Решение: Пусть МС пересекается с AD в точке Е. Тогда АЕ = СЕ, как отрезки касательных, проведенных к окружности из одной точки. Выполним дополнительное построение: проведем АВ и ОС. Так как ОВ = ОС, то Z ОВС = Z ОСВ. Так как прямые ОС и СЕ перпендикулярны, то
Проведение математической олимпиады и подведение ее итогов 29 Z ECD = Z ВСМ= 90° - Z ОСВ = 90° - Z ОВС. Рассмотрим треуголь- ник ABD: Z BDA = 90° - Z ОВС, поэтому Z BDA = Z ECD, а значит, треугольник ECD равнобедренный (по признаку равнобедренного треугольника), поэтому СЕ = DE. А, значит, АЕ = ED. Решение задачи оценивается в 4 балла, если ученик верно выпол- нил дополнительное построение, доказал равенство отрезков АЕи DE, но не сделал промежуточных обоснований (их здесь много), в 5 бал- лов — если доказал требуемое, по не обосновал одного-двух выводов. Если ученик обосновал равенство отрезков АЕ и СЕ (или углов ОСВ и ОВС), то можно дать ему 1 балл, если же он еще дополнительно доказал равенство углов, которые указаны в решении, то можно уже такое решение оценить в 3 балла. В 7 баллов оценивается решение со всеми обоснованиями. Пример 5. Сколькими нулями оканчивается произведение: 1 х 2 х х 3 х 4 х ... х 1999 х 2000 х 2001 х 2003? Решение'. Так как в произведении содержится (400 + 80 + 16 + 3) пятерки, то произведение оканчивается 499 нулями. Верное решение со всеми обоснованиями оценивается в 7 баллов. Если ученик догадался, что число нулей зависит от числа пятерок, но дальше продвинуться не смог, такое решение можно оценить в 2 бал- ла. Решение, в котором подсчитано лишь число нулей, получающееся от перемножения чисел с нулями на конце, оценивается лишь 1 бал- лом. Других вариантов оценивания может и не быть. Пример 6. Можно ли разбить равносторонний треугольник на 2002 равносторонних треугольника (возможно, не все из них равны между собой)? Если можно, то как? Если нет, то объясните, почему? Решение: Равносторонний треугольник на 2002 равносторонних треуголь- ника разделить можно. План построения: 1 Отложить AM =------АВ. 1001 1. Провести MN параллельно АС. 2. На АС отложить 1001 отрезок, равный AM. 3. Из точек Ар Л2, ... А1000 провести прямые, параллельные пря- мой АВ. Как видно на рис. 2, в полосе получается 2001 треугольник, все они равносторонние и все равны между собой, а 2002-й треуголь- ник — это &MBN.
30 Математические олимпиадные работы. 5-11 классы Рис. 2 Неверный ответ или правильный ответ без рисунка и обоснова- ний нужно оценивать в 0 баллов. Если ученик дал правильный от- вет и догадался, что в полосе 2001 треугольник, то можно дать за это решение 4 балла. Если все построения объяснены, есть чертеж, но не обосновано, почему получаются равные треугольники, то решение можно оценить в 5 баллов. В случае правильного решения задачи со всеми обоснованиями ставится 7 баллов. Пример 7. Разделите прямоугольник 3 х 4 на две равные части. Найдите как можно больше способов. Резать можно лишь по стороне квадрата 1 х 1, а способы считаются разными, если фигуры в каждом способе получаются разные. Решение: Всего существует 5 вариантов (рис. 3). а б в г д Рис.З За все найденные варианты решения ставится 7 баллов, за все най- денные решения, но неточные построения — 6 баллов. Если найдено 4 решения, это оценивается в 5 баллов, а если всего 3 — то в 4 бал- ла. За 1 найденное решение ставится 1 балл, за 2 — 2 балла, если это случай а, в или г (наиболее простые случаи), или 3 балла, если среди решений есть случай б или д (наиболее трудные случаи).
Проведение математической олимпиады и подведение ее итогов 31 Конечно, предложенные варианты оценки заданий олимпиады яв- ляются примерными. Число баллов ставит жюри, им виднее. Главное, чтобы при проверке работ учащихся оценивалась деятельность уча- щихся, их идеи (хотя и не доведенные до конца), а не только правиль- ность и неправильность решений. Тогда в протоколах олимпиад не будет оценок лишь в 0 и 7 баллов. После проверки всех работ лучшие работы еще раз перепроверя- ются несколькими членами жюри для их более объективной оценки. После выяснения всех спорных вопросов, проставления итоговых баллов за каждое задание и подсчета общего числа баллов заполня- ется протокол олимпиады. Фамилии учащихся вписываются только после заполнения всех остальных столбцов. Примерная форма вы- глядит следующим образом: Протокол проведения районной олимпиады среди учащихся 8 классов школ Каргопольского района Архангельской области 14.11.09 г. Председатель жюри 8 класса: Члены жюри: ПРИМЕЧАНИЕ Иногда столбец «число баллов за каждое задание» отсутствует. На взгляд ав- тора, это не совсем правильно, так как наличие данного столбца позволит рас- считать трудность каждого из заданий, увидеть, какие задания не могут решать учащиеся района, конкретной школы. Математическая олимпиада отличается от спортивных соревно- ваний тем, что здесь победителей и призеров может быть больше чем по одному на параллель. Как бы жюри ни оценивало задания, от субъективного мнения все равно не избавиться. Поэтому можно поре-
32 Математические олимпиадные работы. 5-11 классы комендовать примерно такие границы для определения победителей и призеров олимпиад: в соответствии с Положением об олимпиаде участники, набравшие максимальное число баллов, признаются по- бедителями, но при условии, что они набрали более 50 % баллов. Та- ким образом, можно порекомендовать следующий вариант определе- ния победителей муниципального этапа математической олимпиады. 1 место присуждается тем учащимся, которые решили наиболь- шее число задач (например, 4 или 5 из предлагавшихся 5-6). Хотя у них останется различие в количестве набранных баллов, как пра- вило, это будет более 75 % (60) баллов от максимально возможного числа или в крайнем случае не менее 50 %. Если все участники набра- ли менее 50 % баллов, то победителей может и не быть. Однако здесь уже возникают вопросы к составителям текстов. Призерами олим- пиады признаются все участники олимпиады, следующие в итоговой таблице за победителями. Как правило, 2 место можно присудить тем участникам олимпиады, которые набрали на 5-10 баллов мень- ше, чем победители, то есть 60-75 % (50-60) от максимального числа баллов. КЗ место можно присудить тем участникам, которые набра- ли меньше, чем занявшие 2 место, на 5-10 баллов или 50-60 % (40- 50) от максимального числа баллов. Квота на число победителей и призеров определяется организато- ром муниципального этапа олимпиады. Как правило, это около 20 % от числа участников, но для 5-6 классов этот процент можно увели- чить до 40. Также можно поощрить учащихся, занявших 4-5 места, специаль- ными дипломами, особенно это целесообразно в 5-6 классах. Рассмотрим конкретный пример. Допустим, что среди 30 участников олимпиады результаты луч- ших будут: 30, 24, 23, 22, 19, 16 и т. д. баллов (из 35 максимально воз- можных). Тогда в соответствии с предложенными границами и тем, что разница в 1-2 балла не является существенной (все же результа- ты оценивают люди), на 1 место можно вывести ученика, набравшего 30 баллов, на 2 место — трех учеников, набравших 24, 23 и 22 балла, на 3 место — ученика, набравшего 19 балов. Всего получается 5 при- зеров. В практике подведения итогов математических олимпиад ино- гда случаются и курьезные случаи, когда максимальное число баллов набирает пять и более человек (такой случай произошел и с автором данной книги, когда он был в числе одного из пяти победителей рай- онной олимпиады в 5 классе). Как быть? Конечно, всем присвоить первое место. Ученики не виноваты в том, что текст олимпиады не вполне соответствовал требованиям.
Проведение математической олимпиады и подведение ее итогов 33 Жюри может установить и поощрительные места, призы. Напри- мер, за самое оригинальное решением какой-то задачи, самому моло- дому, самому опытному участнику и т. д. Жюри может также подвести итоги официального или неофици- ального первенства между школами. Желательно в тот же день провести награждение победителей (особенно это касается районного тура); а участникам, пока жюри проверяет работы, предложить развлекательную программу. Жела- тельно было бы организовать горячее питание или работу буфета. В качестве развлекательной программы можно предложить КВН, концерт, дискотеку, экскурсию по городу и т. п. Все хорошо к ме- сту. Отсрочивать подведение итогов нежелательно: кроме лишней нервотрепки для учащихся, много неприятного может быть и для самих учителей. Начинаются выяснения: а почему у такого ученика столько-то баллов, а у такого-то столько? Необходимо знать, что ни- какие апелляции по олимпиадам после подписания протокола и при- нятия решения о призерах и победителях не принимаются. В качестве поощрений победителям и призерам вручаются дипло- мы районного (городского) отдела образования. Также можно вру- чить и призы. В качестве призов могут быть предложены книги. Победители и призеры муниципального этапа олимпиады соглас- но Положению об олимпиаде могут принять участие в региональном этапе Олимпиады, если они являются учениками 9-11 классов. Хотя, как правило, чаще всего приезжают на региональный этап только по- бедители, да и то не всегда. Если же региональный этап проводится для учащихся более младших классов, то состав его участников опре- деляет организатор олимпиады. Само награждение победителей и призеров олимпиады желатель- но провести в одном из лучших помещений школы, в которой про- водится олимпиада. На эту торжественную часть можно пригласить спонсоров, выдающихся деятелей науки, работников различных ор- ганизаций, которые были в школьные годы победителями районных олимпиад. Желательно итоги районных олимпиад осветить и в районной прессе. В практике подведения итогов олимпиад встречается и такой под- ход, когда победителей всех предметных олимпиад в городе пригла- шают на торжественное специальное мероприятие, где и происходит их чествование. Некоторые школы для победителей и призеров олим- пиад устанавливают денежные премии, которые вручаются учащим- ся в конце года или полугодия.
Примерные тексты математических олимпиад для муниципального этапа Приведенные далее варианты текстов для математических олим- пиад в основном составлены автором лично. По большинству из них проводились муниципальные олимпиады. При включении в пособие часть задач была изменена, в особенности это касается задач, в усло- вии которых фигурировал год. Остальные тексты составлены на основе текстов математических олимпиад второго этапа Всероссий- ской олимпиады, проходившего в различных регионах России. Все эти тексты были переработаны автором для того, чтобы соблюсти сформулированные во введении требования к текстам математиче- ских олимпиад, а также для того, чтобы автора не упрекали в исполь- зовании разработанных кем-то материалов. 5 класс Вариант 1 Вычислить: 2010 - 2009 + 2008 - 2001 +... + 2-1 2010 45 -4- 55 2010
Примерные тексты математических олимпиад для муниципального этапа 35 2. В квартирах 1, 2 и 3 жили три котенка: белый, черный и ры- жий. В квартирах 1 и 2 жил не черный котенок. Белый котенок жил не в квартире 1. В какой квартире жил каждый котенок? 3. Для нумерации детской книги понадобилось 204 цифры. Сколько страниц в книге, если нумерация книги начинается с первой страницы? 4. Разрежьте квадрат размером 4 х 4 на 4 равные фигуры (рис. 4). Резать можно лишь по стороне квадрата 1 х 1, и способы счи- таются разными, если в каждом случае получаются разные фи- гуры. Рис. 4 5. Когда у рыбака спросили, как велика пойманная нм щука, он сказал: «Я думаю, что хвост ее — 1 кг, голова — столько, сколь- ко хвост и половина туловища, а туловище — сколько голова и хвост вместе». Сколько весит щука? 6. Папа купил на праздник своим детям коробку конфет. Федя взял половину конфет и половинку одной конфеты, Аня взяла половину остатка и еще^ полконфеты. Коля взял половину но- вого остатка и еще полконфеты. Маша взяла половину остав- шихся конфет и еще полконфеты. После этого в коробке оста- лась одна конфета. Сколько конфет было в коробке? Вариант 2 Не выполняя умножения, найдите частное: (1003 • 2009 - 1002) : (1003 + 2009 • 1002).
36 Математические олимпиадные работы. 5-11 классы 2. У Кенгуру насморк. Он пользуется квадратными платками размером 25 х 25 см. За восемь дней Кенгуру израсходовал 3 м2 ткани. Сколько платков в день тратил Кенгуру? 3. Восстановите ребус: . КИС КСИ иск Одинаковым буквам должны соответствовать одинаковые цифры, а разным буквам — разные цифры. 4. Две чашки и два кувшина весят столько, сколько 14 блюдец. Один кувшин весит столько, сколько одна чашка и одно блюд- це. Сколько блюдец уравновесят кувшин? 5. Три друга — Винни-Пух, Пятачок и Кролик — пошли гулять в красной, зеленой и синей рубашках. Их туфли были тех же цветов. У Винни-Пуха цвет рубашки и туфель совпадали, у Пятачка ни туфли, ни рубашка не были красными, а Кролик был в зеленых туфлях. Как были одеты друзья? 6. Разделите квадрат 5x5 клеток с вырезанной центральной клет- кой на четыре равные части. Найдите как можно больше спосо- бов. Резать можно только по сторонам квадратов. Вариант 3 1. С двух станций, расстояние между которыми 25,6 км, одновре- менно в одном направлении вышли два поезда. Впереди дви- гался поезд со скоростью 58,4 км/ч и через 4 часа его догнал второй поезд. Найти скорость второго поезда. 2. Составьте из цифр число 2010, используя только знаки ариф- метических операций. н 3. До царя Гороха дошла молва, что кто-то из троих богатырей убил Змея Горыныча. Царь приказал всем троим явиться ко двору и молвили они: Илья Муромец: «Змея убил Добрыня Никитич»,. Добрыня Никитич: «Змея убил Алеша Попович». Алеша Попович: «Я убил Змея». При этом оказалось, что один из них сказал правду, а двое слу- кавили. Кто убил Змея?
Примерные тексты математических олимпиад для муниципального этапа 37 4. Расшифруйте следующую запись примера на сложение, в ко- тором разным буквам соответствуют разные цифры, а одина- ковым буквам — одинаковые: . СПОРТ СПОРТ КРОСС 5. Лев поручил лисе посчитать, сколько в лесу медведей, зайцев и волков. После подсчета лиса доложила, что всего медведей, зайцев и волков в лесу 100, по волков на 25 больше, чем медве- дей, а зайцев на 30 больше, чем волков. Один из зайцев, услы- шав такой ответ, расхохотался и сказал, что такого быть не мо- жет. Кто прав, лиса или заяц, и почему? 6. Вычислите без калькулятора: 89089089089 • 7373 - 73073073073 • 8989. Вариант 4 1. Найти значение выражения: 2011 - 2010 + 2009 - 2008 + 2007 - ... - 2 + 1. 2. Расшифруйте пример: ББ + А __А ССС 3. Разделите прямоугольник 3 х 4 на две равные части. Найдите как можно больше способов. Резать можно лишь по стороне квадрата 1 х 1, а способы считаются разными, если в каждом случае получаются разные фигуры. 4. Илья Муромец, Добрыня Никитич и Алеша Попович вступи- ли в бой с великанами. Получив по три удара богатырскими палицами, великаны обратились в бегство. Больше всего уда- ров (7) нанес Илья Муромец, меньше всех (3) — Алеша Попо- вич. Сколько всего было великанов? 5. Митя, Сеня, Толя, Юра и Костя пошли на концерт и встали в очередь. Если бы Митя встал посередине очереди, то он бы
38 Математические олимпиадные работы. 5-11 классы оказался между Сеней и Костей, а если бы Митя встал в конец очереди, то рядом с ним мог бы быть Юра, но Митя встал впе- реди всех своих товарищей. Кто за кем стоит? Вариант 5 1. С помощью четырех четверок и знаков действий запишите все цифры от 0 до 9. 2. В примере на сложение цифры заменили буквами: одинако- вые — одинаковыми, разные — разными. Получилось АБББ + + А = ВГГГ. Восстановите пример. Объясните, почему это мож- но сделать только одним способом. 3. Дядя Федор, кот Матроскин, Шарик и почтальон Печкин си- дят на скамейке. Если Шарик, сидящий справа от всех, сядет между дядей Федором и котом, то кот окажется крайним слева. В каком порядке они сидят? 4. Если бы школьник купил И тетрадей, то у него осталось бы 500 руб. А на 15 тетрадей у него не хватает 700 руб. Сколько денег было у школьника? 5. Как разрезать прямоугольник, длина которого 16 см, а ширина 9 см, на две равные части, из которых можно составить квадрат? 6. Могут ли десять игрушек стоимостью 105, 211 и 233 руб. сто- ить в сумме 2009 руб.? Вариант 6 1. Как разрезать квадрат со стороной 30 м 15 см на 2010 одинако- вых треугольников? 2. На острове Буяне находятся четыре королевства, причем каж- дое граничит с тремя остальными. Нарисуйте карту острова так, как вы ее представляете. 3. Напишите самое маленькое четырехзначное число, которое при делении на 6 дает в остатке 5. 4. На книжной полке стоят тома в следующем порядке: 1, 2, 6, 10, 3, 8, 4, 7, 9 и 5. Расставьте их по порядку с первого по десятый, но брать можно лишь по два соседних тома и ставить их вместе на другое место (не разъединяя). Выполните задание, переста- вив всего три пары.
Примерные тексты математических олимпиад для муниципального этапа и 39 5. На листе бумаги нарисован круглый циферблат часов с 60 точ- ками, обозначающими минуты. Двое поочередно проводят от- резки прямых, соединяющих две произвольные точки. Не раз- решается проводить отрезок, пересекающий другие отрезки, но они могут иметь разные концы. Проигрывает тот, кто не сумеет провести отрезок. Кто выиграет при правильной игре — начи- нающий или его партнер? Вариант 7 1. Запишите подряд 19 семерок: 7777...7. Поставьте между не- которыми цифрами знаки арифметических действий «+» или «-» так, чтобы в результате получилось число 2009. 2. Внутренние покои султана состоят из 100 одинаковых ква- дратных комнат, расположенных в виде квадрата 10x10. Если у двух комнат есть общая стена, то в ней обязательно есть ров- но одна дверь. Сколько дверей во дворце? 3. Ширина прямоугольного параллелепипеда в 4 раза меньше вы- соты, а высота меньше длины в 2 раза. Вместе длина и высота составляют 60 см. Найдите объем прямоугольного параллеле- пипеда. 4. В примере цифры заменены буквами, причем одинакрвые циф- ры одинаковыми буквами. Расшифруйте пример: . ОДИН один много 5. Как набрать из озера 8 литров воды, имея одно девятилитровое и одно пятилитровое ведро? и Вариант 8 л 2012-2011 + 2010- 2009 + ... + 4- 3 + 2-1 1. Вычислите:--------------------------——------. 2012 73 -+ 2012 27 2. Ваня купил 4 книги для подготовки к олимпиаде по матема- тике. Все книги, кроме первой, стоят в сумме 348 руб., без вто- рой -- 296 руб., без третьей — 292 руб., без четвертой — 288 руб. Сколько стоит каждая книга?
40 Математические олимпиадные работы. 5-11 классы 3. Для нумерации страниц книги понадобилось 183 цифры. Сколько страниц в книге? 4. Медведь, Волк и Лиса разговаривали на поляне. Медведь: «Лиса не самая хитрая», Лиса: «Я хитрее Медведя», Волк: «Лиса хитрее меня». Двое зверей сказали правду, а самый хи- трый соврал. Кто самый хитрый из зверей? 5. Как разрезать прямоугольник длиной 18 см и шириной 8 см на наименьшее число прямоугольников и сложить из них квадрат? 6. Имеется 10 мешков монет. В девяти мешках монеты настоящие (по 10 г), а в одном — фальшивые (по 11 г). Как одним взвеши- ванием на обычных весах определить, в каком мешке фальши- вые монеты? Вариант 9 1. Найдите сумму всех трехзначных чисел, которые можно запи- сать с помощью цифр 1, 2 и 3 так, чтобы в каждом числе все цифры были разными. 2. Два Муравья отправились в гости к Стрекозе. Один всю доро- гу полз, а второй первую половину пути ехал на Гусенице, что было медленнее в 2 раза, чем ползти, а вторую половину пути скакал на Кузнечике, что было в 10 раз быстрее. Какой Мура- вей первым пришел в гости, если они вышли одновременно? 3. Мальчик нарисовал пять лучей с началом в точке О (рис. 5). Сколько всего получилось острых углов? 4. Восстановите пропущенные цифры в примере: *0*3 х * * * 2**** ***6 621 1
Примерные тексты математических олимпиад для муниципального этапа 41 5. В таблице 4 х 4 за один ход можно поменять все знаки в любой строке или столбце на противоположные. Можно ли таким об- разом из таблицы, приведенной на рис. 6, получить таблицу из одних плюсов? + + + - - - - + + + - — - - + Рис. 6 Вариант 10 1. Существует ли натуральное число, которое при делении на 6 дает остаток 2, а при делении на 4 — остаток 3? 2. Одна бутылка лимонада стоит 30 руб. Пустую бутылку можно сдать за 12 руб. Какое наибольшее число бутылок лимонада можно выпить, имея 100 руб.? 3. Произведение цифр трехзначного числа равно 3. Найдите все такие числа. 4. Рост Буратино составляет 1 м 4 дм, а длина его носа раньше была 9 см. Каждый раз, когда Буратино обманывал, длина его носа удваивалась. Как только длина его носа стала больше его роста, Буратино перестал обманывать. Сколько раз он обма- нул? 5. У Коли есть карандаш и фанерный прямоугольник со сторо- нами 3 см и 5 см. Разрешается прикладывать прямоугольник к бумаге и обводить его (полностью или частично) каранда- шом. Любые другие действия (например, делать пометки на фанере) запрещены. Как Коле, не нарушая запрета, нарисовать квадрат со стороной 1 см? Опишите, что и в каком порядке он должен делать.
42 Математические олимпиадные работы. 5-11 классы 6. На первохМ заседании городской Думы присутствовало 80 % списочного состава, а на втором — всего 70 %. Сколько процен- тов депутатов присутствовало на обоих заседаниях? б класс Вариант 1 1. Найдите значение выражения: 2009 - 2007 + 2005 - 2003 + 2001 - 1999 + ... + 9 - 7 + 5 - 3 + 1. 2. В бассейне с горизонтальным дном размером 20 м на 50 м на- ходится 100 000 л воды. Можно ли в этом бассейне проводить соревнования по плаванию? 3. Восстановите ребус: КОШКА + КОШКА КОШКА СОБАКА Одинаковым буквам должны соответствовать одинаковые цифры, а разным буквахМ — разные цифры. 4. Как-то в минуту отдыха мушкетеры Атос, Портос, Арамис и д'Артаньян решили помериться силой, перетягивая канат. Портос с д'Артаньяном легко перетянули Атоса с Арамисом. Но когда Портос встал в паре с Атосом, то победа против Ара- миса с д'Артаньяном досталась им не так легко. Когда же Пор- тос с Арамисом оказались против Атоса с д'Артаньяном, то ни одна из пар не смогла одолеть друг друга. Можно ли опреде- лить, как распределены силы мушкетеров? 5. Отец старше сына в 4 раза, при этом суммарный их возраст составляет 50 лет. Через сколько лет отец станет старше сына в 3 раза? 6. Бабушка дала каждому внуку по несколько яблок и груш, при- чем всем досталось поровну фруктов. Внуку Пете досталась пятая часть всех яблок и седьмая часть всех груш. Сколько вну- ков у бабушки? Ответ объясните.
Примерные тексты математических олимпиад для муниципального этапа 43 Вариант 2 1. Запишите подряд 22 пятерки: 5 5 5 5 ... 5. Поставьте между не- которыми цифрами знаки арифметических действий так, что- бы в результате получилось число 2004. 2. Из представленных на рис. 7 кубиков выберите три, которые могли бы являться изображениями одного и того же кубика. а б в г д Рис. 7 3. Буратино отпил полчашки черного кофе и долил ее молоком. 1 Потом он отпил — чашки и долил ее молоком. Потом он отпил , 3 — чашки и долил ее молоком. Наконец, Буратино допил со- 6 держимое чашки до конца. Чего Буратино выпил больше: кофе или молока? 4. Как разрезать квадрат на: а) 6 квадратов; б) 7 квадратов; в) 8 квадратов? 5. В пакете 9 кг крупы. Как при помощи чашечных весов и одной 200-граммовой гири Отвесить 2 кг крупы, если разрешается сделать только три взвешивания? 6. В древней рукописи приведено описание города, расположен- ного на 8 островах. Острова соединены между собой и с мате- риком мостами. На материк выходят 5 мостов, на 4 островах берут начало по 4 моста, на 3 островах берут начало по 3 моста и на один остров можно пройти только по одному мосту. Воз- можно ли такое расположение мостов?
44 II Математические олимпиадные работы. 5-11 классы Вариант 3 4 . 23232323 1. Сократите дробь-------. 1 1 61616161 2. Счетчик автомобиля показывал 12 921 км. Через два часа счет- чик стал показывать число, которое одинаково читалось в обо- их направлениях. С какой скоростью ехал автомобиль? 3. В рисе содержится 75 % крахмала, а в ячмене — 60 %. Сколько надо взять ячменя, чтобы в нем содержалось столько крахмала, сколько его содержится в 9 кг риса? 4. Докажите, что любую сумму свыше 8 тугриков (тугрик — де- нежная единица страны Бинии) можно набрать, используя лишь монеты достоинством в 3 и 5 тугриков. 5. В классе 30 учеников. Найдется ли месяц, в котором отмечают свои дни рождения не меньше чем 3 ученика этого класса? 6. Какой цифрой оканчивается число 22011? Вариант 4 1. Вычислите: _________666666 - 666666________ 1 + 2 + 34-4 + 5 + 6 + 5 + 4 + 3-1-2 + 1 _____________777777-777777____________ 1 + 2 + 3 + 4 + 5 + 6 + 7 + 6 + 5 + 4 + 3 + 2 + 1 2. Сережа пошел с отцом в тир. Уговор был такой: Сережа делает 5 выстрелов и за каждое попадание в цель получает право сде- лать еще 2 выстрела. Сережа сделал 17 выстрелов. Сколько раз он попал в цель? 3. В ряд выписано 13 девяток: 999999999999 9. Поставьте между ними знаки 4-, -, : , х и скобки так, чтобы получилось число 2009. 4. Кросс осенний вспоминая, Спорят белки два часа: Победил в забеге заяц, Л второй была лиса! — Нет — твердит другая белка —
Примерные тексты математических олимпиад для муниципального этапа 45 Ты мне шутки эти брось. Заяц был вторым, конечно, Первым был, я помню, — лось. — Я, — промолвил филин важный, — В спор чужой не стану лезть. Ноу вас в словах у каждой по одной ошибке есть. Белки фыркнули сердито. Неприятно стало им. Вы уж, взвесив все, решите, Кто был первым, кто вторым. 5. Можно ли выбрать 5 чисел из показанной на рис. 8 таблицы, сумма которых делилась бы на 20? 1 1 1 3 3 3 5 5 5 1 7 7 Рис. 8 6. Мама купила яблок и сказала детям, чтобы они, вернувшись из школы, разделили их поровну. Первым пришел Андрей, взял треть яблок и ушел. Вторым пришел Борис, взял треть остав- шихся яблок и ушел. Затем вернулась из школы Валя, она взяла 4 яблока — треть от числа яблок, которые она увидела. Сколько яблок оставила мама? Вариант 5 1. В ряд выписано 13 двоек: 222222222222 2. Поставьте меж- ду ними знаки +, -,: и х так, чтобы получилось число 2011. п п ,1111 2. Докажите, что между числами 1, —, —иельзя расставить знаки + и - так, чтобы значение выражения было равно 0.
46 Математические олимпиадные работы. 5-11 классы 3. Три купца должны поделить между собой 21 бочонок, из ко- торых 7 бочонков полны кваса, 7 полны наполовину и 7 пус- ты. Спрашивается, как они могут поделиться так, чтобы каждый получил одинаковое количество кваса и одинаковое число бочонков, причем квас переливать из бочонка в бочо- нок нельзя. 4. Ширина прямоугольного параллелепипеда больше единицы, длина больше высоты, а высота больше ширины. При этом дли- на, ширина и высота — натуральные числа, а объем нескольких таких параллелепипедов равен 315. Найдите площадь поверх- ности этого параллелепипеда. 5. В гостинице, имеющей форму квадрата, расположено 9 комнат одинаковых размеров. Каждый из всех 9 постояльцев недово- лен своей комнатой и считает, что любая комната через стенку лучше, чем та, в которой он живет. Может ли хозяйка гостини- цы переселить их так, чтобы каждый постоялец переехал в со- седнюю комнату? Вариант 6 1. Вместо звездочек поставьте такие знаки действий, которые сделают равенство верным: 9 2 (— *0,05)* 1,4 = -. 20 7 2. Во дворе стоят 5 домов, напротив них 5 калиток. Как от каждого дома пройти к калитке с таким же номером, при- чем провести тропинку так, чтобы тропинки не пересекались (рис. 9)? 2 4 1 3 5 ? Т Т ? ? Рис. 9
Примерные тексты математических олимпиад для муниципального этапа 47 3. В рыбоводческсхм хозяйстве для приближенного определения количества рыбы в пруду выловили и пометили 90 рыбин, а затем выпустили их обратно в пруд. Через несколько дней при следующем отлове рыбы среди пойманных 450 рыбин по- меченными оказались две. Подсчитайте, сколько всего рыбин в пруду (помеченные рыбы равномерно распределяются среди остальных). 4. Разрежьте квадрат, показанный на рис. 10, на 4 части одинако- 5. Кот Матроскин продает молоко через магазин и хочет по- лучить за литр 15 руб. Магазин удерживает 20 % стоимости проданного товара. По какой цене будет продаваться молоко в магазине? 6. В файле хранятся 2012 единиц и 1957 нулей. Программа читает из файла два произвольных числа, стирает и записывает вместо них их сумму. Программа запускается многократно. В конце концов, в файле остается только одно число. Какое? Вариант 7 1. Над имеющимся числом разрешается производить два дей- ствия: умножать его на 2 или прибавлять к нему 2. За ка- кое наименьшее число действий вы сможете получить чис- ло 100 из числа 1? 2. Делится ли па 3 число 10 020 + 1010 + 1?
48 Математические олимпиадные работы. 5-11 классы 3. Круглая поляна окружена деревьями. Мальчик и девочка пош- ли вокруг поляны, считая деревья. Они идут в одном направле- нии, но вышли из разных мест. Дерево, которое у девочки было седьмым, у мальчика оказалось двадцатым, а дерево, которое было у мальчика седьмым, у девочки оказалось девяносто тре- тьим. Сколько деревьев растет вокруг поляны? Ответ объяс- ните. 4. Сейчас папе вдвое больше лет, чем было маме тогда, когда папе было столько лет, сколько маме сейчас. Сколько сейчас лет маме и сколько лет папе, если вместе им сейчас 77 лет? 5. Можно ли прямоугольник размером 35 х 23 разрезать без остат- ка на прямоугольники размером 5x7? Если можно, то как? Если нельзя, то почему? 6. На столе лежат 8 монет, при этом три из них вверх орлом, а пять — решкой. Можно ли все монеты положить вверх орлом, если одновременно переворачивать по две монеты? Вариант 8 1. Найдите сумму: 1 1 1 —_ ------1--- 1-2 + 1- 2 + 3- 4+ + 2008 • 2009 + 2009 • 2010 2. В забеге на некоторую дистанцию участвуют черепаха и заяц, который бежит в 35 раз быстрее черепахи. Черепаха затратила на всю дистанцию 2 часа 20 минут. Если известно, что сопер- ники прибыли к финишу одновременно, то сколько времени после начала движения черепахи оставался на старте заяц, пре- жде чем побежал вслед? 3. Незнайка начертил 3 прямые линии. На каждой из них он от- метил 3 точки. Всего Незнайка отметил 6 точек. Как он это сде- лал? Покажите на рисунке. объясните. 5. Хозяин имел двор квадратной формы. В четырех углах двора он посадил по дереву. Прошло время, и он решил увеличить площадь двора в 2 раза, но так, чтобы двор сохранил форму
Примерные тексты математических олимпиад для муниципального этапа 49 квадрата, а деревья росли на линии ограды. Покажите на ри- сунке, как он это должен сделать. 6. Четыре коровы черной масти и 3 коровы рыжей масти за 5 дней дали столько же молока, сколько 3 коровы черной масти и 5 ко- ров рыжей масти за 4 дня. Какие коровы более продуктивны (то есть дают молока больше за 1 день), черные или рыжие? Вариант 9 . т. , 4 5 6 7 1. Расставьте числа в порядке убывания: —, —, ~, —. 2. Космические корабли-спутники летают на высоте от 300 до 400 км. Хватит ли кубиков объемом в 1 мм3, содержащихся в 1 м3, чтобы сложить башню высотой 400 км? 3. В ящике лежат лимоны. Сначала из него взяли половину всех лимонов и половину лимона, затем половину остатка и еще по- ловину лимона, наконец, половину нового остатка и опять по- ловину лимона. После этого в ящике остался 31 лимон. Сколь- ко лимонов было в ящике вначале? 4. Замените буквы цифрами: ПА2 = ПИЛА. Укажите все решения. 5. Сколькими нулями заканчивается произведение всех чисел от 1 до 2011? 6. Может ли Карлсон на спор с Малышом обойти шахматным конем всю шахматную доску размером 7x7 так, чтобы конь побывал на каждой клетке по одному разу и вернулся на на- чальную клетку? Вариант 10 1. Закрасьте восемь одинаковых клеток так, чтобы каждая из за- крашенных клеток имела по две соседние закрашенные клетки. 2. Виноград содержит 40 % воды, а получаемый из него изюм — всего 4 %. Сколько винограда было использовано, если изюма получилось 45 кг? 3. Крокодил Гена с Чебурашкой плыли вверх по течению реки. Гена сидел на веслах, а Чебурашка, сидя на корме, ел апельси- ны. В момент, когда лодка проплыла под мостом, а Крокодил Гена был поглощен движением, Чебурашка заснул и нечаян-
50 я Математические олимпиадные работы. 5-11 классы но столкнул деревянный ящик с оставшимися апельсинами в воду. Гена обнаружил пропажу ящика только через полчаса. Он развернул лодку по течению реки и стал догонять уплываю- щий ящик. Через полчаса Гена догнал ящик на расстоянии 3 км ниже моста по течению реки. Какова скорость течения реки? 4. Семья из четырех человек ночью подходит к мосту (с одной стороны) и собирается перейти через него. У них только один фонарик, без которого невозможно и шагу ступить. Мост вы- держивает только двух человек. Папа может перейти мост за 1 минуту, мама — за 2 минуты, сын ~ за 5 минут, дочь — за 8 минут. Как им всем перейти мост за 15 минут? 5. Расшифруйте следующую запись при условии, что наиболь- шая цифра в записи числа «СИЛЕН» равна 5: РЕШИ + ЕСЛИ СИЛЕН 6. По кругу написано 2011 натуральных чисел. Докажите, что найдутся два соседних числа, сумма которых четная. 7 класс Вариант 1 1. Какой угол образуют часовая и минутная стрелки в 12 часов 40 минут? 2. Какое из чисел, 7777777773 8888888882 ---------- или------------ 7777777778 8888888887 больше? Ответ объясните. Микрокалькулятор применять нельзя. 3. Выясните, делится ли на 3 число 1 + 2 -ь 22 + ...+ 22011 + 22012. 4. Одна из цифр четырехзначного натурального числа равна нулю. При вычеркивании нуля это число уменьшается в 9 раз. На каком месте стоит нуль? Найдите все такие числа. 5. Учитель математики, проверив контрольные работы у трех друзей: Алексея, Бориса и Василия, сказал им: «Все вы напи- сали работу, причем получили разные отметки (“3”, “4”, “5”)-
Примерные тексты математических олимпиад для муниципального этапа и 51 У Василия не “5”. у Бориса не “4”, а у Алексея, по-моему “4”». Впоследствии оказалось, что учитель ошибся: одному ученику сказал отметку верно, а другим двум — неверно. Какие отмет- ки получил каждый из учеников? Вариант 2 1. 16 Вычислите 8 ~^х2 У3 ~~ 0,02 х3 у2, если х равен наибольшему це- лому числу заключенному между числами --9,3 и —15.1, а у — наименьшему простому числу в третьем десятке натуральных чисел. 2. Если товар подорожает сначала на 10 %, а затем подешевеет на 10 %, то когда его цена будет ниже — до подорожания или после снижения? 3. Игра-лотерея проводится следующим образом. Выбирается случайное число от 1 до 1000. Если оно делится на 2, платят 100 руб.; если делится на 10 — 200 руб.; если на 12 — 500 руб.; если на 20 — 1000 руб.; если оно делится на несколько этих чи- сел, то платят сумму Сколько можно выиграть (за один раз) в такой игре? 4. Один из внешних углов равнобедренного треугольника ра- вен 32°. Найти угол между основаниехМ этого треугольника и высотой треугольника, проведенной из вершины угла при основании. 5. Чему равен угол между часовой и минутной стрелками в 7 ча- сов 38 минут. 6. В ряд выписано 13 девяток: 999999999999 9. Поставьте между ними знаки х и скобки так, чтобы получилось число 2009. Представьте как можно больше способов. Вариант 3 1. Дан угол 34°. Постройте угол в 5°. 2. Путник шел в гору со скоростью и км/ч, а с горы — 2v км/ч. Какова скорость путника, если он поднимался в гору и воз- вращался в исходный пункт у подножия горы по одной и той же тропинке?
52 Математические олимпиадные работы. 5—11 классы 3. Можно ли соединить 9 телефонов между собой попарно так, чтобы каждый был соединен ровно с тремя другими? 4. На сколько частей окажется разделенным прямоугольник, по- казанный на рис. И, если провести еще две его диагонали? Рис. 11 5. Делится ли число 2 • 12 • 22 • 32 • 42 • ... • 2002 • 2012 - 2 на 10? Почему? Вариант 4 1. Поезд из пункта А в пункт В шел со скоростью 60 км/ч, а воз- вращался со скоростью на 20 км/ч меньше. Какова средняя скорость поезда? 2. Пусть 12 человек несут 12 хлебов. Каждый мужчина — 2 хлеба, 1 Л ” 1 г каждая женщина — по — хлеба, каждый ребенок — по — хлеба. Сколько было мужчин, женщин и детей, если в переноске уча- ствовали все 12 человек? 3. Велосипедист проехал — пути и еще 40 км, после этого ему 15 осталось 0,75 пути без 118 км. Как велик путь? 4. Пол в комнате прямоугольной формы размером 20 м на 15 м нужно покрыть квадратными плитками со стороной 20 см. Сколько потребуется плиток? 5. Как разложить 8 монет в центры клеток доски 4x4, чтобы пи на одной прямой не лежало трех монет (монеты считать точеч- ными)?
Примерные тексты математических олимпиад для муниципального этапа 53 6. Докажите, что при любом натуральном числе п число 4л? + 5 де- лится на 3. Вариант 5 1. После семи стирок длина, ширина и толщина куска мыла уменьшились вдвое. На сколько таких же стирок хватит остав- шегося мыла? 2. Три брата имеют звания: капитан, старшина и сержант. Из трех утверждений: «Алексей — старшина», «Владимир — не стар- шина», «Семен — не сержант» лишь одно верное. Какое звание имеет каждый из братьев? 3. Существует ли такое натуральное число п, что п + п1 + и3 + ... + + „2009 = ^+ 1)2010? 4. Весь путь автобус ехал с неизменной скоростью. В первую часть пути автобус проехал столько километров, сколько ми- нут ему осталось ехать. Во вторую часть пути автобус проехал столько километров, сколько минут ехал первую часть пути. Какова скорость автобуса? 5. Вычислить наиболее рациональным способом: о 1 z 1 Л16 -118 5 117 119 117 119 119 6. На плоскости дан угол, градусная мера которого от 90 до 120°. Как проверить, используя только циркуль, равен ли этот угол 108°? Вариант б 1. Какие две цифры надо поставить вместо *, чтобы пятизначное число 510** делилось на 6, 7, 9? 2. Найдите все такие простые числа а и Л, для которых сумма а + b и разность а-b также являются простыми числами. 3. Из пунктов А и В одновременно навстречу друг другу выеха- ли велосипедист и мотоциклист. Через 30 минут велосипеди- сту оставалось проехать 3 км до середины пути. Мотоциклист же через 20 минут после начала движения уже отъехал на 2 км ' от середины пути. Через какое время после начала движения произошла их встреча?
54 Математические олимпиадные работы. 5-11 классы 4. Каждая точка плоскости окрашена либо в белый, либо в чер- ный цвет. Докажите, что найдутся две точки одного цвета, рас- стояние между которыми равно 1. 5. Буратино связал 17 кукол Мальвины лентами красного, сине- го и зеленого цвета так, что от каждой куклы к каждой другой кукле ведет ровно одна лента. Докажите, что найдутся 3 куклы, связанные лентами одного цвета. Вариант 7 1. У Алены есть 5 плиток шоколада. Может ли она, поделив каж- дую из плиток на 3, 5 или 7 кусочков, получить 30 кусков шо- колада? 2. 2009 Сравните числа----- F 2010 20092009 20102010 ’ 3. Морская вода содержит 5 % соли (по весу). Сколько килограм- мов пресной воды нужно добавить к 60 кг морской воды, чтобы содержание соли в полученной воде составило 4 %? 4. Разложите на множители многочлен х3 - 1х + 6. 5. Разрежьте показанную на рис. 12 фигуру, полученную из ква- драта 7x7 вырезанием четырех угловых клеток 1 х 1, на угол- ки, представленные на рис. 13 (уголки состоят из квадратиков размером 1x1) так, чтобы заштрихованные квадратики оказа- лись только в больших уголках. Рис. 12
Примерные тексты математических олимпиад для муниципального этапа 55 Вариант 8 (4 • З22 + 7 • З21) 57 1. Вычислите:-----------т-т---. (19 -274)2 2. Магазин продал одному покупателю 25 % имеющегося в куске полотна, второму покупателю — 30 % остатка, третьему — 40 % нового остатка. Сколько процентов полотна осталось непро- данным? 3. К числу 10 припишите слева и справа по одной цифре так, что- бы получилось число, кратное 72. 4. На плоскости имеется 5 точек с целыми координатами. Дока- жите, что середина одного из соединяющих их отрезков также имеет целые координаты. 5. Можно ли на доске 7 х 7 с вырезанными угловыми клетками разложить шнур так, чтобы он не проходил через вершины кле- ток и в каждой клетке побывал один раз? Вариант 9 2х —12 х I 1. Решите уравнение:____!_L = о. х- 1 2. В трех пакетах было 136 мандаринов. В первом пакете вдвое больше мандаринов, чем во втором, а во втором — на 8 ман- даринов больше, чем в третьем. Каждый мандарин первого пакета стоит 3 руб., а третьего — 5 руб. Сколько стоит каждый мандарин второго пакета, если, смешав все мандарины и про- давая каждый из них по 4 руб., можно получить 8,8 % прибыли по отношению к их стоимости?
56 Математические олимпиадные работы. 5-11 классы 3. Предположим, что 2009 человек выстроились в шеренгу. Всег- да ли можно их расставить по росту, если разрешается пере- ставлять только двух людей, стоящих через одного? 4. Путешественник в первый день прошел 20 % всего пути и еще 2 км. Во второй — 50 % остатка и еще 1 км. В третий день — 25 % оставшегося пути и еще 3 км. Остальные 18 км пути он прошел в четвертый день. Какова длина пути, пройденного пу- тешественником? 5. Выясните, для каких натуральных значений п число 7" - 1 де- лится на 5. 6. Как надо разрезать квадрат размером 4 х 4 на 3 части, что- бы из них можно было сложить тупоугольный треугольник (рис. 14)? Рис. 14 Вариант 10 1. У троих мальчиков есть некоторое количество яблок. Первый мальчик дает другим столько яблок, сколько каждый из них имеет. Затем второй мальчик дает двум другим столько яблок, сколько каждый из них теперь имеет; в свою очередь, и третий дает каждому из двух других столько, сколько есть у каждого в этот момент. После этого у каждого из мальчиков оказыва- ется по 8 яблок. Сколько яблок было у каждого мальчика вна- чале? 2. Какой вес должна иметь каждая из трех гирь для того, чтобы с их помощью можно было взвесить любое число килограммов от 1 кг до 10 кг на чашечных весах (гири можно ставить на обе чашки). Обоснуйте свой ответ. 3. Найти трехзначное число, куб которого оканчивается на 777.
Примерные тексты математических олимпиад для муниципального этапа 57 4. Все числа от 1 до 10 выписали в произвольном порядке. Затем каждое из этих чисел сложили с номером места, на котором оно находится в строке. Докажите, что по крайней мере у двух из полученных сумм совпадают последние цифры. 5. 6. тг Л 11 Что больше:-------h ---- 1001 1002 1 5 -----или — ? 2010 8 В квадрат 1x1 бросили 51 точку Докажите, что найдется пря- 13 О Q моугольник со сторонами — и — , содержащий не менее 3 точек. 8 класс Вариант 1 1. На какую цифру оканчивается число, равное сумме чисел З2008 + + 42009? 2. Решите ребус: УДАР г УДАР ДРАКА Одинаковыми буквами обозначают одинаковые цифры, раз- ными — разные. 3. Дан угол в 34°. Можно ли с помощью циркуля и линейки по- строить угол в 12°? Если да, то обоснуйте, если нет, тоже обос- нуйте. 4. Три брата, Александр, Борис и Сергей, преподают различные предметы в школах Архангельска, Северодвинска и Котласа. Александр работает не в Архангельске, а Борис — не в Северо- двинске. Архангелогородец преподает не математику. Тот, кто работает в Северодвинске, преподает химию. Борис преподает физику Какую дисциплину преподает Сергей и в школе какого города? 5. Аня младше Вани. Когда Ване было столько лет, сколько Ане сейчас, их матери было на 3 года меньше, чем Ане с Ваней те- перь. Сколько лет было Ване, когда матери было столько лет, сколько Ване теперь?
58 Математические олимпиадные работы. 5-11 классы Вариант 2 1. Число 2005 представьте в виде разности квадратов двух нату- ральных чисел. 2. Сумма двух натуральных чисел равна 777. Какое наибольшее значение может принимать общий делитель этих чисел? 3. В треугольнике АВС биссектрисы углов ВАС и АВС пересека- ются в точке О. Найти угол АСВ, если угол АОВ равен 125°. 4. В двух бочках было воды поровну. Количество воды в первой бочке сначала уменьшилось на 10 %, а затем увеличилось на 10 %. Количество воды во второй бочке сначала увеличилось на 10 %, а затем уменьшилось на 10 %. В какой бочке стало больше воды? 5. Имеются 132 гири весом 1,2, 3,4,... 132 г. Можно ли разложить эти гири на 3 кучки так, чтобы в каждой кучке было одинако- вое число гирь и вес каждой кучки был одинаков? Вариант 3 a b 1 1. Упростите выражение: —т-у + —----7 4- -. сС + b2 а2 - b2 Ь + а 2. На сторонах АВ, ВС, С А равностороннего треугольника АВС взяты соответственно точки D, Е, Гтак, что AD == BE = CF. Ка- ков вид треугольника DEF? Докажите. 3. Из города А в город В корабль плывет по реке одни сутки, а об- ратно ~ трое суток. За какое время можно добраться из города А в город В на плоту? 4. Какой цифрой оканчивается число 220И? 5. Как отложить угол 1°, зная угол в 19е? 1 30 6. Решите уравнение в натуральных числах: х 4--- = —. # + ~ z Вариант 4 1. Решите уравнение: |х + 2| = 2 • (3 - х) .
Примерные тексты математических олимпиад для муниципального этапа 59 2. Расшифруйте ребус, где одинаковым буквам соответствуют одинаковые цифры, а разным — разные: Яс = семья. 3. Двое рабочих могут выполнить некоторую работу за 7 дней при условии, что второй рабочий приступит к ней на два дня позже первого. Если бы ту же работу каждый выполнял в оди- ночку, то первому понадобилось бы на 4 дня больше, чем вто- рому. За сколько дней каждый рабочий мог бы выполнить эту работу, если известно, что число дней, необходимых каждому из них, —• целое? 4. Постройте угол в 5°, если дан угол в 34°. 5. В прямоугольнике АВ CD вершину А соединили с серединами сторон ВС и CD. Может ли один из отрезков оказаться вдвое длиннее другого? 6. Найдите все пары целых чисел (х, у), удовлетворяющих урав- нению: х2 = у2 + 2у + 13. Вариант 5 1. Площадь трех граней коробки прямоугольной формы равна 5, 12 и 15 дм2. Найдите объем коробки. 2. Отцу и сыну вместе 65 лет. Сын родился, когда отцу было 25 лет. Каков возраст отца и сына? 3. Окружность разделена на 100 равных дуг и в 100 точках деле- ния поставлено по фишке. Среди фишек 76 красных и 24 си- ние. Докажите, что среди красных фишек найдутся: (а) две диаметрально противоположные друг другу; (б) четыре, обра- зующие квадрат. 4. Докажите, что при любых значениях А и В, отличных от нуля, хотя бы одно из следующих двух уравнений имеет корень: 20Ах2 + 2х + 10В = 0; 5. Докажите, что биссектрисы внутренних углов прямоугольни- ка, пересекаясь, образуют квадрат.
60 Математические олимпиадные работы. 5-11 классы Вариант 6 1. Отгадайте ребус, представленный на рис. 15. ★★★★★★★ ★★ **★ ★*g** ** ** *** *** ~0 Рис. 15 2. Вычислите без помощи микрокалькулятора: 1,20124 + 0,79884 - - 1,20123 • 0.79882 - 1 20122 0J9883 + 4,8048 • 3,1952. 3. Решите уравнение: ^1 + ^2 + Jx - 2 . 4. На доске был нарисован параллелограмм ABCD, на котором отмечены точка Е стороны ВС и середина F стороны CD. Де- журный стер параллелограмм, но оставил точки Л, £, F. Как по этим точкам восстановить параллелограмм? 5. В компании 10 человек. Каждому из десяти нравится ровно 5 человек из компании. Докажите, что найдутся два человека, которые нравятся друг другу. Вариант 7 1. Упростите выражение: V2 + l + V3 + V2+" + ЛОО + Л» ’ 2. Докажите, что — + — + — является целым числом при любом 3 2 6 целом х. 3. В магазин поступила тонна фруктов: яблоки в ящиках по 48 кг, груши в ящиках по 20 кг, сливы в коробках по 14 кг и виш- ни в коробках по 10 кг. При этом яблок поступило в два раза
Примерные тексты математических олимпиад для муниципального этапа М 61 больше, чем груш, а вишен столько же, сколько слив. Сколько фруктов каждого вида поступило в магазин? 4. В четырехугольнике ABCD (рис. 16) углы при вершинахBnD прямые, АВ = ВС, а высота ВН равна 1 дм. Найдите площадь четырехугольника. 5. Решите уравнение: х1099 + 2х1199 + Зх1299 + ... + 20х1999 + 21х2009 - = 232. Варианте 1. «Ну, погоди!» — зарычал волк, заметив в 30 м зайца, и бросил- ся за ним в погоню, когда тому оставалось до места укрытия 333 м. Догонит ли волк зайца, если волк пробегает за минуту 600 м, а заяц — 550 м? Ответ обоснуйте. 2. Сумма двух чисел равна 2, а сумма их квадратов равна 3. Най- дите сумму кубов этих чисел. 3. В равностороннем треугольнике АВС с длиной стороны а точки М, N, Р, Q, О расположены так. что N лежит на стороне АВ, Q — на стороне ВС, а М и Р — на стороне АС так, что О является точ- кой пересечения отрезков NP и Л/Q. Известно, что МА + AN = = PC + CQ = а. Найдите величину угла NOQ. 4. Докажите, что число 2010 невозможно представить в виде раз- ности квадратов двух целых чисел. 5. Александр, Борис и Виктор решили 100 задач, причем каждый из них решил 60. Назовем задачу «трудной», если ее решил толь-
62 Математические олимпиадные работы. 5-11 классы ко один из мальчиков, и «легкой», если ее решили все 3 мальчи- ка. Докажите, что «трудных» задач на 20 больше, чем «легких». Вариант 9 1. Упростите выражение: (V2 + I)2 - (V2 - З)2 - V32 - >/з - 2V2 + V11 + 6V2 . 2. Найдите все трехзначные числа, которые в 18 раз больше сум- мы своих цифр. 3. В прямоугольном треугольнике один из углов равен 30°. До- кажите, что отрезок перпендикуляра, проведенного к гипоте- нузе через ее середину до пересечения с катетом, втрое меньше большего катета. 4. Даны действительные числа а, Ь, с, причем а > b > с. Докажите неравенство: а2 b + Ь2 с + с2 а> Ь2 а + а2 с + с2 Ь. 5. На Луне, где в 2012 году состоялась первая встреча землян с инопланетянами, встретились несколько землян, имеющих 4 конечности, и несколько инопланетян, которые имели по 7 конечностей. Сколько было землян на встрече, если всего ко- нечностей было 53? Вариант 10 1. Найдите значения а и Ь, при которых значение многочлена а3 + + - ab наименьшее, если известно, что а + b = 1. 2. Сравните З76 и 550. 3. Рыбак живет на полуострове, который ио форме напоминает со- бой острый угол (рис. 17). Рыбак должен дойти до одного берега полуострова, затем до другого и вернуться домой (в точку D). Как он должен идти, чтобы пройти по самому короткому пути? 4. Решите уравнение в целых числах: х3 + х2 + х - 3 = 0. 5. Среди четырех монет одна фальшивая. Она отличается от на- стоящих монет весом, однако неизвестно, легче она или тяжелее
Примерные тексты математических олимпиад для муниципального этапа к 63 настоящих монет. Масса настоящей монеты равна 5 г. Кроме четырех монет имеется гиря массой 5 г. Как с помощью двух взвешиваний на чашечных весах обнаружить фальшивую мо- нету и определить, легче она или тяжелее настоящих? 9 класс Вариант 1 1. Паганель, путешествуя по Африке, однажды остановился на ночлег на берегу небольшого озера с чистейшей водой (на дне били ключи). Однако угром к озеру подошло стадо слонов. Паганель насчитал 183 головы. Па следующее утро они ушли, оставив вместо озера грязную лужу. Через год Паганель вновь оказался на этом месте. Озеро вновь было полно воды, как и раньше. Но утром опять появилось стадо слонов. На этот раз в стаде было 37 слонов и воды им хватило на 5 дней. Покидая берега выпитого до дна озера, Паганель задумался: за сколько дней сможет осушить озеро один слон? Решите эту задачу и вы. 2. Пусть а и h — катеты прямоугольного треугольника, а с — гипо- тенуза. Сравните я3 + Ь2 и с3. 3. Определите коэффициенты квадратного уравнения х2 + рх + + q = 0 так, чтобы его корни были равны pnq.
64 Математические олимпиадные работы. 5-11 классы 4. Вычислите следующую сумму, если xyz = 1: 1 1 1 -----------1-----------1----------- 1 + X + Ху 1 + у + у 2 1 + Z + ZX 5. На доске написаны числа 1, 2, 3,2009, 2010, 2011. Разрешает- ся стереть с доски любые 2 числа и вместо них записать модуль их разности. В конце концов, на доске останется одно число. Может ли оно равняться нулю? Объясните. Вариант 2 1, Докажите неравенство: Ь* > a3b + ab3. 2. В разгар летнего сезона по сравнению с началом сезона ягоды дешевеют на 15 %, а сахар дорожает на 10 %, в результате приго- товление варенья из ягод по определенному рецепту обходит- ся на 10 % дешевле. Сколько процентов от стоимости варенья (приготовленного по тому же рецепту) составляет стоимость ягод в начале сезона? 3. Вычислите значение выражения V2009 • 2010 2011 • 2012 + 1, не применяя калькулятора. 4. В прямоугольник со сторонами 3 и 4 м вписан другой прямо- угольник, стороны которого относятся как 1 : 3 (рис. 18). Най- дите стороны вписанного прямоугольника. 5. Из трех различных цифр х, у, z (xt 0; уф 0; z^ 0) образова- ны все возможные трехзначные числа. Сумма этих чисел в три раза больше трехзначного числа, каждая цифра которого есть х. Найти цифры х, у, z.
Примерные тексты математических олимпиад для муниципального этапа 65 Вариант 3 1. Решите уравнение: хл/х2 - Зх + ^Зх - х2 - 2 = V3 + у/х -1,5. 2. Докажите, что число 210 4- 512 является составным. 3. Рассматриваются квадратичные функции вида у = х2 4- рх 4 д, 1 для которых р 4- —q = 2010. Докажите, что графики данных функций проходят через одну точку 4. Дан параллелограмм ABCD. К — середина стороны ВС, М — се- редина стороны CD, АК = 6, AM = 3, Z КАМ = 60°. Найдите длину стороны AD. Ответ обоснуйте. 5. При подготовке задач для олимпиады 30 студентов с пяти кур- сов придумали 40 задач, причем однокурсники — одинаковое число задач, а студенты с разных курсов — разное. Сколько сту- дентов придумали ровно по одной задаче? Вариант 4 1. Докажите, что 5 4- 52 4- 53 4-... 4- 52010 делится на 6. 2. Построены четыре круга с центрами в вершинах квадрата и ра- диусами, равными стороне квадрата. Найдите площадь пересе- чения этих кругов, если сторона квадрата равна 1 см (рис. 19). Рис. 19
66 Математические олимпиадные работы. 5-11 классы 3. Решите уравнение: г/4 + 2лл + 1 = 4х2г/. 4. Мотоциклист и велосипедист выехали одновременно из пун- кта А и Б. Проехав треть пути, велосипедист остановился и тронулся дальше лишь тогда, когда мотоциклисту остава- лось проехать треть пути до пункта Б. Мотоциклист, доехав до пункта Б, без остановки поехал обратно в пункт А. Велосипе- дист больше остановок в пути не делал. Кто приедет раньше: мотоциклист в пункт А или велосипедист в пункт В? 5. В комнате собрались 8 человек. Некоторые из них лгут, а остальные всегда говорят правду. Один из собравшихся ска- зал: «Здесь нет ни одного честного человека». Второй сказал: «Здесь не больше одного честного человека». Третий сказал: «Здесь не более двух честных людей» и т. д. до восьмого, ко- торый сказал: «Здесь не более семи честных людей». Сколько в комнате честных людей? Ответ обоснуйте. Вариант 5 1. Про три простых числа известно, что одно из них равно разно- сти кубов двух других. Найдите эти числа. 2. Докажите, что в остроугольном треугольнике АБС расстояние •от центра описанной окружности до стороны БС вдвое меньше расстояния от точки пересечения высот треугольника до вер- шины А. 3. Решите систему уравнений: х2 - 2у + 1 = О, уг - 2г + 1 = О, z2 -2.V+1 =0. 4. Может ли дискриминант квадратного уравнения с целыми ко- эффициентами быть равным 2010? 5. В каждую клетку квадратной таблицы 25 х 25 вписано произ- вольным образом одно из чисел: 1 или - 1. Под каждым столб- цом пишется произведение всех чисел, стоящих в этом столб- це. Справа от каждой строки пишется произведение всех чисел, стоящих в этой строке. Докажите, что сумма пятидесяти напи- санных произведений не может оказаться равной нулю.
Примерные тексты математических олимпиад для муниципального этапа 67 Вариант 6 1. Докажите, что уравнение х* ~ 2Х3 + Зх2 - 4х + 5 =" О не имеет решений. 2. Докажите, что значение выражения (л/4 + \/2 + 1) х (^4 - 1) х х (л/4 - д/2 4 1) будет целым, и найдите его. 3. Значения квадратного трехчлена ах1 + 2Ьх + с отрицательны при всех х. Докажите, что значения я2х2 + 2Ь2х + с2 при всех х положительны. 4. Угол между диагоналями трапеции равен 120°. Одна из ее диа- гоналей равна 4, а высота — 2. Найдите длину второй диагонали. 5. В каждой вершине правильного 2011-угольника записано по- ложительное число, причем каждое из этих чисел равно сред- нему арифметическому двух чисел, записанных в соседних вершинах. Докажите, что все записанные в вершинах числа равны между собой. Вариант 7 1. Делится ли на 7 число 2010 • 2011 • 2012 + 201 Зв? 2. Решите систему уравнений: лт/ - z, < yz = X, XZ = у. 3. Найдите все значения а, при которых уравнение х2+ ах + а = 0 имеет целые корни. 4. Бумажный выпуклый четырехугольник разрезали на четыре части по отрезкам, соединяющим середины противоположных сторон. Докажите, что из этих частей можно сложить паралле- лограмм. 5. В квадрат 1x1 бросили 51 точку. Докажите, что найдется круг 1 радиусом у, содержащий не менее трех точек. Вариант 8 1. Решите уравнение: (2х - 1) х (2х + 3) х (Зх - 2) х (Зх + 4) = 35.
68 Математические олимпиадные работы. 5-11 классы 2. Один из углов треугольника на 120° больше другого. Докажи- те, что биссектриса, проведенная из третьего угла этого треу- гольника, вдвое длиннее высоты, проведенной из того же угла. 3. Квадрат несколькими сквозными разрезами, параллельными его сторонам, разделили на прямоугольники. Оказалось, что сумма периметров этих прямоугольников в 100 раз больше периметра исходного квадрата. Какое наибольшее число пря- моугольников могло при этом получиться? Максимальность предложенного в ответе числа нужно доказать. 4. В краже классного журнала из учительской подозреваются четверо учащихся: Александр, Борис, Виктор и Григорий. При выяснении того, кто повинен в воровстве журнала, они сказали директору школы: Александр: «Это сделал Борис». Борис: «Это сделал Григорий». Виктор: «Это сделал не я». Григорий: «Борис лжет, что это сделал я». Правду сказал только один. Кто совершил кражу? 5. Докажите, что при любом нечетном целом лг, большем едини- цы, следующее произведение делится на п: 1 1 1 (1 + 4 + | + ... + -Ц-).2-3.... (п-1) 2 3 п -1 6. Па столе лежат 2007 монет. Двое, играя в игру, ходят по очере- ди: за ход первый игрок может взять со стола любое нечетное число монет от 1 до 99, второй — любое четное число монет от 2 до 100. Проигрывает тот, кто не сможет сделать ход. Кто выи- грает при правильной игре? Вариант 9 1. Па рис. 20 указаны величины углов ABC, BCD, CDE, DEF. Яв- ляются ли прямые АВ и EF параллельными? Почему? 2. Сумма 2012 натуральных чисел — число нечетное. Каким чис- лом, четным или нечетным, является произведение этих чисел? 3. На медиане ВМ треугольника АВС взята произвольная точка D. Через точку D проведена прямая, параллельная стороне АВ,
Примерные тексты математических олимпиад для муниципального этапа 69 а через вершину С — прямая, параллельная медиане ВМ. полученные прямые пересекаются в точке Е. Докажите, что BE-AD. 4. Гриб называется плохим, если в нем живет больше 11 червяков, и г 1 Червяк считается тощим, если он съел не более — части грпоа, 5 в котором живет. Четверть всех грибов в лесу плохие. Докажи- те, что не менее трети всех червяков тощие. 5. Расстановка шахматных королей на доске называется правиль- ной, если ни один из них не бьет другого, а каждое поле доски либо находится под боем, либо занято одним из королей. Какое минимальное и какое максимальное число королей можно пра- вильно расставить на шахматной доске размером 8x8? Вариант 10 1. Пусть М — внутренняя точка квадрата АВ CD. Найдите пло- щадь квадрата ABCD, если известно, что МА = 7 см, МВ =17 см, МС = 23 см. 2. Можно ли выписать в ряд семь некоторых целых чисел так, чтобы сумма любых трех смежных чисел была отрицательной, а сумма всех — положительной?
70 Математические олимпиадные работы. 5-11 классы 3. Докажите, что если А х (А + В + С) < 0, то уравнение Лх2+ Вх + + С = 0 имеет два действительных корня. 4. В некоторой трапеции длина одной из диагоналей равна сум- ме длин оснований трапеции, а угол между диагоналями ра- вен 60°. Докажите, что эта трапеция равнобокая. 5. Найдите все трехзначные числа, которые в 12 раз больше сум- мы своих цифр. 6. Перед Бабой Ягой и Кощеем Бессмертным лежат две кучи му- хоморов, в одной из них 2010 штук, а в другой 13 штук. Эти персонажи по очереди берут грибы кучи, за один раз можно взять любое ненулевое число грибов из одной из куч. Пропу- скать ход нельзя, выигрывает тот, после хода которого грибов не останется. Первой ходит Баба Яга. Кто из игроков выиграет при правильной игре? 10 класс Вариант 1 1. Сумма двух натуральных чисел равна 2011. Если у одного из них зачеркнуть последнюю цифру, то получится второе число. Найдите все такие числа. 2. Известно, что а + b + с < 0 и что уравнение ах2 + Ьх + с = 0 не имеет действительных корней. Определите знак с. 3. На прямоугольном столе лежат 5 журналов, полностью закры- вая его. Журналы могут выступать за пределы стола. Докажите, что можно убрать два из них так, чтобы оставшиеся закрывали 3 •гд. • не менее — площади стола. 1 5 4. На карьере добыли 36 камней. Их веса составляют арифмети- ческую прогрессию: 490, 495, 500, ..., 665 кг. Можно ли увезти эти камни на семи трехтонных грузовиках? 5. На чудо-дереве растут бананы и ананасы. За один раз разре- шается сорвать с него два плода. Если сорвать два банана или два ананаса, то вырастает один ананас, а если сорвать один ба- нан и один ананас, то вырастает один банан. В итоге на дереве
Примерные тексты математических олимпиад для муниципального этапа 71 остался один плод. Какой это плод, если известно, сколько ба- нанов росло вначале? Вариант 2 1. Найдите наибольшую возможную площадь четырехугольника, у которого произведение длин двух любых соседних сторон равно 1. 2. п .66,66 Расположите числа sm~, cos—, tg—, ctg— в порядке возрас- 5 5 5 5 тапия. Обоснуйте ваш ответ. 3. Найдите все пары целых чисел (х, #), для которых 2х2 + у2 = = 2ху + 4х. 4. Решите систему уравнений: х _ у _ z __ и __ s у г и s t * х = 8zz, X + у + Z + U S + t = 15 —. 5. Определенная на всей числовой оси функция /(х) такова, что для любого числах выполнено равенство: f (/(/(.../(х)...))) = 1. 2009 Какое наименьшее число корней может иметь уравнение/(х) = 1? Вариант 3 1. Какое из чисел больше: -тгг»----- 22012 . 22012 +1 или 22013 2. Найдите все целые числа п, при которых пъ 4- 3 делится нацело на пг 4- 1. 2 X 1 1 3. Решите уравнение: х + — - — + — = 5. 2 2х х 4. Из трех разных вершин треугольника проведены биссектриса, медиана и высота соответственно. Могут ли медиана и биссек- триса разделить высоту на три равные части?
72 Математические олимпиадные работы. 5-11 классы 5. (Задача Ньютона). Трава на всем лугу растет одинаково быстро и густо. Известно, что 70 коров съели бы ее за 24 дня, а 30 ко- ров — за 60 дней. Сколько коров съели бы всю траву за 96 дней? Предполагается, что коровы съедают траву равномерно. Вариант 4 1. Определите а так, чтобы сумма квадратов корней уравнения х2 + (2 - а) • х - а - 3 = 0 была наименьшей. 2. Докажите, что если в прямоугольном треугольнике все сторо- ны выражаются целыми числами, то среди катетов найдется такой, длина которого делится на 3. 3. На стороне АВ правильного треугольника АВС взяли точку М и на отрезке МС по ту же сторону от него, что и точка В, по- строили правильный треугольник МКС. Докажите, что прямые АС и ВК параллельны. 4. В каждой вершине правильного 2011-угольника записано по- ложительное число, причем каждое из этих чисел равно сред- нему геометрическому двух чисел, записанных в соседних вер- шинах. Докажите, что все записанные в вершинах числа равны между собой. 5. Шахматная доска размером 6x6 покрыта 18 костяшками до- мино (каждая костяшка покрывает две клетки доски). Докажи- те, что независимо от расположения костяшек всегда можно разрезать доску по прямой, параллельной одной из сторон края доски, не повредив ни одной костяшки домино. Вариант 5 1. Несколько учащихся ушли из лицея и несколько пришли. В ре- зультате число учащихся уменьшилось на 10 %, а доля мальчи- ков в лицее увеличилась с 50 до 55 %. Увеличилось или умень- шилось число мальчиков? 2. Число а положительно, а числа b и с ~ отрицательны. Какое из . а а+с„ чисел больше: — или------? b Ь + с 3. Первый член числовой последовательности равен 1, каждый из двух следующих равен 2, каждый из трех следующих за ними равен 3 и т. д. Чему равен 2009-й член этой последовательности?
Примерные тексты математических олимпиад для муниципального этапа 73 4. На окружности выбраны диаметрально противоположные точ- ки Л и В и отличная от них точка С. Касательная к окружности в точке А и прямая ВС пересекаются в точке D. Докажите, что прямая, касающаяся окружности в точке С, делит пополам от- резок AD. 5. Найдите все пары целых положительных чисел (х, у), для кото- рых справедливо равенство: 11 1 [ _____________1____t х + у + НОК(х, у) + НОД(х, у) ~ Вариант 6 1. Решите уравнение: у/х + 3-4>/х - 1 + ух + 8 -6yfx~1 = 1 . 2. Докажите, что если в бесконечную арифметическую прогрес- сию с положительной разностью входят числа 25, 43, 70 (не обязательно стоящие рядом), то в эту прогрессию входит и чис- ло 2014. 3. Могут ли три параболы, изображенные на рис. 21, при каких- либо значениях а, Ь, с быть графиками функций? Ответ обо- снуйте. 4. За круглым столом сидят 9 человек: рыцари (говорящие всегда правду) и лжецы (лгущие всегда). Каждый сказал: «Мои сосе- ди — лжец и рыцарь». Сколько всего лжецов за столом?
74 Математические олимпиадные работы. 5-11 классы 5. Даны две параллельные прямые и точки А и В на них. Построй- те две окружности, у которых бы радиус одной был больше ра- диуса другой в два раза, и которые бы внешне касались друг друга и касались данных прямых, одна в точке Л, другая — в точке В. (В точке А — большая окружность.) Вариант 7 1. Докажите, что синус суммы двух углов треугольника равен си- нусу третьего угла. 2. Постройте график уравнения: |х| + |г/| = 1. 3. Если в многочлен Ах3 + Вх1 + Сх3 + Dx2+ Ex + Е вместо букв Л, В, С, D,E, Ев любом порядке подставлять числа 21, -16,13, -47, 46, -17, то будут получаться многочлены от переменной х. До- кажите, что у всех этих многочленов общий корень. 4. Па продолжениях сторон АВ, ВС, CD и DA выпуклого четы- рехугольника откладываются отрезки ВВ{ = АВ, СС\ = ВС, DD{ = CD, ЛЛ1 = DA. Докажите, что площадь четырехугольника в 5 раз больше площади исходного четырехугольника ABCD. 5. На острове рыцарей и лжецов живут 2009 человек. Рыцари всегда говорят правду, лжецы всегда врут. Во время социологи- ческого опроса каждый житель острова заявил: «Среди осталь- ных островитян более половины — лжецы». Сколько лжецов живет на острове? Р п . 7 8 - sin2 х 6. Решите уравнение: 4 4-х =------— . 2 4-х4 Вариант 8 1. (Задача Гипсикла). Докажите, что в арифметической прогрес- сии с положительной разностью и с четным числом членов сумма членов второй половины превышает сумму членов пер- вой на число, кратное квадрату половины числа членов. 2. Найдите какое-нибудь целое положительное число, которое само делится на 2009 и сумма цифр которого делится на 2009. 3. Трава на всем лугу растет одинаково густо и быстро. Известно, что если выпустить на луг 20 коров, то они съедят траву полно- стью за 8 дней, а если выпустить на тот же луг 26 коров, — то
Примерные тексты математических олимпиад для муниципального этапа 75 за 6 дней. Какое наибольшее число коров может кормиться на этом лугу все лето? Аппетит у коров в течение лета одинаков и неизменен. Скорость роста травы постоянна. 4. На продолжении СЕ стороны АС равностороннего треуголь- ника АВС за вершину С построен произвольный равносто- ронний треугольник CDE. Докажите, что треугольник СМР, где М и Р — середины отрезков AD и BE соответственно, тоже равносторонний. 5. Прямоугольник, у которого одна из сторон в два раза длиннее другой, несколькими сквозными разрезами, параллельными его сторонам, разделили на прямоугольники. Оказалось, что сумма периметров этих прямоугольников в 101 раз больше пе- риметра исходного прямоугольника. Какое наибольшее число прямоугольников могло при этом получиться? Максималь- ность предлагаемого в качестве ответа числа нужно доказать. 6. Назовем «дельфином» фигуру, которая ходит по клеточному полю на одно поле вверх или вправо и по диагонали палево и вниз (рис. 22). Рис. 22 Может ли «дельфин», йЫйв из левого нижнего угла доски раз- мером 8x8, обойти всю доску, побывав на каждой клетке ровно по одному разу? Вариант 9 1. Решите уравнение: (sin х)2012 + (cos х)2012 = 1. 2. Расставьте в клетках таблицы 3x3 вещественные числа так, чтобы сумма любых двух соседних по горизонтали чисел была равна 6, а произведение любых двух соседних по вертикали чи- сел было равно 4.
76 Математические олимпиадные работы. 5-11 классы 3. Докажите, что в ходе любого сыгранного футбольного матча был момент, когда одна из команд забила голов столько же, сколько другой осталось забить. 4. Пусть АН — высота остроугольного треугольника АВС, К и L — основания перпендикуляров, опущенных из точки Н на сторо- ны АВ и АС. Докажите, что точки В, К, L и С лежат на одной окружности. 5. Докажите, что бесконечная возрастающая арифметическая про- грессия из натуральных чисел либо совсем не содержит квадра- тов натуральных чисел, либо содержит их бесконечно много. 6. Можно ли выписать в ряд семь некоторых целых чисел так, чтобы сумма любых трех смежных чисел была отрицательной, а сумма всех семи чисел — положительной? Вариант 10 1. Докажите, что если стороны треугольника образуют геометри- ческую прогрессию, то их высоты тоже образуют геометриче- скую прогрессию. 2. Дана система уравнений: Icosa + cosp = 1, [sin a + sin£ = a. Докажите, что а < -х/З. 3. Докажите, что число 22(НЗ - 1 составное. 4. В турнире по настольному теннису, где каждый сыграл с каж- дым ровно одну партию, а ничьи были невозможны, участво- вали блондины и рыжие, причем блондинов было втрое боль- ше. Оказалось, что число партий,выигранных рыжими, равно числу партий, выигранных блондинами. Сколько игроков уча- ствовало в турнире и кто победил: блондин или рыжий? 5. Прямая произвольным образом окрашена в два цвета. Дока- жите, что на ней обязательно найдется отрезок, у которого оба конца и середина окрашены в один цвет. 6. Хорда удалена от центра окружности на расстояние h. В каж- дый из двух сегментов круга, стягиваемый этой хордой, впи- сан квадрат так, что пара его соседних вершин лежит на хор-
Примерные тексты математических олимпиад для муниципального этапа М 77 де, а другая пара соседних вершин — на соответствующей дуге окружности. Найдите разность длин сторон квадратов. 11 класс Вариант 1 1. Докажите, что sin6 х + cos6 х > 0,25. 2. Не выполняя деления, докажите, что значение выражения 200 800 • 1003 + 2008 • 100 800 делится на 2011. 3. Утром 1 сентября учитель математики написал на доске числа 6 и 7. Затем он поручил дежурному каждое утро стирать на- писанные на доске числа и писать вместо них их среднее ариф- метическое и среднее гармоническое. Чему будет равно произ- ведение чисел, записанных на доске, днем 22 октября? л- л 1 а + Ь Средним арифметическим чисел а и b называется число---, 2 2 а их средним гармоническим — число -- а*~Ь 4. На медиане ВМ треугольника ЛВС взята произвольная точ- ка D. Через точку D проведена прямая, параллельная стороне АВ, а через вершину С — прямая, параллельная медиане ВМ. Две полученные прямые пересекаются в точке £. Докажите, что BE = AD. 5. Для участников математической олимпиады и членов жюри было приготовлено конфет столько же, сколько булочек и ста- канов чая вместе. Каждый школьник съел по конфете и выпил по стакану чая, после 4*его осталось стаканов чая и конфет вме- сте столько, сколько булочек. Найдется ли хотя бы один стакан чая для членов жюри? 6. Решите в целых числах уравнение: ху + х - 5у = - 6. Вариант 2 1. 1 Решите уравнение: sin х • cos х • cos 2х • cos 8х = — sin 12х.
78 Математические олимпиадные работы. 5-11 классы 2. Докажите, что сумма членов каждой горизонтальной строки равна квадрату нечетного числа: 1 2 34 3 4 5 6 7 4 5 6789 10 3. Имеется два слитка сплавов золота и меди. В первом слитке от- ношение золота к меди равно 1 : 2, а во втором 2 : 3. Если спла- 1 5 вить — первого слитка с — второго, то в получившемся слитке 3 6 окажется столько золота, сколько было в первом меди, а если 2 1 — первого сплавить с — второго, то в получившемся слитке окажется меди на 1 кг больше, чем было золота во втором слит- ке. Сколько золота было в каждом слитке? 4. На сторонах AD и CD квадрата АВ CD со стороной 3 взяты две точки М и Д^так, что MD + DN = 3. Прямые ВМ и CD пересека- ются в точке Е. Найдите длину отрезка NE, если ME = 4. 5. Клетки двух таблиц 2010 х 2011 закрашены в красный и синий цвета так, что в каждой строке и каждом столбце четное чис- ло синих клеток. Одну из таблиц положили на другую так, что они совместились. При этом одна из синих клеток наложилась на красную клетку. Докажите, что найдутся по крайней мере еще три клетки, покрытые клетками другого цвета. Вариант 3 1. В усеченный конус вписан шар. Сумма длин диаметров верх- него и нижнего оснований конуса в 5 раз больше длины радиу- са шара. Найдите угол между образующей усеченного конуса и плоскостью основания. 2. Решите систему уравнений: у2 = х3 - Зх2 + 2х, х2 = у3 - Зу2 + 2у.
Примерные тексты математических олимпиад для муниципального этапа 79 3. На координатной плоскости задан четырехугольник с верши- нами в точках (0; 6), (8; 12), (11; 8) и (3; 2). Вычислите площади фигур, на которые разбивает его прямая х + 7 у - 67 = 0. 4. Чему равно произведение sin 1° • sin 3° • sin 5° •... • sin 87° • sin 89° ? 5. Найдите такую дробь с наименьшим знаменателем, для кото- о 1 т 1 рои справедливо следующее:------< — <-----. 2011 п 2010 Вариант 4 1. Докажите, что произведение четырех последовательных целых чисел, сложенное с единицей, есть точный квадрат. , 1 Ji 1 1 2. Выясните, которое из чисел больше: 1+^+з+”+ 1 А 1 Л 1 1 1 ) 2011J 2012 2 3 2012 [ 3. Четырехугольник ABCD вписан в окружность с центром О. Диагонали АС и BD взаимно перпендикулярны. Докажите, что длина перпендикуляра ОН, проведенного из центра окружно- сти к стороне AD, вдвое меньше стороны ВС. 4. Решите в натуральных числах уравнение: 1 + х + х2 + л"3 2'Л 5. Существует ли многогранник с нечетным числом граней, каж- дая из которых есть многоугольник с нечетным числом сто- рон? Вариант 5 1 1 1 1. Можно ли из последовательности 1, —, —...., — выделить ариф- 2 3 п метическую прогрессию, состоящую из 2012 чисел? 2. Решите уравнение: sin4 4х + cos2x -= 2 sin 4л • cos^x. 1 — х 3. Постройте график функции у(х) = arctg х + arctg--. 1 + х
80 Математические олимпиадные работы. 5-11 классы 4. В треугольнике АВС проведены медианы АК и СМ. Оказалось, что /.ВАК = /ВСМ = 30°. Докажите, что треугольник АВС равносторонний. 5. Найдите сечение наименьшей площади, проходящее через диа- гональ куба. Вариант б 2 2 2 л х + ах + а а 1. Решите уравнение: —-------- = —. х1 - ах + xz 2. Докажите, что правильный тетраэдр можно пересечь плоско- стью так, чтобы в сечении получился квадрат. 1 3. Докажите, что касательные к гиперболе у == — образуют с ося- х ми координат треугольники одной и той же площади. 4. Из набора гирек, имеющих массу 1, 2,3,..., 101 г, удалили гирьку массой 19 г. Можно ли остальные гирьки разложить по 50 штук на каждую из чашек весов так, чтобы весы были в равновесии? 5. В каждой вершине правильного 2011-угольника записано по- ложительное число, причем каждое из этих чисел равно либо среднему арифметическому, либо среднему геометрическому двух чисел, записанных в соседних вершинах. Докажите, что все записанные в вершинах числа равны между собой. Вариант 7 3 1. Вычислите: J ||х| - 2| dx, -з 2. Решите систему уравнений: 2х + у х—=1, х -у х + 2у х - у 3. Внутри квадрата АВ CD задана точка К так, что треугольник АКБ — равнобедренный с углом при основании 15°. Докажите, что треугольник CKD — правильный.
Примерные тексты математических олимпиад для муниципального этапа 81 4. Можно ли число 2010 представить в виде разности х" - уп, где п > 2, причем п — четное, а х, у — натуральные числа? 5. Найдите все пары целых чисел (х, г/), для которых 2х2 + у2 = 2ху + + 3г/. Вариант 8 1. Найдите число а = log3 2 • log4 3 • log- 4 • ... • log2013 2012. Г1 2. Докажите, что при хе —; +©о функция у = х2 - Зх3 принимает 1 значения, меньшие —. 60 3. В треугольнике с углом 120° стороны треугольника образуют арифметическую прогрессию с разностью 1. Найдите стороны треугольника. 4. Можно ли завернуть кубик с ребром 1 в квадратный листок бу- маги со стороной 3? 5. Докажите, что уравнение Юх3 - 17г/3 = 51 не имеет решений в натуральных числах. Вариант 9 1. Сравните 20102010 и 20112009. 2. Найдите все числа х и г/, удовлетворяющие уравнению: 3. Дан выпуклый четырехугольник ABCD, в котором точка Е — середина стороны CD. Докажите, что если площадь треуголь- ника АВЕ равна половине площади четырехугольника ABCD, то сторона AD будет параллельна стороне ВС. 4. Найдите все трехзначные числа, кратные 13, сумма цифр кото- рых также кратна 13. 5. Гриб называется плохим, если в нем живет больше 11 червяков. Червяк считается тощим, если он съел не более — части гриба, 5 в котором живет. Четверть всех грибов в лесу плохие. Докажи- те, что не менее трети всех червяков тощие.
82 я Математические олимпиадные работы. 5-11 классы Вариант 10 1. Найдите наименьшее значение функции /(х) = х(х + 1)(х + 2) (х+ 3). 2. Решите уравнение: х1 - 4х ~ 1 0. 3. Площадь прямоугольного треугольника равна S. I [айдите пло- щадь треугольника с вершинами в основаниях перпендикуля- ров, проведенных из точки пересечения медиан данного тре- угольника на катеты и гипотенузу 4. Найдите все такие натуральные числа а и h (а < Ь), чтобы чис- ло (За - 1) делилось на Ь, а число (ЗЬ - 1) — на а. 5. В кубе со стороной 1 м находится 2009 тараканов. Докажите, что хотя бы трех из них можно поймать сферой радиусом
Решения, указания, ответы В приведенных далее решениях текстов олимпиад по математике автором показаны возможные решения задач. 5 класс Вариант 1 2010 - 2009 + 2008 - 2007 +... + 2 -1 1005 L 2010 45 + 55 2010 ’ ~ 2010(45 + 55) “ 1005 2010 100' Так как тему «Сокращение дробей» учащиеся по большинству учебников еще не изучали, то ответ можно оставить в таком виде. 2. Так как в квартирах 1 и 2 черный кот не живет, то он живет в квартире 3. Так как белый кот не живет в квартире 1, а квар- тира 3 занята черным котом, то белый кот живет в квартире 2. Тогда рыжий кот живет в квартире 1.
84 Математические олимпиадные работы. 5-11 классы Ответ: В квартире 1 живет рыжий кот; в квартире 2 — белый кот; в квартире 3 — черный кот. 3. Для нумерации страниц с первой по девятую понадобится 9 цифр, для нумерации страниц с 10 по 99 — 90 х 2 = 180 цифр. Итого требуется 189 цифр. Остается 204 - 189 = 15. Так как с сотой страницы на нумерацию одной страницы требуется 3 цифры, то всего страниц в книге будет 99 + 15 :3 = 99 + 5 = 104. Ответ: В книге 104 страницы. 4. Всего получается 4 способа (рис. 23). Рис. 23 5. Пусть 2х кг весит туловище щуки, тогда голова будет весить (х + 1) кг. Из условия, что туловище весит столько же, сколько голова и хвост вместе, получаем уравнение: 2л: = х + 1 + 1. От- сюда х = 2, а вся щука весит 8 кг. Ответ: Щука весит 8 кг. 6. Решаем задачу с конца. 1) (1 + -^ )• 2 = 3 (конфеты) осталось после Коли. 2) (3 + — )• 2 = 7 (конфет) осталось после Ани. 3) (7 + — )• 2 = 15 (конфет) осталось после Феди. 1 4) (15 +— )• 2 = 31 (конфета) была в коробке. 2 Ответ: В коробке была 31 конфета.
Решения, указания, ответы 85 Вариант 2 1. (1003 • 2009 - 1006) : (1003 + 2009 • 1002) = ((1002 + !)• 2009 - - 1006): (1003 + 2009 х 1002) = (1002 • 2009 + 2009-1006): (1003 + + 2009 • 1002) = (1002 • 2009 + 1003) : (1003 + 2009 • 1002) = = (1003 + 1002 • 2009):( 1003 + 2009 • 1002) = 1. 2. 1) 25 х 25 = 625 (см2)— площадь одного платка. 2) Зм2 = ЗО 000 см2. 3) 30 000 : 625 = 48 (платков) израсходовал Кенгуру за 8 дней. 4) 48 : 8 = 6 (платков). Ответ: 6 платков использует Кенгуру за один день. 3. 495 + 459 = 954. Начать с цифр десятков: И = 0 или И = 9. Пер- вый случай не может быть, так как С + И = К. Таким образом, И = 9, тогда К = 4, соответственно, С = 5. 4. Задача имеет множество решений. Рассмотрим одно из воз- можных. Так как 2 чашки и 2 кувшина уравновесят 14 блюдец, то 1 чаш- ка и 1 кувшин уравновесят 7 блюдец. Так как 1 кувшин весит столько, сколько 1 чашка и 1 блюдце, то 2 чашки и 1 блюдце весят столько, сколько 7 блюдец. Отсюда получим, что 1 чашка весит столько, сколько 3 блюдца. Значит, 1 кувшин уравнове- сят 4 блюдца. 5. Узнаем сначала туфли друзей. Так как у Кролика туфли зеле- ные, а у Пятачка — не красные, то красные туфли будут у Вини- Пу ха, а синие — у Пятачка. Так как у Вини-Пуха цвета рубашки и туфель совпадают, а туфли красные, то и рубашка красная. Так как у Пятачка ни туфли, ни рубашка не красные, а туфли синие, то рубашка может быть только зеленая. Поэтому у Кро- лика рубашка синяя. 6. Всего существует 7 способов (рис. 24). Рис. 24
86 к Математические олимпиадные работы. 5-11 классы Вариант 3 1. 1) 25,6 : 4 = 6,4 ( — ) — скорость сближения поездов. ч 2) 58,4 + 6,4 = 64,8 ( — ) — скорость второго поезда. ч Ответ: Скорость второго поезда равна 64,8 —. ч 2. Возможный вариант: 9-8-7-4-6-5 + 4- 3- 2 + 0 = 2010. 3. Предположим, что правду сказал Алеша Попович, то есть Змея убил он. Но тогда получается, что и Добрыня Никитич сказал правду, а это противоречит условию задачи. Предположим, что правду сказал Добрыня Никитич, то есть Змея убил Алеша По- пович. Тогда получается, что и Алеша Попович сказал правду, что противоречит условию задачи. Значит, правду сказал Илья Муромец, то есть Змея убил Добрыня Никитич. Этот вариант не противоречит условию задачи. Получается, что и Добрыня Никитич, и Алеша Попович слукавили. Ответ: Змея убил Добрыня Никитич. 4. Начнем с буквы О. Так как О + О = О, то О = 0 (в этом случае Р < 5) или 0 = 9 (тогда Р > 4). Рассмотрим случай 0 = 0. Так как Р + Р = С и Т + Т = С, то получаем, что Р = С, что про- тиворечит условию задачи. Значит, 0 = 9. Далее, Р + Р = С + + 10 (у нас Р > 4) и Т + Т = С. Учитывая также, что С < 5 (так как С+С=К), имеем Р = 7, Т = 2, а С = 4. Других вариантов нет. Так как С = 4, то К = 8. В итоге, имеем: С = 4, П = 3, Т = 2; Р = 7, К = 8, О = 9. + 43972 Ответ: 43972. 87944 5. Так как зайцев на 30 больше, чем волков, то без 30 зайцев жи- вотных в лесу будет 70, причем среди них зайцев и волков одинаковое количество. Так как волков на 25 больше, чем мед- ведей, то с 25 дополнительными медведями в лесу животных будет 95, причем всех животных поровну. Но 95 на 3 не делит- ся. Значит, лиса не права, а прав заяц. Ответ: Прав заяц. 6. 89089089089 • 7373 - 73073073073 • 8989 = 89 • 1001001001 х х 73 • 101 - 73 • 1001001001 • 89 • 101 = 0.
Решения, указания, ответы 87 Вариант 4 1. 2011 - 2010 + 2009 - 2008 + 2007 - ...-2 + 1 = (2011 - 2010) + + (2009 - 2008) + (2007 - 2006) + ...+ (3 - 2) + 1 = 1005 + 1 = = 1006. 2. Так как сумма двузначного и двух однозначных чисел — трех- значное число, то С = 1, а Б = 9. Тогда А = 6. 6 + 99 Ответ: g. tri 3. Всего существует 5 вариантов (рис. 25). Рис. 25 4. Так как больше всего ударов (7) нанес Илья Муромец, а мень- ше всех Алеша Попович, то Добрыня Никитич нанес от 4 до 6 ударов. Всего ударов великаны получили от 14 до 16. Из это- го промежутка только число 15 делится на 3. Следовательно, великанов было 5. Ответ: Великанов было 5. 5. Если бы Митя встал в конце очереди и рядом с ним был Юра, то Юра был бы последним. Если бы Митя встал в середину оче- реди, то рядом с ним оказались бы Сеня и Костя, значит, Сеня и Костя стоят рядом: Костя за Сеней или Сеня за Костей. А так как Митя стоит впереди очереди, то возможны два варианта: первый вариант: Митя, Толя, Сеня, Костя, Юра; второй вари- ант: Митя, Толя, Костя, Сеня, Юра. Ответ: Есть два возможных варианта. Первый вариант: Митя, Толя, Сеня, Костя, Юра; второй вариант: Митя, Толя, Костя, Сеня, Юра. Вариант 5 1. 4 + 4-4 -4-0, (4 + 4) : (4 + 4) = 1,
88 Математические олимпиадные работы. 5-11 классы 4:44-4:4 = 2, 4-4:4- 1 = 3, 4 - (4 - 4) + 4 = 4, (4 + 4-4):4 = 5, 4 + (4 + 4):4 = 6, 4 + 4-4:4 = 7, 4+444-4 = 8, 4 + 4 +4:4 = 9. 2. Буква Б — обязательно 9, иначе при сложении не будет перехо- да через тысячу Буква А — обязательно 1, иначе сумма АБББ + + А не будет оканчиваться на три одинаковые цифры. Таким образом, получается единственный способ: 1999 + 1 = 2000. Ответ: 1999 + 1 = 2000. 3. Так как после пересаживания Шарика между котом и дядей Федором кот оказывается крайним слева, а рядом с ним сидит дядя Федор, то далее слева направо сидят почтальон Печкин и Шарик. Ответ: Слева направо на скамейке сидят кот, дядя Федор, поч- тальон Печкин и Шарик. 4. Так как после покупки 11 тетрадей у ученика остается 500 руб., а для покупки 15 тетрадей у него не хватает 700 руб., то 4 тетра- ди будут стоить 500 + 700 = 1200 руб. Тогда одна тетрадь будет стоить 300 руб. Следовательно, у школьника было 11 • 300 + + 500 = 3800 руб. 5. На рис. 26 показан вариант разрезания указанного прямо- угольника на две части, из которых можно составить квадрат. Рис. 26
Решения, указания, ответы 89 6. Сумма четного числа и 10 нечетных чисел четная. У нас 10 игру- шек, цена каждой игрушки составляет нечетное число рублей, значит, сумма 10 нечетных чисел должна быть четным числом. Но 2009 — число нечетное, поэтому получить его в виде суммы 10 нечетных чисел нельзя. Вариант б 1. Разделим две противоположные стороны квадрата на отрезки длиной 3 см. Соединяя концы получившихся отрезков, получим 1005 одинаковых прямоугольников. Проведя в каждом прямо- угольнике диагональ, получим 2010 одинаковых треугольников. 2. Возможный вариант показан на рис. 27. 3. Так как самое большое трехзначное число, которое делится на 6, есть 996, а искомое больше него на 5, то число равно 1001. 4. Переложим 4 и 7, получим: 1261034789 5. Переложим 6 и 10, получим: 12347895610. Наконец, переложив 5 и 6, в итоге мы получим: 12345678910. 5. Выиграет начинающий. Своим первым ходом он соединяет любые две диаметрально противоположные точки. На после- дующих ходах он строит отрезок, симметричный относитель- но этого диаметра последнему отрезку, проведенному вторым игроком. Вариант 7 1. Возможный вариант: 777 + 777 + 777 - 77 - 77 - 77 - 77 - 7 - - 7 = 2009.
90 Математические олимпиадные работы. 5-11 классы 2. Всего комнат 100. Все комнаты, кроме тех, что находятся вдоль сторон квадрата, имеют по 4 общие стены, поэтому в них по 4 двери. Таких комнат 64. В угловых комнатах (их 4) — по 2 двери. И в оставшихся 32 комнатах, расположенных вдоль сторон квадрата, — по 3 двери. Учитывая, что дверь соединяет две комнаты, получаем: (64 • 4 + 4 • 2 + 32 • 3): 2 = 180. Ответ: 180 дверей. 3. Пусть х см — ширина параллелепипеда, тогда 4х см — его вы- сота, а 8х см — длина. Так как длина и высота вместе состав- ляют 60 см, получаем уравнение: 4х + 8 х = 60, из которого находим х = 5 см. В результате получаем, что объем паралле- лепипеда равен 4000 см3. 4. Так как суммирование двух четырехзначных чисел дает пяти- значное число, то М s= 1, а О > 4. Так как II + Н = О, то О — циф- ра четная. Таким образом, О равно 6 или 8. Рассмотрим первый случай. Пусть 0’6. Тогда Н = 3 и Д = 8. Так как И + И = Г и Г < 8, то И = 1,2 или 3. Но цифры 1 и 3 заняты. Остается И = 2. Рассмотрим второй случай: 0 = 8. Тогда, с одной стороны, II = 4 (так как Н + Н = О), а с другой — Н = 6 или 7 (так как О^О = Н), чего не может быть. Значит, этот случай отпадает. + 6823 Ответ: 6823. 13646 5. Решение задачи понятно из схемы (рис. 28). Ведро 1 Шаг 2 Шаг ЗШаг 4 Шаг 5 Шаг 6 Шаг 5 л 0 л 5 л 0 л 4 л 4 л 5 л 9 л 9 л / 4 л 4 л / 0 л 9 л / Z 8 л Рис. 28 Варианте 2012 -2011 + 2010 - 2009 + ... + 4- 3 + 2-1 2012-73 + 2012-27
Решения, указания, ответы 91 _ (2012 - 2011) + (2010 - 2009) +... + (4 - 3) + (2 - 1) _ 2012 (73+ 27) __ 1 + 1 + 1 + ... + 1 _ 1006 _ 1 2012 100 ~ 2012 • 100 ~ 200 2. Если мы сложим стоимость всех книг без первой, без второй, без третьей и без четвертой, то получим утроенную стоимость всех четырех книг. Тогда стоимость всех книг составит 408 руб. Соответственно, первая книга тогда будет стоить 408 - 348 = = 60 руб., вторая — 112 руб. (408 - 296), третья — 116 руб., чет- вертая — 120 руб. Ответ: Стоимость первой, второй, третьей и четвертой книг соответственно равна 60, 112, 116 и 120 руб. 3. На первые 9 страниц понадобится 9 цифр, остается 174. 174:2 = = 87. Значит, будет еще 87 страниц. Всего в книге 9 + 87 = = 96 страниц. 4. Пусть самый хитрый Медведь. Тогда он солгал, сказав, что Лиса не самая хитрая. Значит, Лиса должна быть самой хитрой. Но Медведь и Лиса не могут быть одновременно самыми хитры- ми. Значит, Медведь не самый хитрый, и он сказал правду, что Лиса не самая хитрая. Тогда получается, что самый хитрый Волк. Действительно, так как самый хитрый солгал, получает- ся, что Лиса не хитрее Волка. А так как Лиса и Медведь сказали правду, то все сходится. Ответ: Самый хитрый — Волк. 5. Необходимо разрезать прямоугольник на три прямоугольника размером 8 х 12, 4 х 6, 4 х 6, из которых и сложить квадрат раз- мером 12 х 12. 6. Пронумеруем мешки и возьмем из каждого мешка количество монет, соответствующее его номеру, то есть из первого мешка берем 1 монету, из второго — 2 монеты, из третьего - 3 монеты и т. д. Всего, таким образом, возьмем 1+2+3+4+5+6+7+ + 8 + 9+10 = 55 монет. Если бы все монеты были настоящие, то их масса была бы 550 г, но среди них есть фальшивые монеты, поэтому масса монет будет больше. Если масса будет больше на 1 г, то фальшивые монеты в первом мешке, если на 2 г ~ во втором мешке и т. д. Таким образом, определив разницу, мы узнаем номер мешка с фальшивыми монетами.
92 Математические олимпиадные работы. 5-11 классы Вариант 9 1. Таких чисел будет шесть: 123, 132, 213, 231, 312, 321. Их сумма равна 1332. 2. Первый Муравей пришел в гости раньше, так как пока второй Муравей половину пути ехал на Гусенице, первый приполз к Стрекозе, потому что его скорость была больше в 2 раза. 3. Всего будет 10 острых углов: /АОВ, /АОС, /.AOD, /АОЕ, ZВОС, /BOD, ZВОЕ, Z COD, Z СОЕ, /.DOE. 4. Так как при умножении четырехзначного числа на трехзнач- ное столбиком получилось всего два слагаемых, то вторая цифра второго множителя будет 0. Так как произведение циф- ры 3 и последней цифры второго множителя оканчивается на 1, то последняя цифра второго множителя будет 7. Так как произведение цифры 3 на первую цифру второго множителя равно 6, то первая цифра второго множителя будет 2. Таким образом, второй множитель равен 207. Так как произведение первого множителя на 207 заканчивается на 21, то предпослед- няя цифра первого множителя будет 0. А так как произведе- ние первого множителя на 207 начинается с 621, то первые две цифры первого множителя могут быть только 3 и 0. Таким об- разом, получается произведение: Y 3003 207 21021 6006 . 621621 5. Да, можно сначала проделать операцию со второй, затем — с четвертой строкой, и, наконец, с четвертым столбцом. Вариант 10 1. Нет, так как это число должно быть одновременно четным и не- четным. 2. На одну пустую бутылку лимонада не купить. Сам лимонад без бутылки стоит 30 - 12 = 18 рублей. Поэтому можно купить (100- 12): 18 = 4 бутылки, на пятую не хватит денег. Сначала можно купить 3 бутылки, а затем на сданные бутылки еще одну
Решения, указания, ответы 93 3. Так как произведение цифр трехзначного числа равно 3, то эти цифры: 1, 1 и 3. А из них можно составить следующие числа: ИЗ, 131,311. 4. Найдем, какая длина носа будет после каждого обмана: 1) 9x2=18 см — длина носа Буратино после того, как он об- манул в первый раз; 2) 18 х 2 = 36 см — длина носа Буратино после того, как он обманул во второй раз; 3) 36 х 2 = 72 см — длина носа Буратино после того, как он обманул в третий раз; 4) 72 х 2 = 144 см — длина носа Буратино после того, как он обманул в четвертый раз. Но 1 м 4 дм = 140 см. Так как 144 > 140, то Буратино перестал обманывать. Ответ: Буратино обманул 4 раза. 5. Четырежды приложив шаблон, нарисуем прямоугольники раз- мером 5 х 6 и 6 х 5, расположенные, как показано на рис. 29. Осталось, пользуясь стороной фанерного прямоугольника как линейкой, продолжить их стороны, чтобы в правом верхнем углу образовался квадрат со стороной 1. Рис. 29 6. На первое заседание не явилось 20 % депутатов, на второе — 30 %. Если считать, что это были разные депутаты, то на обоих заседаниях как минимум было 50 % депутатов. Максимально же могло быть 70 % депутатов на обоих заседаниях, если все пришедшие второй раз были и в первый раз. Таким образом, всего депутатов на обоих заседаниях было от 50 до 70 %.
94 я Математические олимпиадные работы. 5-11 классы 6 класс Вариант 1 1. 2009 - 2007 + 2005 - 2003 + 2001 - 1999 + ... + 9 - 7 + 5 - 3 + 1 = « (2009 - 2007) + (2005 - 2003) + (2001 - 1999) + ... + (9 - 7) + + (5 - 3) + 1 = 2 + 2 + 2 + ... + 2 + 2 + 1 = 2 • 502 + 1 = 1005. 2. 100 000 л = 100 000 куб. дм =100 куб. м. Площадь бассейна рав- на 1000 кв. м. Значит, высота бассейна будет 0,1 м (или м). При такой глубине соревнования проводить нельзя. 3. Так как КА + КА + КА оканчивается на КА, то КА = 50, а зна- чит, К = 5, А = 0. Так как Ш + L1H ПН 1 оканчивается на 0, то Ш = 3. Так как сумма трех чисел, начинающихся на 5, может начинаться лишь с 1, то С = 1. Рассматривая варианты для О, получаем, что О = 6, а значит, Б = 9. Итак, получаем: 56350 + 56350 56350 169050 4. Так как Портос с д'Артаньяном легко перетянули Атоса с Ара- мисом, но Портос с Атосом перетянули Арамиса с д'Артаньяном уже с трудом, то д'Артаньян сильнее Атоса. Так как силы Пор- тоса с Арамисом и Атоса с д'Артаньяном равны, в одной паре должны быть два мушкетера: самый сильный и самый слабый, а в другой — двое средних по силе. Так как в первых двух слу- чаях перетягивания каната победила пара, где был Портос, то самый сильный — он. Тогда Арамис — самый слабый. Ответ: Да, можно. Портос сильнее д'Артаньяна, д'Артаньян сильнее Атоса, а Атос сильнее Арамиса. 5. Обозначим возраст сына за х лет, тогда возраст отца будет 4х. Так как суммарный их возраст составляет 50 лет, то имеехм уравнение х + 4х = 50. Из уравнения получаем х = 10. Итак, вначале сыну было 10 лет, отцу — 40. Пусть отец станет старше сына в 3 раза через п лет, тогда 3(10 + п) = 40 + п. Решением уравнения будет п = 5. То есть отец будет старше сына в 3 раза через 5 лет.
Решения, указания, ответы 95 6. Если бы Пете досталась не седьмая, а пятая часть всех груш, то он получил бы пятую часть всех фруктов. Но это больше, че^м он получил на самом деле. Значит, на самом деле доля каждого вну- ка составляет меньше пятой части всех фруктов. Поэтому вну- ков больше пяти. Однако если бы Пете досталась седьмая часть всех яблок, он получил бы седьмую часть всех фруктов. Но это меньше, чем ои получил на самом деле. Поэтому внуков меньше семи. А если внуков больше пяти, но меньше семи, то их шесть. Вариант 2 1. Возможное решение: 2004 =5х5х5х5+5х5х5х5+5х5х х5х5 + 5х5х5 + 5 + 5:5“5:5-5:5. 2. Есть 2 варианта решения: необходимо выбрать кубики под бук- вами б, в и д или а, г и д. 3. Посчитаем, сколько всего кофе выпил Буратино. В первый раз i 1 р он выпил полчашки кофе, iо есть —. Во второй раз он выпил тре- 2 111 тью часть разбавленного кофе, а самого кофе --= — . И в тре- 3 2 6 тип раз он допил чашку с кофе, при этом кофе там осталось . 1 1 1 _ г а к 11 1--- ---. 1аким образом, кофе Буратино выпил —+ —+ 2 6 3 2 6 1 < 1 1 1 < + ~ - 1. Столько же ои выпил молока: — + — + — = 1. Поэтому 3 2 3 6 У Буратино молока и кофе выпил одинаково. 4. Возможные варианты приведены на рис. 30. Рис. 30
96 Математические олимпиадные работы. 5-11 классы 5. При первом взвешивании положим на левую чашу гирю в 200 г и уравновесим весы с помощью крупы, тогда крупы на левой чаше будет 4400 г, а на правой — 4600 г. Теперь 4400 г разделим пополам: тогда на каждой чаше будет по 2200 г крупы. При тре- тьем взвешивании отвесим с помощью гири 200 г крупы, тогда получим массу оставшейся крупы в одной из кучек: 2000 г = 2 кг. G. Найдем число концов у всех мостов: 5+4x4+3x3+1 = 31 — это число является нечетным. Так как число концов у всех мостов должно быть четным, то такого расположения мостов быть не может. Вариант 3 1. 23232323 = 23 х 1010101, а 61616161 = 61 х 1010101. Тогда дробь 23 сокращается на 1010101 и получается в результате —. 2. Через два часа счетчик автомобиля будет показывать число, которое начинается на 13 и оканчивается на 31, так как следую- щая возможная пара (14 и 41) не удовлетворяет условию зада- чи (за 2 часа автомобиль не может проехать больше 1000 км). Таким образом, получаем число 13*31. Определим, какая циф- ра может стоять в разряде сотен. Для этого рассмотрим все воз- можные варианты: 1) 13 031 - 12 921 = 110 км, 110:2 = 55 км/ч — скорость автомобиля; 2) 13 131 - 12 921 = 210 км, 210:2= 105 км — скорость автомобиля; 3) 13 231 - 12 921 =310 км, 310:2= 155 км/ч -- скорость автомобиля; 4) 13 331 - 12 921 =410 км, 410 : 2 = 205 км/ч — скорость автомобиля. Данный случай, как и все предыдущие, является нереальными. Ответ: 55, 110, или 155 км/ч. 3. Найдем количество крахмала, содержащегося в 9 кг риса: 9 • 0,75 = 6,75 (кг). Такое же количество крахмала будет и в яч- мене. Так как это составляет 60 %, то всего ячменя будет 7,75:0,6= 11,25 (кг). Ответ: Ячменя необходимо взять 11,25 кг.
Решения, указания, ответы 97 4. Рассмотрим, как можно получить денежные суммы от 8 до 15 тугриков: 8 = 3 + 5, 9 = 3 + 3 +3, 10 = 5 +5, 11=3 + 3 + 5, 12 = 3 + 3 + 3 + 3, 13 = 3 + 5 +5,14 = 3 + 3 + 3 +5,15 = 5 + 5 + 5. Тогда суммы 5 > 15 можно получить в виде набора монет из 8 и 5 - 8 тугриков. При этом 5 - 8 = 8п + k, где пе N, k = 0; 1; 2; 3; 4; 5; 6; 7. А 8п и k + 8 представляются через монеты 3 и 5 тугри- ков. Например, 16 = 8 + 8 = 3 + 5 + 3 + 5, 17 = 8 + 9 = 5 + 3x4, 50 = 8 +42 = 8 + 5x8 + 2 = 5x8 +5+ 5 и т. д. 5. Пусть в каждом месяце дни рождения отмечают не более 2 уче- ников. Так как месяцев в году 12, то учеников в классе будет не более 24. Получается противоречие. Значит, найдется месяц, в котором отметят дни рождения не менее 3 учеников. 6. Найдем закономерность: 21 = 2, 22 = 4, 23 = 8, 24 = 16, 25 = 32, то есть последняя цифра степени двойки повторяется че- рез 4 шага: 2, 4, 8, 6 и снова 2. Так как 2011 = 4 х 502 + 3, то есть при делении на 4 дает остаток 3, то 22011 оканчивается той же цифрой, что и 23, то есть цифрой 8. Вариант 4 t _________666666 -666666_________ 1+2+3+4+5+6+5+4+3+2+1 ____________777777 - 777777_________= 1 + 2 + 3 + 4 + 5 + 6 + 7 + 6 + 5 + 4 + 3 + 2 + 1 666666 - 666666 777777 - 777777 6-6 7-7 = 111111-111111 111111111111^ 0 1-1 1-1 2. Сережа сделал призовых выстрелов: 17 - 5 = 12, поэтому по- паданий в цель было: 12:2 = 6. 3. 2009 = 999 + 999 + (9 : 9) • (9 :9) + 9 : 9 + 9. 4. Рассмотрим все 4 варианта: 1) Первая и вторая белки сказали правду в первый раз, а во второй соврали. Тогда получается, что первые заяц и лось, чего не может быть.
98 Математические олимпиадные работы. 5-11 классы 2) Пусть обе белки первый раз ошиблись, тогда на втором ме- сте окажутся лиса и заяц. А этого быть не может. 3) Пусть первая белка ошиблась в первый раз, а вторая — во второй. Тогда получается, что первым был лось, а лиса — второй. Заяц тогда будет третьим. Все получается. 4) Пусть первая белка ошиблась во второй раз, а вторая — в первый. Тогда заяц получается одновременно и первый, и второй, чего не может быть. Ответ: Первым был лось, второй — лиса, третьим — заяц. 5. Нет, так как сумма 5 чисел из таблицы будет нечетным числом, а нечетное число на 20 не делится. 6. Решаем задачу с конца. Так как 4 яблока составляют треть от того количества, что осталось после Бори, то весь остаток - 2 это 12 яблок. Но 12 яблок составляют — яблок, оставшихся по- 3 еле Андрея, значит, после Андрея осталось 18 яблок, которые 2 в свою очередь составляют — числа яблок, купленных мамой. Значит, мама купила 18 : 2 • 3 = 27 (яблок). Ответ: Мама купила 27 яблок. Вариант 5 1. Возможный вариант: 2222 - 222 + 2- 2 - 2 + 2 + 2 : 2 = 2011. 2. Приведем сумму дробей к общему знаменателю: .1111 3 4-5 + 2-3-5 + 4-5 + 3-5 + 4-3 2 3 4 5 60 В числителе каждое слагаемое, кроме последнего, делится на 5, поэтому вся сумма не будет делиться на 5. Значит, и после за- мены некоторых знаков + на - она не будет делиться на 5. Поэ- тому она не будет делиться и на 0. 3. Каждому из купцов должно достаться по 7 бочонков, из кото- рых с квасом будут 3,5 бочонка. Возможный вариант: у первого и второго купца будет 3 полных. 3 пустых и 1 наполовину на- полненный бочонок, у третьего же — 1 полный, 1 пустой и 5 на- половину наполненных бочонков.
Решения, указания, ответы 99 4. Обозначим длину, ширину и высоту параллелепипеда соот- ветственно как х, у, h. Объем п параллелепипедов равен nxyh, а 315 = 3 • 3 • 5 • 7. При этом у > 1, х > h, h > у. Число 315 есть произведение четырех простых множителей, значит, п, х, у, h могут быть только этими множителями. Так как у < h < х, а всего различных множителей три, то у = 3, h = 5, х = 7. Пло- щадь поверхности прямоугольного параллелепипеда со сторо- нами х, г/, /? равна 2ху + 2xh + 2yh. В итоге получаем 42 + 70 + + 30 = 142. 5. Раскрасим комнаты в шахматном порядке, как на рис. 31. При этом соседние комнаты окрасятся в разные цвета. При пере- езде цвет комнаты должен поменяться, поэтому постояльцы, живущие в пяти черных комнатах, должны переехать в белые комнаты, которых четыре. Такой обмен невозможен, поэтому хозяйка гостиницы не сможет переселить их так, чтобы каж- дый постоялец переехал в соседнюю комнату. Рис. 31 Вариант 6 ( 9 А 2 1 —*0,05 * 1,4 = — ь 20 J 7 ( 9 1^72 1^20 20 J 5 7 Рассматривая различные варианты знаков, в итоге получим: ( 9 Л 2 — -0,05 :1,4 = —. (^20 J 7 2. Возможное решение задачи показано на рис. 32. 3. Так как из 450 рыбин помеченными оказалось 2 рыбины, то есть каждая 225-я, то всего в хозяйстве будет 90 • 225 = = 20 250 рыбин. 4. Решение показано на рис. 33.
100 Математические олимпиадные работы. 5-11 классы 5. Пусть цена молока, продаваемого в магазине, равна х руб. Так как магазин удерживает 20 %, то есть 0,2 х, то Матроскин по- лучит 0,8 х. А так как он хочет получить 15 руб. с литра, то 0,8 х = 15, то есть х = 18,75. Ответ: Молоко будет продаваться в магазине по 18 руб. 75 коп. 6. При замене двух любых чисел их суммой сумма всех чисел в файле не меняется. То есть5 6 7'йнвариантом является сумма чи- сел в файле. Поэтому последнее^ число — оно же сумма чисел в конце — равно сумме чисел в начале, то есть 2012. Вариант 7 1. Начнем решение задачи с конца: 1) 100:2 = 50; 2) 50 - 2 = 48; 3) 48 : 2 = 24;
Решения, указания, ответы 101 4) 24:2= 12; 5) 12:2 = 6; 6) 6 : 2 = 3; 7) 3-2 = 1. Итак, необходимо как минимум 7 действий. 2. Да, делится, так как сумма цифр числа 10020 + 1010 + 1 равна 3. 3. Дерево, которое у девочки было седьмым, у мальчика было двадцатым. Отсчитаем 6 деревьев назад. Получаем, что первое дерево у девочки было у мальчика четырнадцатым, а послед- нее дерево у девочки было у мальчика тринадцатым. Седьмое дерево у мальчика было у девочки девяносто третьим. Отсчи- таем 6 деревьев вперед. Получается, что тринадцатое дерево у мальчика — это девяносто девятое дерево девочки. Так как оно у девочки последнее, то всего деревьев вокруг поляны было 99. 4. Для решения воспользуемся чертежом, показанным на рис. 34. Р М Т N Рис. 34 Здесь Р — это момент рождения папы; М — момент рождения мамы; Т — момент, когда папе было столько лет, сколько маме сейчас; N — нынешний момент. Из условия ясно, что папа стар- ше мамы, поэтому моменты отмечены буквами именно в таком порядке. В момент Т возраст папы — это отрезок РТ, а возраст мамы — отрезок MN. По условию РТ = MN. Вычитая из от- резков РТ и MN их общую часть МТ, получим PM = TN. Усло- вие «сейчас» (то есть в момент N) означает, что PN = 2 МТ, но PN = РМ + МТ + TN, поэтому РМ + TN= МТ Обозначив РМ = х, получим, что TN = х, МТ= 2х, PN = 4х и MN = Зх. По условию PN + MN = 77, поэтому 7х = 77, а значит, х = 11. То есть папе сейчас 44 (4х), а маме — 33 года (Зх). 5. Нельзя, так как 23 нельзя представить в виде суммы пятерок и семерок. 6. Рассмотрим все возможные случаи: 1) Перевернем 2 монеты, лежащие вверх орлом, чтобы они лег- ли вверх решкой. Тогда всего монет, лежащих вверх орлом, будет на 2 меньше (одна).
102 Математические олимпиадные работы. 5-11 классы 2) Перевернем 2 монеты, лежащие вверх решкой, чтобы они легли вверх орлом. Тогда всего монет, лежащих вверх ор- лом, будет на 2 больше (пять). 3) Перевернем монету, лежащую вверх орлом, и монету, ле- жащую вверх решкой. Тогда монет, лежащих вверх орлом, снова будет пять. Таким образом, во всех трех случаях число монет, лежащих вверх орлом, получается нечетным. А так как 8 — число четное, то повернуть все монеты вверх орлом нельзя. Вариант 8 1 i + 1 * * * 5 i 1 1 1-2 + 2- 3 + 3- 4+ + 2008• 2009 + 2009• 2010 " = 1_1 1_________________________1 1__________1__ “ 1 2 + 2 3 + 3 4 + + 2008 2009+ 2009 2010 " = 1 1 _ 2009 ” 1 2010 “ 2010’ 2 . Обозначим скорость черепахи как х км/ч. Тогда 35 х км/ч — это 1 скорость зайца. Расстояние, пройденное черепахой, равно 2 — х км. Так как заяц сидел на финише некоторое время (пусть это бу- дет время t ч), то он был в пути (2 — - I) ч. Значит, он за это вре- 3 мя пробежит расстояние 35х (2 - t) км. Так как расстояния, пройденные зайцем и черепахой, равны, получаем уравнение: 2- х = 35х(2- -t). 3 3 1 1 4 Так как л 0 , то 2 — = 35 (2 — - t), откуда находим t = 2 — ч, или 2 ч 16 мин. 3. Решение представлено на рис. 35. 5 5 4. 7777777778< 888888887 =>----------->----------=> 7777777778 888888887
Решения, указания, ответы 103 7777777773 >t 8888888882 7777777773 < 7777777778 > 8888888887 7777777778 < 8888888882 С 8888888887' 5. Хозяин должен увеличить свой двор так, как показано на рис. 36. Деревья обозначены точками. Площадь старого двора (квадрат, расположенный внутри), будет в 2 раза меньше ново- го двора (больший квадрат). 6. Пусть х л дает 1 черная корова в день, а у л — 1 рыжая корова в день. Тогда за 5 дней 4 черные и 3 рыжие коровы дают 5(4т ь + 3#) л, а 3 черные и 5 рыжих коров за 4 дня — 4(3х + 5z/) л. Так как данные выражения равны, то имеем уравнение: 5(4х + 3z/) = 4(3х + 5г/), из которого получаем у = 1,6т, а это означает, что рыжие коровы более продуктивны. Вариант 9 1. Найдем дополнения каждой дроби до 1: .4 1.5 1.6 1.7 1 1--= = ---------= -,1 = -. 55 66 77 88
104 Математические олимпиадные работы. 5-11 классы А 1 1 1 1 4 5 6 7 5678 5678 2. Найдем высоту данной башни. Так как 1 м = 1000 мм, то ку- биков всего будет 1000 х 1000 х 1000 = 1000 000 000. А так как 1 000 000 000 мм = 100 000 000 см = 1 000 000 м = 1000 км, то кубиков хватит. Ответ: Хватит. 3. Задачу можно решить с помощью уравнения или же путем рас- суждений с конца задачи. Рассмотрим второй вариант. Так как в итоге в ящике остался 31 лимон, а до этого мы взяли половину оставшихся лимонов и поллимона, то перед последним разом лимонов было (31 + 0,5)-2 = 63. Аналогично рассуждая для вто- рого раза, получим, что перед тем как взять половину всех ли- монов и еще поллимона, лимонов оставалось (63 + 0,5)- 2 = 127. Тогда первоначально лимонов было (127 + 0,5)- 2 = 255. Ответ: Первоначально лимонов в ящике было 255. 4. Так как квадрат двузначного числа — это число четырехзнач- ное, начинающееся с той же цифры, что и само число, то П = 9. Так как произведение двух одинаковых цифр оканчивается на ту же цифру, то А может быть 0, 1,5 или 6. С помощью подста- новки убеждаемся, что первые два варианта не подходят. Итак, получается два варианта: 952 = 90 25 пли 962 = 9216. 5. Нули будут получаться в результате умножения пятерок на четные числа. Найдем количество пятерок в произведении. Пятерки содержатся в множителях: 5, 10, 15, 20, 25, 30,..., 2010. Таких множителей 402. Но в множителях 25,50, 75,..., 2000 при разложении на простые множители будет не одна пятерка, а больше. Таких множителей 80. Учтем, что в множителях 125, 250, ..., 2010 содержится по 3 пятёрки (таких множителей 16), а в множителях 625, 1250, 1875* пятерок уже 4. Тогда всего пятерок при разложении всех чисел от 1 до 2011 на простые множители будет 402 + 80+ 16 + 3 = 501. Так как четных про- стых множителей — двоек много больше, то произведение всех чисел от 1 до 2011 будет заканчиваться 501 нулем. 6. Пусть копь стоит на клетке белого цвета, после очередного хода он окажется на клетке черного цвета, так как конь ходит бук- вой Г. Таким образом, при движении коня цвета клеток будут чередоваться. Если конь побывает на всех клетках по одному
Решения, указания, ответы 105 разу, то он сделает 48 ходов и окажется на клетке белого цве- та, с которой на клетку белого цвета ему не попасть. Значит, Карлсон не сможет обойти шахматным конем всю шахматную доску размером 7x7 так, чтобы конь побывал на каждой клетке по одному разу и вернулся на начальную клетку. Вариант 10 1. Решение показано на рис. 37. 2. Так как в изюме 4 % воды, то сухого вещества там 96 %, то есть 43,2 кг. Так как в винограде сухого вещества 60 %, а это 43,2 кг, то всего винограда взято 43,2 : 0,6 = 72. Таким образом, вино- града было 72 кг. Ответ: Было использовано 72 кг винограда. 3. Так как от потери ящика до его нахождения прошел 1 час, а ящик за это время уплыл на 3 км, то скорость течения реки составляет 3 км в час. Ответ: Скорость течения реки равна 3 км в час. 4. Первыми за 2 минуты переходят мост папа и мама, затем папа возвращается за 1 минуту, сын и дочь переходят мост за 8 ми- нут, мама возвращается обратно за 2 минуты, папа и мама пере- ходят мост вновь за 2 минуты. Всего получается 15 минут. 5. Так как наибольшая цифра в числе «СИЛЕН» равна 5, а С = 1, то остальные 4 цифры в данном числе будут 0, 2, 3, 4 или 5. Так как Н < 6, то И = 2. А значит, Н = 4. Так как Л>Е(Е+1=Л и Е < 5), то Л = 5 и Е = 3. А тогда уже легко находим остальные цифры: UJ = 8, Р = 9. В итоге получается: . 9382 3152 12534.
106 Математические олимпиадные работы. 5-11 классы 6. Сумма двух соседних чисел будет четной, если они оба четные или оба нечетные. Сумма двух чисел будет нечетной, если одно из них четное, а другое — нечетное. Если сумма любых двух соседних чисел нечетная, то четные и нечетные числа должны чередоваться, поэтому общее число чисел должно быть четное. По условию же чисел 2011, а 2011 — число нечетное. Значит, найдутся два соседних числа, сумма которых четная. 7 класс Вариант 1 1. В 12:00 стрелки часов сходятся вместе. После этого за 40 минут 2 минутная стрелка проходит — окружности, то есть описывает 3 угол в 240°. Часовая стрелка движется в 12 раз медленнее ми- нутной (так как описывает круг за 12 часов). Поэтому она за 40 минут опишет угол в 240 : 12 = 20° и образует с минутной стрелкой угол в 120 + 20 = 140°. 2. Преобразуем левую и правую части: 7777777773 _ t 5 . 8888888882 _ t 5 -777777778 - 7777777778, 8888888887 “ 8888888887’ 8888888887 > 7777777778 => 5 . 5 * * В 8888888887 < 7777777778 5 t 5 8888888882 8888888887 > 7777777778 8888888887 7777777773 7777777778’ 3. 1 + 2 + 22 + ... + 22011 + 22012 = (1 + 2) + 22 (1 + 2) + ... + 22(И0 (1 + + 2) + 22012 = 3 (1 + 22+ ...+ 22010) + 22{)12. В данной сумме первое слагаемое делится на 3, а второе слагае- мое не делится на 3. Поэтому сумма не делится на 3. 4. Четырехзначное число, одна из цифр которого 0, может иметь вид: aQbc, abOc или ahcQ. Последний случай невозможен, так как при вычеркивании нуля число abcO уменьшится в 10 раз.
Решения, указания, ответы 107 Используя условие задачи, имеем: aObc = abc х 9. Из данного равенства получим, что с = 5. Тогда аОЬ5 = аЬ5 х 9. Найдем Ь. Так как 96 + 4 оканчивается на Ь, то b = 2 или 7. Для первого случая получаем равенство: «025 = «25 х 9. Из данного равен- ства подбором находим а = 2. Получаем число 2025. Проводя аналогичные рассуждения для Ь = 7, находим а = 6, а число по- лучаем 6075. Для второго случая чисел не найти. Ответ: Нуль стоит на месте сотен, искомые числа: 2025 и 6075. 5. Рассмотрим три случая. Первый случай. Пусть учитель сказал верно Алексею. Значит, у Алексея «4». Так как Борису и Василию учитель назвал невер- ные отметки, то у Бориса «4», а у Василия «5». Получилось, что у двух учеников оказались одинаковые отметки, что противо- речит условию задачи. То есть данный случай невозможен. Второй случай. Пусть учитель сказал верно Василию. Тогда у Василия не «5». Так как учитель сказал неверно об отметках Алексея и Бориса, то у Алексея не «4», а у Бориса «4». Тогда выходит, что у Алексея отметка «5», а у Василия «3». Третий случай. Пусть учитель сказал верно про отметку Бо- рису. Тогда Борис получил не «4». Так как утверждения про отметки Алексея и Василия ложные, то Алексей получил не «4», а Василий «5». Получается, что отметку «4» не получил ни один из учеников. Этот случай также противоречит усло- вию задачи. Ответ: Алексей получил «5», Борис «4», а Василий «3». Вариант 2 1. Наибольшее целое число, заключенное между -9,1 и -15,1, есть -10, а наименьшее простое число в третьем десятке нату- ральных чисел — это 23. Подставив данные значения вместо х и у, после упрощений получим 10 590 580. 2. Пусть первоначальная цена товара была х руб. Тогда после по- дорожания на 10 % она будет 1,1 х руб. А после того как товар подешевеет на 10 %, цена товара будет 1,1 х - 0,11 х = 0,99 х руб. Так как 0,99 х < х, то цена товара получается ниже после сниже- ния, чем до подорожания. Ответ: Цена ниже после снижения.
108 Математические олимпиадные работы. 5-11 классы 3. Максимальный выигрыш равен 1800 руб. (100 + 200 + 500 + + 1000 = 1800) тогда, когда названное число делится и на 2, и на 10, и на 12, и на 20. Например, таким числом является 60. Ответ: 1800 руб. 4. Данный угол не может быть при основании равнобедренного треугольника, так как в этом случае сумма внутренних углов треугольника была бы больше 180°. Значит, данный угол на- ходится при вершине. Тогда смежный с ним внутренний угол треугольника равен 148°, соответственно, углы при основа- нии — по 16°. Значит, угол между основанием треугольника и высотой треугольника, проведенной из вершины угла при основании, равен 180 - 90 - 16 = 74°. 5. Искомый угол равен разности углов АОВ и АОС (рис. 38). Рис. 38 ХАОВ = + 30°'38 = 49°, ХАОС = — 360° -180° = 48°. 12 60 30 Значит, искомый угол равен Г.
Решения, указания, ответы 109 6. Возможные варианты: 1) (9999 : 9 - 999 : 9) • (9 + 9) : 9 + 9 = 2009; 2) 999 + 999 + (9 : 9) • (9 :9) + 9 : 9 + 9 = 2009; 3) 999 + 999 + 9 + 9 : 9 + 9 : 9 + 9 - 9 = 2009. Вариант 3 1. Рассмотрим один из возможных вариантов. Строим сначала угол 135° = 3 • 45°. Затем строим угол 136° = 4 • 34°. Теперь строим разность 1° = 136° - 135°. Отложив 5 раз угол по 1°, по- лучаем требуемый угол 5°. 2. Обозначим за 5 км расстояние, пройденное путником в одном s направлении, тогда — часов — это время, затраченное на подъ- s ем, а — часов — время, затраченное путником на спуск. Сле- довательно, время, затраченное на подъем и на спуск, равно — + — часов, а пройденное расстояние — 2s км. Значит, сред- 9 (s 5 V 4v няя скорость путника равна “ + 7^* -“^"км/3 4- 3. Предположим, что мы соединили 9 телефонов так, что каждый из них связан ровно с тремя другими. Посчитаем количество проводов, соединяющих эти телефоны. Их число должно быть четное, а так как от каждого из 9 телефонов отходит 3 конца, по- лучаем 27 — число нечетное. Поэтому 9 телефонов соединить между собой попарно так, чтобы каждый был соединен ровно с тремя другими, нельзя. 4. Надо посчитать, со сколькими прямыми встретятся диагонали и на сколько отрезков разделится каждая из них. Поскольку числа 4 и 5 взаимно простые, то каждая диагональ разделится на4 + 5-1 = 8 отрезков. Каждый из этих 16 отрезков двух диа- гоналей отделит еще одну часть данного прямоугольника. Зна- чит, к имеющимся уже 4 • 5 = 20 частям добавится еще 16 ча- стей. Итого получается 36 частей (рис. 39). Ответ: Прямоугольник разделится на 36 частей.
110 Математические олимпиадные работы. 5-11 классы 5. Число 2 х 12 х 22 х 32 х 42 х ... х 2002 х 2012 - 2 будет делиться на 10, если оно оканчивается на 0. Найдем последнюю цифру числа 2 х 12 х 22 х 32 х 42 х ... х х 2002 х 2012. Она будет совпадать с последней цифрой чис- ла 2202. Так как 202 = 4 х 500 + 2, то последняя цифра числа 2202 совпадет с последней цифрой числа 22 (последняя цифра степе- ни двойки равна 2,4,8, 6, 2,4..., то есть повторяется через 4). Так как 22 = 4, то число 2 х 12 х 22 х 32 х 42 х ... х 2002 х 2012 окан- чивается на 4, поэтому число 2 х 12 х 22 х 32 х 42 х ... х х 2002 х х 2012 - 2 оканчивается на 2, а значит, оно не делится на 10. Вариант 4 1. Обозначив весь путь от Л до В за 5, найдем время движения по- езда из Л в В и из В в А. Оно будет равно, соответственно, — 5 5 и — , а общее время поезда — . Тогда средняя скорость по- 2s езда будет равна —, то есть 48 км/ч. 24 2. Обозначим за х число мужчин, за у — число женщин, а за г — число детей, тогда в соответствии с условием, получим: х + ул- z = 12, 2х + — у + — г = 12. 2* 4
Решения, указания, ответы 111 Умножив первое уравнение на 2, и вычтя из него второе урав- < / 1 v нение, получим после преооразовании: х = 4 -t — z. Учитывая, 6 что в переносе участвовали все 12 человек, ах, z — целые числа, получаем: z = 6 и х = 5. Тогда у = 1. Итак, в переносе хлеба уча- ствовало 5 мужчин, 1 женщина и 6 детей. 3. Обозначим за х км весь путь. Тогда в соответствии с условием задачи получим уравнение: х + 40 + 0,75х -118 = х. Решением данного уравнения будет х = 360. Ответ: 360 км. 4. Найдем площадь пола: 20'15 = 300 м2 и площадь одной плитки 0,2 • 0,2 = 0,04 м2. Тогда плиток потребуется: 300 : 0,04 = 7500. Ответ: 7500 плиток. 5. Возможный вариант расположения точек представлен на рис. 40. 6. Число 4” + 5 делится на 3, если число 4" - 1 = (4Л + 5) - 6 делит- ся на 3. Но 4" - 1 = (2я - 1)(2" +1). Так как 2" не делится на 3, то одно из чисел 2п - 1 или 2" + 1 делится на 3. А тогда на 3 делится и число 4м - 1. Следовательно, число 4" + 5 делится на 3. Вариант 5 1. Так как длина, ширина и толщина куска мыла уменьшаются вдвое, то объем уменьшается в 8 раз. Так как было 7 стирок, то объ- 7 < 1 < ем уменьшился на — исходного объема, то есть на — объема за 8 8 одну стирку Поэтому оставшегося мыла хватит на одну стирку
112 Математические олимпиадные работы. 5-11 классы 2. Первое утверждение не может быть верным, так как в этом случае и второе оказывается верным. Значит, первое утверж- дение ложно, то есть Алексей — не старшина. Пусть верно вто- рое утверждение: «Владимир — не старшина», тогда ложны и первое, и третье утверждения. Поэтому Алексей — не стар- шина, а Семен — сержант. Но тогда получается, что Владимир и Алексей — оба капитаны, варианта этого не может быть. Рас- смотрим последний вариант: верно то, что Семен — не сержант, а ложны первые два утверждения. Тогда Алексей — не старши- на, а Владимир — старшина. Тогда получается, что Семен — ка- питан, а Алексей — сержант. Все сходится. 3. Пусть п — четное число, тогда левая часть равенства является числом четным, а правая — нечетным. Если же п — число не- четное, то левая часть является числом нечетным, а правая — четным. Значит, такого числа не существует. 4. Пусть первая часть пути, которую проехал автобус, составляет х км, вторая — у км. Тогда первую часть пути он ехал у мин, вторую — х мин. Следовательно, его скорость на первой части х . у . А пути равна — км/мин, на второй — — км/мин. А так как ско- У х рость движения автобуса была неизменной, то — = ~. Учиты- У х вая, что х и у — натуральные числа, имеем х = у. Если х = у = 1, то скорость автобуса равна 1 км/мин = 60 км/ч. Если же х = у = 2, то скорость автобуса равна 2 км/мин = 120 км/ч, а с та- кой скоростью перевозить пассажиров нельзя. А при х = 3, 4 и выше получаются скорости, нереальные для движения. Поэто- му мы имеем единственное решение. Ответ: Скорость автобуса равна 1 км/мин. _ Q 1 .1 .116 -118 5 Q 1 . 1 5. 3----4------1-----5----------= 3-----4------ 117 119 117 119 119 117 119 12 н---н------------------= 12 +---1----1- 117 119 117119 119 117 119
Решения, указания, ответы 113 1 6 2 1 5 10 117119 117 119 117119 119 117 6. Так как 108° -10= 1080° = 360° • 3, то десять углов по 108° долж- ны составить три полных угла. Циркулем чертим окружность с центром в вершине данного угла. Окружность пересечет сто- роны угла в точках Л и В. Углу 108° будет соответствовать дуга АВ. Отложим на окружности эту дугу циркулем от точки В де- вять раз. Если после этого мы попадем в точку А, то данный угол будет равен 108°, а если не попадем -- то не будет. Вариант 6 1. Если число делится на 6, 7 и 9, то оно делится и на наименьшее общее кратное чисел 6, 7, 9, то есть 126. Если разделим 51 000 на 126 с остатком, то получим в частном 404 и остаток 96. Что- бы число 510** делилось без остатка, оно должно быть больше 51 000 на 126 - 96 = 30, то есть должно быть числом 51 030. Если же мы прибавим еще 126, то получим число большее, чем 510**. Таким образом, надо поставить цифры 3 и 0. 2. Числа а + b и а - b всегда одной четности. Так как оба числа (и а + 6, и а - Ь) простые, то они обязательно нечетные (среди простых чисел лишь одно четное число — 2). Значит, надо ис- кать простые числа а и b разной четности, то есть 6 = 2. Так как из трех чисел: а - 2, а, а + 2 одно обязательно делится на 3, то а = 5. Других чисел а нет. 3. Обозначим скорости велосипедиста и мотоциклиста за х и у км/ч. Так как за полчаса велосипедист проехал на 3 км км меньше половины пути, а мотоциклист за — ч проехал на 2 3 1 больше половины пути, то получим уравнение 0,5х + 3 = —у - 2. 3 Выразим из данного уравнения у = 1,5х + 15. Обозначим время встречи за t ч. Тогда расстояние между пунктами А и В равно (х + у)-1= (2,5х + 15)-1. Найдем расстояние АВ иначе: так как 0,5х + 3 составляет половину АВ, то АВ равно х + 6 км. Из урав- нения (2,5.x + 15)-t = х + 6 находим искомое время: х+6 _ х + 6 2,5х+15 2,5(х + 6) = | (ч) = 24 (мин). Ответ: Встреча произойдет через 24 мин.
114 Математические олимпиадные работы. 5—11 классы 4. Рассмотрим на плоскости равносторонний треугольник с дли- ной стороны 1. Так как треугольник имеет 3 вершины, то, по крайней мере, 2 из 3 вершин такого треугольника окрашены в один цвет. 5. Обозначим одну из 17 кукол Мальвины буквой А. Так как от куклы А к другим 16 куклам идет 16 лент разных цветов, то по принципу Дирихле среди этих 16 лент найдется как минимум 6 лент одного цвета. Обозначим кукол, к которым идут ленты одного цвета, за Л., Л.., A.v А., А_, А... Первый случай. Если одна из лент, соединяющих кукол At и А, имеет тот же цвет, что и выбранные нами 6 кукол, то куклы А, Ai и Л. связаны лентами одного цвета. Второй случай. Все соединяющие куклы At и А. ленты должны быть одного из оставшихся двух цветов. Из At выходит 5 лент к куклам Л2, А3, Л4, Л5, А6. Среди них можно выбрать 3 ленты одного цвета, выходящие к куклам Ах, Ayi Az. Получаем следую- щие два варианта: 1) Все 3 ленты, соединяющие Ах, Ayt Az, имеют одинаковый цвет, что и требовалось доказать. 2) Одна из лент, соединяющих Аг, А , Az, имеет тот же цвет, что и лента, соединяющая А* с Аг, Ау, А_. В этом случае концы этой ленты и At соединены лентами одного цвета. Вариант 7 1. Нет, так как если сложим 5 нечетных чисел, то получим нечет- ное число, а 30 ~ число четное. 2. ~ 20092009 2009 10001 2009 1ак как---------=------------ =----, то числа равны. 20102010 2010-10001 2010 3. Соли в 60 кг морской воды было 60 • 0,05 = 3 кг. Эти 3 кг в раз- веденной морской воде составят 4 %, то есть всей разведенной морской воды будет 3 : 0,04 = 75 кг. Значит, придется добавить 75 ~ 60 = 15 (кг) пресной воды. Ответ: Необходимо добавить 15 кг пресной воды. 4. х3 - 7х + 6 = (х3 - х) - (6х - 6) = х(х2 - 1) - 6(х - 1) = (х - 1 )(х(х+ + 1) - 6) = (х - 1)(х5 + х - 6) = (х - 1)(х2 - 4 + х - 2) = (х - 1) х X ((х - 2)(х + 2) + (х - 2)) = (х - 1)(х - 2)(х + 3).
Решения, указания, ответы н 115 5. Пример разрезания показан на рис. 41. Рис. 41 Вариант 8 (4 З22 + 7 З21) 57 (4 З22 + 7 З21) 3 19 _ L (19 274)2 ” 192 З24 _ 4 З22 + 7 32i _ 321(12 + 7) _ 19 З23 ” 19-З23 ~9 2. После первого покупателя осталось 75 % полотна, второму про- дали 75 • 0,3 = 22,5 (%). Поэтому осталось 75 - 22,5 = 52,5(%). Так как третий купил 40 % этого остатка, то он купил 52,5 • 0,4 = = 21 (%). Таким образом, осталось полотна 52,5 -21= 31,5 (%). Ответ: Непроданным осталось 31,5 % полотна. 3. Так как число делится на 72, то оно делится на 9 и на 4. Значит, сумма цифр нового числа делится на 9, а число, образованное последними двумя цифрами, делится на 4. Этим двум услови- ям удовлетворяют числа 8100,4104 и 9108. С помощью провер- ки убеждаемся, что из них на 72 делится лишь 4104. 4. Среди пяти точек найдутся три, абсциссы которых все четные или все нечетные. Тогда среди этих трех точек найдутся две с абсциссами и ординатами одинаковой четности. Л значит, середина отрезка, соединяющего эти две точки, будет иметь целые координаты.
116 Математические олимпиадные работы. 5-11 классы 5. Произведем шахматную раскраску доски (рис. 42). Рис. 42 Заметим, что при выполнении требуемых условий шнур будет соединять 2 клетки разного цвета. Поэтому вдоль шнура цвет клеток должен чередоваться, а значит, число клеток одного цве- та должно отличаться от числа клеток другого цвета не более, чем на 1. Однако на данной доске эти числа будут отличаться на 3, так как все вырезанные клетки одного цвета (24 черные клетки и 21 белая клетка). Следовательно, разложить шнур требуемым образом нельзя. Вариант 9 1. При х > 0 уравнение имеет вид 0 = 0, если х Ф 1. При х < О уравнение решения не имеет. Ответ: [0; 1)и (1;°°). 2. Если в пакетах было соответственно а, b и с мандаринов, тогда а = 2b, b = с + 8, а = 2с + 16. Пусть х — стоимость одного манда- рина из второго пакета, тогда (Зя + 5с + xb) х 1,088 = 4(я + b + + с). Так как а + b + с = 136, то, подставив в это уравнение b = = с + 8, а = 2с + 16, найдем сначала с = 28, а затем а = 72 и b = 36. Получим х = 4 руб. 3. Не всегда. При перестановке сохраняется четность номера ме- ста. Поэтому если самый высокий человек стоит, например, вторым, то он никогда не станет первым. Здесь число 2009 роли не играет.
Решения, указания, ответы 117 4. Если 5 км — весь путь путешественника, то в первый день он прошел (0,25 + +2) км, во второй — 0,45 км, в третий — (0,15 + + 2,5) км, в четвертый — 18 км, поэтому 0,25 + 2 + 0,45 + 0,15 + + 2,5 +18 = 5 Откуда находим 5 = 75. 5. Заметим, что 1п может оканчиваться только на цифру 7, 9, 3 или 1. Причем на 1, только если п кратно 4, поэтому только в этих случаях 7" - 1 делится на 5. 6. Решение приведено на рис. 43. Рис. 43 Вариант 10 1. Решаем задачу с конца с помощью таблицы: Номер маль- чика Число яблок в конце Число яблок до передачи их третьим мальчиком Число яблок до передачи их вторым мальчиком Исходное число яблок 1 8 8:2 = 4 4:2 = 2 2 + 4 + 7= =13 2 8 8:2 = 4 4 + 2 + 8 = 14 14 : 2 = 7 3 8 8 + 4 + 4 = 16 16:2 = 8 8:2 = 4
118 Математические олимпиадные работы. 5-11 классы Таким образом, у первого, второго и третьего мальчиков перво- начально было соответственно 13, 7 и 4 яблока. 2. Возможный вариант: 1, 3 и 6 кг. Действительно: 10 = 6 + 3 + 1, 9 = 6 + 3, 8 = 6 + 3-1,7 = 6+1,6 = 6, 5 = 6-1,4 = 3+1,3 = 3, 2 = 3- 1, 1 = 1. 3. Пусть искомое число равно abc. Так как последняя цифра у куба искомого числа — это 7, то с = 3. В этом случае у куба искомого числа предпоследняя цифра совпадает с последней цифрой числа 21ft +2 и равна 7, поэтому ft = 5. В то же время последняя искомая цифра куба равна последней цифре числа 21а + 28 и равна 7, что возможно только для числа а = 7, то есть искомое число равно 753. 4. Допустим, что ни у одной из 10 сумм последняя цифра не со- впадает. А так как всего цифр десять (0, 1, 2, 3, 4, 5, 6, 7, 8, 9), то их сумма равна 45, при этом последняя цифра нечетная. По- считаем сумму иначе, сложив все числа от 1 до 10 и их места от 1 до 10. В итоге получаем сумму, оканчивающуюся на чет- ную цифру. То есть имеем противоречие. Поэтому найдется по крайней мере две суммы, у которых последняя цифра будет одинаковой. 1111 11 5. Найдем суммы чисел-и-,-и-,...,-и- 1001 2010 1002 2009 1505 1506 1 1 _ ЗОИ 1 1 _ ЗОИ 1001 + 2010 ” 1001 • 2010 ’ 1002 + 2009 ” 1002 • 2009 ’ 1 1 ЗОИ 1505 + 1506 ” 1505 1506* Так как 1001 • 2010 < 1002 • 2009 < ... < 1505 ♦ 1506, то: 11 1 ЗОИ ЗОИ 101 5 1001 1002 2010 1505 1506 301 1506 8 1 1 6. Разделим квадрат на квадратики размером — х —. Так как квад- 5 5 ратиков 25, а точек — 51, то найдется квадратик, в который по- падут как минимум 3 точки. А этот квадратик можно накрыть 1 3 прямоугольником размером ~х —.
Решения,указания,ответы 119 8 класс Вариант 1 1. Найдем последнюю цифру 3" при различных значениях п: 3; 9; 7; 1; 3; 9;... ЗамечаехМ зависимость: через 4 числа цифра повторя- ется. Так как 2008 = 502 • 4 + 0, то число З2004 оканчивается той же цифрой, что и З4, то есть 1. Рассматривая различные степе- ни числа 4, получаем зависимость: если показатель степени п — четный, то 4П оканчивается на цифру 6, а если нечетный — то на цифру 4. Так как 2009 — число нечетное, то 4П оканчивается на цифру 4, а значит, число 32()0й + 42009 оканчивается на цифру 5. 2. Начнем с буквы У, У > 5. Д = 1. Рассмотрим различные вари- анты для У: 1) У = 6, тогда Р = 2, чего не может быть (Р + Р = А, значит, А = 4, но так как Д = 1, то Д + Д = 2 или 3, но не 4). 2) У = 7, тогда Р = 4, значит, А = 8, чего не может быть, так как д=1. 3) У "= 8, тогда Р = 6, А = 2, К == 5. Все получается. В итоге имеем: . 8126 8126 16252 4) У = 9, тогда Р = 8, А = 6, чего не может быть. + 8126 Ответ: 8126. 16252 3. Один из возможных вариантов решения выглядит следующим образом: строим угол 90° с помощью циркуля и линейки, за- тем от одной из сторон угла 3 раза откладываем угол 34°. Тогда 12° = 34° -3-90°. 4. В данной задаче говорится о трех братьях (Александр, Борис и Сергей), городах, в которых они работают (Архангельск, Се- веродвинск и Котлас), и предметах, которые они преподают (математика, химия, физика). Определим сначала предмет, который преподают братья, ис- пользуя таблицу.
120 Математические олимпиадные работы. 5-11 классы Математика Химия Физика Архангелогородец Северодвинец Котлашанин Так как архангелогородец преподает не математику, то ста- вим «минус» в соответствующей клетке. Так как северодвинец преподает химию, то ставим «плюс» в соответствующей клет- ке. Поэтому северодвинец не может преподавать математику и физику, а архангелогородец и котлашанин — химию. В итоге получаем такую таблицу. Математика Химия Физика Архангелогородец — - Северодвинец — + - Котлашанин - Из таблицы следует, что архангелогородец преподает физику, а значит, математику преподает котлашанин. Теперь выясним, как зовут архангелогородца, северодвинца и котлашанина. Для этого снова воспользуемся таблицей. Учтем, что Александр работает не в Архангельске, а Борис — не в Северодвинске. Александр Борис Сергей Архангелогородец - Северодвинец - Котлашанин Так как по условию задачи Борис преподает физику, а из пре- дыдущих рассуждений мы выяснили, что архангелогородец преподает физику, получаем, что Борис живет в Архангельске. Учитывая это, имеем следующую таблицу. Александр Борис Сергей Архангелогородец - + — Северодвинец - Котлашанин —
Решения, указания, ответы 121 Больше никаких сведений о зависимостях между именами и городами у нас нет. Рассмотрим два возможных случая. 1) Пусть Александр работает в Северодвинске. Тогда Сергей будет работать в Котласе. 2) Пусть Александр работает в Котласе. Тогда Сергей будет работать в Северодвинске. Оба случая возможны. Таким образом, Сергей может работать в Котласе учителем математики или в Северодвинске учите- лем химии. 5. Пусть Ане сейчас а лет, Ване — b лет, маме — с лет. Ване было столько лет, сколько сейчас Ане b - а лет назад, но маме тогда было лет с - (Ь - а) = с + а - Ь, и это число равно а + b - 3. Та- ким образом, с = 2Ь - 3. Маме было столько лет, сколько Ване теперь, с - b лет назад, по Ване тогда было fe-c + fe = 2fe-c = 3. Итак, Ване было 3 года. Вариант 2 1. Так как 2005 = 5 • 401, а а2 - Ь2 = (а - Ь)(а + й), то для нахож- дения решения задачи надо найти решения следующих систем уравнений: fa + b = 2005, [ а - b = 1; Ь + £ = 401, [ а - b = 5. Решениями данных систем уравнений являются следующие пары чисел: (1003; 1002), (203; 198). 2. Разложим 777 на простые множители: 777 = 7 • 3 • 37. Предста- вим данное произведение в виде следующих двух слагаемых: 777 = 7 • 37 + 7 • 37 • 2 = 259 + 518. Наибольшим делителем чисел 259 и 518 является 259. В других случаях наибольший общий делитель слагаемых меньше. Ответ: 259. 3. Обозначив угол ВАО через х, выразим другие углы (рис. 44): Z.ABO = 180° - 125° - х = 55° - х; А А = 2х, ZB = 110° - 2х. Тогда ZC= 180°-(ZA + Z.B) = 180° - 110° = 70°. Ответ: Z.ACB = 70°.
122 Математические олимпиадные работы. 5-11 классы 4. В первой бочке после уменьшения на 10 % стало 90 %, а после увеличения на 10 % — 99 %. Во второй бочке после увеличения на 10 % стало 110 %, а после уменьшения на 10 % — тоже 99 %. Таким образом, в обеих бочках воды осталось поровну 5. Да, возможный вариант решения приведен в таблице: 1-я кучка 1 6 7 12 ... 132 2-я кучка 2 5 8 11 131 3-я кучка 3 4 9 10 130 Вариант 3 a b 1 а2 + Ь2 а2 -Ь2 Ь + а _ а3 ~ аЬ2 + a2b + b3 + а3 + b2a - cCh - b3 _ 2сг3 2. Так как AD = BE -= CF, а А АВС равносторонний, то DB = ЕС = FA. Поэтому AADF= A BED = A CFE (по двум сторонам и углу между ними), следовательно, FD = DF = EF, поэтому ADEFkk- ляется равносторонним (рис. 45). Ответ: Треугольник является равносторонним. 3. Обозначим собственную скорость корабля за х км/сут, а ско- рость движения воды — за у км/сут. Так как из Л в В корабль доплывает за сутки, а обратно — за трое, то из Л в В корабль
Решения, указания, ответы 123 плывет по течению, скорость его равна (х + у) км/сут. А так как он плывет одни сутки, то АВ = (х + у) км. Аналогично рас- суждая про путь из В в А, получим, что АВ = 3 (х - у) км. При- равнивая правые части в данных равенствах, получаем, что х = 2у. Так как время плота находится как отношение АВ к у, АВ х+и Зу получаем:----=-----— = — = 3. У У У Ответ: Чтобы добраться на плоту, понадобится 3 суток. 4. Рассматривая различные степени числа 2, получаем зависи- мость: если показатель п = 4£ +1, то 2" оканчивается цифрой 2; если п = 4k + 2 — то цифрой 4; если п = 4k + 3 — то цифрой 8; если п = 4k — то цифрой 6. Так как 2011 = 502 • 4 + 3, то 22011 оканчивается цифрой 8. 5. 361° = 19° • 19, 1° = 361° - 360°. Отложив 19 раз угол 19°, по- лучаем угол 1° как разность между 361° и полным углом (360°). 30 2 6. Представим — как 4 + —. Так как х, y,z— натуральные числа, и у + — > 1, то х = 4 и z 12^ 17 --- = —. Тогда у + — = —, откуда у = 3, У + ~ 2 z=2. Итак, х = 4, у = 3, z = 2. Рис. 45
124 Математические олимпиадные работы. 5-11 классы Вариант 4 1. Возведем обе части уравнения |х + 2| = 2 • (3 - х) в квадрат. По- сле упрощения получим уравнение: Зх2 - 28х + 32 = 0, корнями 4 которого являются числа 8 и — , но х = 8 не удовлетворяет ис- ходному уравнению. 4 Ответ: х = — . 3 2. Применим перебор. Начнем с буквы Я. Я не может равняться 0, 1 или 2. (Так как С < 9, а О9 = О, I9 = 1, 29 < 10 000, то цифры 0, 1, 2 для буквы Я отпадают). Рассмотрим случай Я = 3. Найдем такую степень тройки, что- бы получилось пятизначное число. Получаем З9 = 19 683, что не удовлетворяет условию. Рассмотрим случай Я = 4. Аналогично получаем 47 = 16 384 и 48 = 65 536. Оба варианта не подходят. Проверим случай Я = 5. Получаем 57 = 78 125. Получается. Проверяя аналогично случаи: Я = 6, 7, 8,9, получаем числа 66 = = 46 556, 75 = 16 807, 85 = 32 768, 95 = 59 049, которые не под- ходят по условию задачи. Ответ:Я = 5, С = 7, Е = 8, М = 1, Ь = 2. 57 = 78 125. 3. Пусть х дней — время, необходимое первому рабочему, чтобы выполнить работу, а у дней — время, необходимое второму ра- бочему Всю работу примем за 1. Тогда производительности < , 1 1 W первого и второго рабочих будут: — и —. Учитывая условие х у задачи, получим систему уравнений: [7Л=1, < X у х — у — Решая данную систему, получаем: х = 14, у = 10. 4. Отложив 5 раз угол 34°, получим угол 170°. Так как разность развернутого угла и угла в 170° равна 10°, то разделим угол в 10° на 2 равных угла и получим угол 5°.
Решения, указания, ответы 125 5. Обозначим стороны AD = Ча и CD = 2b (рис. 46). Тогда ВМ = a, ND = b. Применим теорему Пифагора к треугольникам АВМ и AND\ AM2 = 4ft2 + a2, AN2 = 4я2 + ft2. Пусть AN = 2АМ, тогда AN2 = 4ЛЛ/2, откуда получаем, что ft = О, чего не может быть. Значит, предположение неверно. Рис. 46 6. Уравнение х2 = у2 + 2у + 13 преобразуем к уравнению (х - у - - 1 ) (х + у + 1) = 12. Так как 12 = 1 • 2 • 2 • 3, то решения послед- него уравнения мы будем искать как решения соответствую- щих систем уравнений (достаточно рассмотреть лишь произ- ведение двух четных целых чисел): х - у -1 = 2, х + у + 1 = 6; х - у -1 = -~2, х + у + 1 - -6; х - у -1 = 6, х + у + 1 = 2; х - у - 1 = -6, х + у + 1 = -2. Решениями данных систем уравнений, а, значит, и соответ- ствующего уравнения, являются пары: (4; 1), (-4; -3), (4; -3), (-4; 1). Ответ: (4; 1), (-4; -3), (4; -3), (-4; 1).
126 Математические олимпиадные работы. 5-11 классы Вариант 5 1. Находя произведение площадей трех граней коробки, получаем 900 дм6. Учитывая, что данное число представляет собой ква- драт объема коробки, получаем, что объем коробки равен 30 дм3. 2. Так как сын родился тогда, когда отцу было 25 лет, разница в их возрасте составляет 25 лет. Поэтому 65 - 25 = 40 лет — это удвоенный возраст сына, а значит, сыну 20 лет, а отцу 45. 3. а) Разобьем имеющиеся 100 фишек на 50 пар диаметрально противоположных фишек. Синие фишки могут входить самое большее в 24 такие пары. Так как всего пар 50, то найдется та- кая пара (и не одна), которая будет состоять только из красных фишек. б) Занумеруем все фишки по порядку от 1 до 100. Фишки с но- мерами 1, 26, 51 и 76 делят окружность на 4 равные дуги, поэто- му являются вершинами вписанного в окружность квадрата. Аналогично фишки 1 + п, 26 + п, 51 + п, 76 + п, где п == 1 ... 24, образуют другие 24 квадрата. Таким образом, 100 фишек раз- биваются на 25 четверок, которые образуют 25 квадратов. Так как синих фишек всего 24, то найдется квадрат, у которого все вершины красные. ПРИМЕЧАНИЕ Задачу можно также решить, применив принцип Дирихле. 4. Применим метод от противного. Пусть оба уравнения не имеют корней. Это возможно, если их дискриминанты отрицательны, то есть: 4 - 4 • 20 ♦ 10 • АВ < 0 и 1 + 4 • < 0. Это возможно, АВ если в первом неравенстве АВ > 0, а во второ,м АВ < 0. Причем оба эти условия должны выполняться одновременно. Так как это невозможно, то хотя бы одно из уравнений имеет корень. 5. Проведем в прямоугольнике ABCD биссектрисы AAV BBV СС{ и DDV Пусть К, L, М, Н — точки их попарных пересечений в по- рядке, показанном на рис. 47. В AAHD углы при стороне AD по построению равны 45° каждый. Значит, угол DHA, а с ним и вертикальный ему угол КНМ — прямые. Аналогично доказываем, что углы HKL, KLM и LMH прямые. Таким образом, мы доказали, что KLMH — пря-
Решения, указания, ответы 127 моугольник. Пусть Р — середина отрезка AD, a Q — середина отрезка ВС. Прямая PQ является общим серединным перпен- дикуляром этих отрезков. Так как треугольник AHD равнобе- дренный, точка Я равноудалена от концов отрезка AD, поэтому лежит на прямой PQ. Аналогично, точка L равноудалена от кон- цов отрезка ВС, поэтому она лежит на прямой PQ. Рассуждая так, можно показать, что точки К и М лежат на прямой, деля- щей пополам отрезки АВ и CD. Но эта прямая перпендику- лярна прямой PQ. Значит, диагонали прямоугольника KLMH перпендикулярны, следовательно, KLMH является квадратом. Вариант б 1. Две цифры, стоящие до и после восьмерки, являются нулями. Так как произведение 8 и двузначного числа — число двузнач- ное, то делитель может быть равен 10, 11 или 12. Но 10 и 11 от- падают, так как в последнем действии произведение делителя на цифру дает трехзначное число. Поэтому делитель равен 12. Тогда последняя цифра в частном равна 9. Аналогично, первая цифра частного тоже равна 9. Таким образом, получается: _ 1089708112___ 108 190809“ _97 96 _108 108 0
128 Математические олимпиадные работы. 5-11 классы 2. Предварительно заметим, что 1,2012 + 0,7988 = 2, а 4 • 1,2012 = = 4, 8048 и4- 0,7988 = 3,1952. Обозначим х = 1,2012, у = 0,7988 и учтем, что х + у = 2. Тогда получим: 1,20124 + 0,7988' - 1,20 1 23-0,79 882 - 1,20 1 22-0,79 883 + + 4,8048 • 3,1952 = х' + г/4 - х3-г/2 - х2 • у3 + 16х • у = х4 + г/4 - - 2х2 • у2 + 16х • у = (х2 - у2)2 + 16хг/ = (х - у)2-(х + у)2 4- 16 ху = = 4(х - у)2 + 1 бхг/ = 4(х + у)2 =16. 3. Возведя исходное уравнение в квадрат, получим уравнение: 1 + л/2 + Vx = 4. Упростив его и возведя еще раз в квадрат, получим: 2 + л/х = 9. Еще раз упростив и возведя обе части уравнения в квадрат, по- лучим х = 49. Ответ: х = 49. 4. Задача решается в несколько этапов. 1) Находим середину отрезка АЕ - О и проводим прямую OF. 2) Откладываем от точки О влево отрезок ОК = ~ OF. 3) Проводим прямую ОК. 4) Через точки Е, F, А проводим прямые EL, FP, AN, параллель- ные прямым OF, АК, OF соответственно. Точки пересече- Рис. 48
Решения, указания, ответы 129 ния получившихся прямых обозначим соответственно В, С, D. Полученная фигура АВ CD является параллелограммом (рис. 48). 5. Обозначим каждого человека точками А, В, С, D, N, М, Н, R, Q, S. Соединим две точки линией со стрелкой, идущей, например, из А в В, если человеку А нравится человек В. Всего таких стре- лок будет 10*5=50. Л если посчитаем число линий, соединяю- щих 10 точек, то их будет 10 • (10 - 9) : 2 = 45. Таким образом, па 45 линиях образовалось 50 стрелок, а поэтому найдутся двое человек, которые нравятся друг другу. Вариант 7 1. Преобразуем левую часть равенства, домножив числитель и знаменатель каждой дроби на число, сопряженное знамена- 1 1 1 телю. В итоге получаем: -~т=--и-7=--+...+ —==-----т=~ V2 + 1 V3 + V2 V100 + V99 2. Приведя дроби к общему знаменателю и разложив числитель на множители, получаем: х ( х~ ( х% 2х + Зх~ + х^ х(х + 1)(х + 2) з + Т+T ” ё “ ё Так как в числителе стоят три последовательных целых чис- ла, то одно из них обязательно делится на 2 и еще одно на 3. Тогда произведение этих три чисел будет делиться на 6, то есть —।----1-- 3 2 6 является целым числом при любом целОхМ х. 3. Обозначив число ящиков с яблоками, грушами, сливами и вишнями соответственно х, г/, z, t, получаем в соответствии с условием задачи три уравнения с четырьмя неизвестными. Упрощая систему уравнений, получаем: Зу +1 - 50, < 7z = 5£, 6х = 5 г/.
130 в Математические олимпиадные работы. 5-11 классы Учитывая, что х, у, z, I являются целыми, а также то, что z и х делятся на 5, получаем, что z = х = 10, у 12, t = 14. В результате получаем, что в магазин привезли 10 ящиков яблок, 12 ящиков груш, 10 ящиков слив и 14 ящиков вишен. 4. Выполним дополнительное построение: проведем прямую ВК, параллельную прямой AD, и продолжим отрезок DC за точ- ку С (рис. 49). Рис. 49 В результате получаем два равных треугольника АВН и СВК (они прямоугольные, АВ = СВ, Z ABH = Z СВК = 90° - Z НВС), поэтому ВК = ВН 1 дм. Значит, площадь четырехугольника ABCD равна площади квадрата HBKD со стороной 1 дм, то есть 1 ДМ2. 5. Заметим, что сумма коэффициентов в левой части уравнения равна 232, поэтому х = 1 — это решение уравнения. Если 0 < х < 1, то Z < 1 для каждого натурального k, поэтому такие значения неизвестной не могут быть решениями уравнения, аналогично не являются решениями значения неизвестной, большие 1. Кро- ме того, если х < 0, то левая часть уравнения меньше 0. Итак, уравнение имеет единственное решение: 1. Ответ: х 1. Вариант 8 1. Скорость сближения волка и зайца составляет 50 м в минуту. 3 3 Значит, волку, чтобы догнать зайца, надо минуты. Но за — мин заяц пробежит 330 м, то есть чуть-чуть не успеет в укрытие. Ответ: Догонит.
Решения, указания, ответы 131 2. Пусть аиЬ — нужные числа, то есть а + b == 2, а2 + Ь2 = 3, тогда: 1 1 1 ab = j ((а + Ь)2 - (а2 + Ь2)) - - (22 - 3) = -, = (а + by - 1 ~ 3ab(a + b) = (a + b)(a2 + b2 - ab) == 2(3 - — ) - 5. 3. Из условия задачи вытекает, что AN = МС, АР = CQ, ZA = = Z С = 60°. Следовательно, треугольники NAP и MCQ равны по двум сторонам и углу между ними, и в них ZANP = Z QMC и ZAPN = Z CQM. А так как во всяком треугольнике сум- ма углов равна 180°, то ZNOQ- ZMOP- 180° - (ZQMC+ + ZAPN)- 180°-(180°- ZC- ZMQC+ ZCQM) = 60°. 4. Квадрат всякого целого числа при делении на 4 дает в остатке либо 0, либо 1. Тогда разность квадратов двух целых чисел мо- жет давать при делении на 4 в остатке 0, 1 или 3. Число 2010 = = 4 • 502 + 2 при делении па 4 дает остаток 2. Следовательно, оно не может быть разностью квадратов целых чисел. 5. Обозначим за х число «трудных» задач, за у — число «легких» задач. Тогда z — это число задач, которые решили два мальчика. Так как число всех задач равно 100, то имеем первое уравнение: х + у + z = 100. Так как «трудные» задачи решил всего 1 маль- чик, «легкие» — 3 мальчика, а оставшиеся задачи — 2 мальчика, а также, учитывая, что все 3 мальчика вместе решили 180 за- дач, получаем второе уравнение: х + 2z + Зу = 180. Из этих двух уравнений получаем: 2у + z = 80. А вычитая из первого уравне- ния полученное уравнение, имеем х - у = 20, что и требовалось доказать. Вариант 9 j (V2 + I)2 -(72-З)2 ->/32-7з-2л/2 +л/11 + 6>/2 = = 2 + 2л/2+ 1- 2 + 6V2- 9-4>/2-7(72 -1)2 + 7(3 +72 )2 = = 4х/2 - 8 -(72 -1) + (3 + 72) = 472 - 4. 2. Условие задачи эквивалентно равенству abc = 18(я + b + с), 100(7 + 106 + с = 18я + 18b + 18с, 82« = 8b + 17с, причем циф- ра с должна быть четной, поэтому не больше 8, и 8Ь + 17с —
132 Математические олимпиадные работы. 5-11 классы 3. не больше 208, то есть а < 3. Если а = 1, то 82 = 8Ь + 17с, , 82-17с 2-с к к b --------= Ю - 2с ч---, следовательно, с = 2, b = 6, аос = 8 8 = 162. Аналогично, при а = 2 находим, что b = 12, но b — цифра, поэтому единственное решение — 162. Ответ: 162. Используя свойство угла 30° в прямоугольных треугольниках ABC, BDK, АСК, признаки равенства треугольников и свойства 1 серединного перпендикуляра к отрезку, имеем: СК = DK = — ВК, откуда DK = — ВС (рис. 50). 3 Рис. 50 4. Найдем разность левой и правой частей неравенства: d2h + b2c + + с2а - b2a - а2с - c2h == ab(a с(а2 - Ь2) + с2(а - Ь) = (а- Ь)х х (ab - ас - Ьс + с2) = (а - b)(b - с)(а - с). Так как (а - b)(b - с)(а - с) > 0, то а2Ь + Ь2с + с2а > Ь2а + а2с + + с2Ь. 5. Обозначим число землян х, а инопланетян — у, а посколь- ку всего конечностей было 53, получаем уравнение: 4х + у -f- 1 + 7 у = 53. Выражая из негох, получаем х = 13 - 2у ч- --. Тог- 4 да у + 1 делится на 4. Учитывая, что у < 53:7, то есть у < 8,
Решения, указания, ответы 133 получаем у = 7 (х = 1) или у = 3 (х = 8). А так как землян было несколько, то х = 8. Ответ: х = 8. Вариант 10 1. Так как а + b = 1, то: а3 ± b3 - ab = (а + b)3 ~ ЗаЬ(а + /?) - ab = = 1 - 4 ab = 1 - 4я(1 - а) = 4«2 - 4я + 1 = (2а - I)2. 2 1 Так как (2а -1) > 0, то наименьшее значение будет при а = —. Поэтому 6 = ~. Ответ: Значение многочлена а3 + b3 - ab наименьшее, если , 1 а = b = —. 2 2. Выполним следующие преобразования: З76 = 81 • (З3)24, 550 = = 25 • (52)24. Так как 81 > 25 и 27 > 25, то З76 > 550. 3. Отобразим дом рыбака симметрично относительно берегов (лучей АВ и АС) и соединим точки Я, D, Я (рис. 51). Так как кратчайший путь между двумя точками — прямая (NH), a MD = МН, KD = NK, то кратчайший путь — это DMKD. Рис. 51
134 Математические олимпиадные работы. 5-11 классы 4. Перенесем число -3 в правую часть уравнения и вынесем за скобки х, тогда получим х(х2 + х +- 1) = 3. Так как х — это целое число и делитель числа 3, то проверяем возможные значения для х\ -1, 1, -3, 3. Из всех данных значений корнем уравнения является лишь х = 1. Ответ: х = 1. 5. Поступаем следующим образом. Пронумеруем все четыре мо- неты. На одну чашу весов кладем монеты 1 и 2, на другую -- мо- нету 3 и гирю. Тогда возможны 3 варианта. 1) Весы находятся в равновесии. Это означает, что фальшивая монета — 4, и вторым взвешиванием можно определить, тя- желее или легче она настоящей, сравнив ее с гирей. 2) Весы не находятся в равновесии, причем тяжелее чаша с ги- рей. Это означает, что фальшивая монета находится на ве- сах, а монета 4 не фальшивая. При этом, если это монета 3, то опа тяжелее настоящей, а если это монета 1 или 2, то она легче настоящей. При втором взвешивании на одну чашу весов кладем монеты 1 и 3, на другую — монету 4 и гирю. Тогда возможны еще 3 варианта: во-первых, весы находятся в равновесии, а это означает, что монеты 1 и 3 настоящие, а монета 2 фальшивая и она легче настоящей монеты; во-вторых, перевесила чаша с монетами 1 и 3, а это озна- чает, что монета 3 фальшивая и она тяжелее настоящей монеты; в-третьих, перевесила чаша с монетой 4 и гирей, а это означает, что монета 1 фальшивая и она легче настоящей монеты. 3) Весы не в равновесии, при этом легче чаша с гирей. Это означает, что фальшивая монета на весах, а монета 4 не фальшивая. При этом, если это монета 3, то она легче на- стоящей, а если это монета 1 или 2, то она тяжелее настоя- щей. При втором взвешивании на одну чашу весов кладем монеты 1 и 3, на другую — монету 4 и гирю. Тогда снова возможны 3 варианта: во-первых, весы находятся в равновесии, а это означает, что монеты 1 и 3 настоящие, а монета 2 фальшивая и она тяжелее настоящей монеты;
Решения, указания, ответы 135 во-вторых, перевесила чаша с монетами 1 и 3 а это озна- чает, что монета 1 фальшивая и она тяжелее настоящей монеты; в-третьих, перевесила чаша с монетой 4 и гирей, а это означает, что монета 3 фальшивая и она легче настоящей монеты. 9 класс Вариант 1 1. Обозначим хл объем воды в озере, ул — объем воды, поступаю- шей в озеро из родников за сутки, z л — объем воды, выпивае- мой одним слоном за 1 день. Тогда в соответствии с условием задачи получаем следующую систему уравнений: х + у ~ 183z, х + 5у = 5 • 37z. Вычитая из второго уравнения системы первое, получаем 4 г/ = 2z, откуда имеем z = 2у. Подставляя в первое уравнение z = 2 г/, получаем х = 365г/. Пусть 1 слон выпьет всю воду из озера за t дней, тогда получаем уравнение х + t • у = t • z. Учитывая, что z = 2г/, получаем, что х = у • t. А из этого уравнения получаем: х 365?/ t = — =--— = 365 (дней). То есть слон осушит озеро за 365 дней. У У Ответ: Слон осушит озеро за 365 дней. 2. Используя теорему Пифагора, получаем равенство а2 + Ь2 = с2. Умножая обе части равенства на с, получим: а2с + Ь2с = с3. Так как с > а и с > Ь, то а2с > а3 и Ь2с > Ь\ Тогда а2с + Ь2с > а3 + Ь\ Но а2с + Ь2с = с3, значит, с3 > а3 + Zz3. 3. Так как корни квадратного уравнения х2 + рх + q = О равны р и q, то по теореме Виета имеем: Р + Я - РЯ = <7- Тогда решениями данной системы являются: 1) р = 7 = 0; 2) р- 1,7 =-2. Ответ: рх = 7* = 0;р? = 1. 72 = -2.
136 Математические олимпиадные работы. 5-11 классы 4. Умножаем числитель и знаменатель второй дроби на х, а тре- тьей — на ат/. Тогда получаем: 1 X XI/ -----_-----1--------------------------. 1 4- х 4- ху х + ху 4- xyz ху 4- xyz + xyzx Учитывая, что xyz = 1, имеем: 1 х ху --------4-------------+-------Z-------- 1 + X + ху X 4- XI/ 4- xyz ху 4- xyz 4- xyzx 1 X XI/ 1 4- х 4- XI/ =---------4----------4----------=---------- = 1 . 1 4- X 4- Ху X 4- Ху 4- 1 XI/ 4- 1 4- X I + X + Ху 5. Сумма всех записанных на доске чисел нечетная (сумма 1006 нечетных чисел и 1005 четных чисел является числом нечетным). При стирании двух чисел возможны гри вари- анта: 1) стираются два четных числа, тогда модуль разности получа- ется четным числом, а новая сумма — нечетным; 2) стираются два нечетных числа, тогда модуль разности по- лучается четным числом, а новая сумма — нечетным; 3) стираются одно четное и одно нечетное числа, тогда модуль разности получается нечетным числом и новая сумма тоже. Таким образом, после каждого стирания сумма оставшихся на доске чисел оказывается нечетной, а число чисел уменьшается на 1. После 2010 стираний на доске останется единственное не- четное число. Так как нуль — число четное, то оставшееся чис- ло нулем быть не может. Вариант 2 1. а* + // > а3Ь 4- ab3 => а' + b* - a3b - ah3 = (а* ~ а3Ь) 4- (bf' - ab3) = = а3(а - Ь) 4- Ь\Ь - л) = {а - Ь)(а^ - Ь3) (а - Ь)2 (а2 + ab 4- Ь2) > 0. 2. Примем стоимость 1 кг ягод в начале сезона за х руб., а 1 кг сахара -- за у руб. Пусть для приготовления варенья понадо- бится и кт ягод и v кг сахара. Тогда в соответствии с условием задачи для приготовления варенья по одному и тому же рецеп- ту будет затрачено в начале сезона хи 4- yv руб., а в разгар сезо- на — 0,85хи 4- 1Jyv руб. Так как приготовление варенья в разгар сезона дешевле на 10 %, получаем: 0,85.rw 4- \>\yv = 0,9xw 4- 0,9z/tj.
Решения, указания, ответы 137 Отсюда хи = ^yv. В результате отношение стоимости ягод к сто- имости варенья в начале сезона будет составлять: —- • 100% = • 100% = 80%. хи + yv 5yv Ответ: 80 %. 3. Обозначаем 2009 = а, отсюда получаем: л/2009-2010-2011-2012+Т = = у[а(а + 1)(а + 2)(« + 3) + 1 = у]у2 + 2г/ + 1 = | у + 1|, где у а2 + За. Окончательно имеем: V2009 -2010 •’20ГГ2012 + 1 = (20 092 + 60 27 + 1) = (20 00 + 9)2 + + 6028 = 4 000 000 + 36 000 + 81 + 6028 = 4 042 109. 4. Треугольники ЛЛ17)] и DD^C^ являются подобными, поэтому соответствующие стороны этих треугольников пропорцио- нальны, то есть: DDt DC{ у > Обозначая ADX = у, C{D = z, находим DD{ = 4 - //, СС} = 3 - z = АА t (рис. 52). Подставляя вместо ЯЛ р DDVADV соответственно 27 9 3 - z, 4 - г/, у, z из равенств (*), находим у = -— , z = —. Тогда ЗдЛоб Рис. 52
138 » Математические олимпиадные работы. 5-11 классы Ответ: Стороны вписанного прямоугольника равны зЛоё и------см. 8 5. Так как по условию: (100.x + 10г/ + z) + (100х + 10z + у) + + (100.// + 10x + z) + (100// + 10z + х) + (100z + 10г/+х) + (100z + + 10х + у) = ЗООх + ЗОх + Зх. то после упрощения имеем х = = 2(//4-z), то есть х —четная цифра. Так как х> 1,г/> l,z> 1, г/ tz, то х > 6. Значит, х = 6 или х = 8. Учитывая это, а также то, что 1 у + z~ —х, перебором находим данные цифры: 1) 6,1,2; 2) 6,2, 1; 3) 8, 1,3; 4) 8,3, 1. Вариант 3 1 Так как область определения данного уравнения является пу- стым множеством, то данное уравнение не имеет корней. 2. Число 210 + 512 является составным, так как справедливы сле- дующие преобразования: 2t0 + 512 = (25)2 + (56)2 = (25)2 + (56)2 -+ 2 • 25 • 56 - 2 • 25- 56 = (25 + 56)2 - 26 • 56 = (25 + 55 - 23- 53)(25 + 56 + 23 • 53). 3. Рассмотрим значение трехчлена в точке х0 = 2: у{} = 4 + 2р + q = = 4 + 2| р + ^] =4024. Г 2 J Получается, что графики всех функций вида у = х2 + рх + q про- ходят через точку (2; 4024). 4. Задача имеет множество решений. Рассмотрим одно из них (рис. 53). Проведем в трапеции AKCD среднюю линию ML. Она будет параллельна AD и КС, причем AL == 3 см. Обозна- чим AD 2х, тогда КС х. Так как треугольник ALM равнобе- дренный с углом при вершине 60°, то он еще и равносторонний, поэтому LM = 3 см. А тогда, используя свойство средней линии 2х 4- X трапеции, имеем: ----- = 3, откуда х 2, а значит, AD = 4 см.
Решения, указания, ответы 139 5. Пусть а — наименьшее количество задач, придуманных одним студентом. Выбрав по одному студенту с каждого курса и учи- тывая, что все они придумали разное число задач, найдем мини- мальное число придуманных ими совместно задач: 5/7 + 10. Так как всего задач придумано 40, то на 25 студентов осталось не больше 30 - 5а задач. Так как хотя бы по одной задаче студенты придумали, то 30 - 5а > 25, откуда а < 1. А так как наименьшее число задач, придуманных одним студентом, ненулевое (а > 0), то получаем, что а = 1. То есть 25 студентов придумали по 1 за- даче, а еще 1 задачу придумал студент из 5 отобранных по одно- му с курса. Таким образом, по 1 задаче придумали 26 студентов. Вариант 4 1. 5 + 52 + 53 + ... + 52010 = 5(1 + 5) + 53( 1 + 5) + ...+ 52009(1 ±5) = = 6(5 + 53 + ...+ 52009). Так как первый множитель делится на 6, то и все произведение делится на 6. 2. Искомая площадь равна площади четырех сегментов АВ и квад- рата ABCD. Так как Z.MOB = 60° (треугольник МВО и ANO — равносторонние со стороной 1 см), то Z NOB = 30°. Тогда цен- тральный угол АОВ сегмента АВ будет равен 30°. Поэтому . ГО-2 -30 1 2 опо ПГ2 >'2 площадь сегмента АВ равна-----------г sin 30 =---------, 360 2 12 4
140 Математические олимпиадные работы. 5-11 классы а учетверенная площадь сегмента при г « 1 составляет — -1. Площадь же квадрата равна 2 - >/3, поэтому площадь пересече- ния четырех кругов составляет -^ + 1 - >/3 (см2) (рис. 54). N М О Рис. 54 3. Преобразуем уравнение к виду: (у2- I)2 + 2(г/- х2)2 = 0. Это уравнение равносильно следующей системе уравнений: р2-1 = о, у - х2 - 0. Данная система уравнений имеет два решения: (1; 1)и(-1; 1). Ответ: (1; 1); (-1: 1). 4. Так как велосипедист, проехав треть пути, остановился и ждал, пока мотоциклист проедет две трети пути, то треть пути велоси- педист проехал быстрее, чем мотоциклист две трети. Поэтому велосипедист и весь путь 5 проедет быстрее, чем мотоциклист 2s. Значит, велосипедист приедет в пункт В раньше. 5. Начнем рассуждения с высказывания восьмого человека: «Здесь не больше семи честных людей». Если восьмой человек
Решения, указания, ответы 141 говорит правду, то все хорошо. Если же он лжет, то это означа- ет, что в комнате 8 честных людей, а это противоречит тому, что восьмой человек лжет. Значит, восьмой не может лгать, то есть он говорит правду. Первый человек сказал, что в комна- те нет честных людей. Но мы выяснили, что восьмой — чест- ный человек, значит, первый солгал, то есть он лжец. Изучая высказывание седьмого человека, выясняем, что он не может быть лжецом, иначе в комнате должно было быть 7 или 8 чест- ных людей. Но первый — лжец, поэтому седьмой должен быть честным. Рассуждая далее аналогично, получаем, что второй, третий и четвертый лгут, а шестой и пятый говорят правду. В результате мы выяснили, что в комнате 4 честных человека. Вариант 5 1. Обозначим искомые числа х, у и z, при этом z = г3 - у3. Тогда: Z = (Х - Z/)(x2 + ху+ у2). Чтобы число z было простым, необходимо, чтобы х - у = 1, а это при условии простоты чисел X и у возможно только при X = 3, у = 2. Отсюда z — 19. Ответ: Искомые числа —- это 2, 3 и 19. 2. Рассмотрим рис. 55. Рис. 55
142 Математические олимпиадные работы. 5-11 классы На данном рисунке OS — расстояние от центра описанной окружности О до стороны ВС, а РА — расстояние от точки пере- сечения высот треугольника до вершины А. Выполним допол- нительное построение: проведем диаметр CW. Тогда отрезок OS будет средней линией треугольника BCN, а значит, равняться половине BN. NBA. ВС (так как угол NBC прямой), АКА.ВС, значит, прямые NB и АК параллельны. Аналогично, прямые NA и ВР параллельны. Таким образом, ANBP — параллелограмм, поэтому АР = NB = 205. 3. Сложим все уравнения системы и выделим полные квадраты разностей: (х - 1 )2 + (у - 1 )2 + (z - 1 )2 = 0. Решение данного уравнения, а значит, и исходной системы: х = у = z ~-= 1. 4. Пусть дано квадратное уравнение ax2 + hx + с = 0, где а, Ь, с — целые числа. Выясним, может ли выполняться равенство Ь2 - - 4ас = 2010. Число Ь2 делится на 4 без остатка (при четном Ь) или дает в остатке 1 (при нечетном Ь). Так как 4<тс делится на 4, то весь дискриминант либо делится на 4, либо дает в остатке 1. Л 2010 при делении на 4 дает остаток 2. Следовательно, равен- ство Ь2 - 4ас = 2010 невозможно ни при каких целых а, Ь, с. 5. Перемножая все 50 произведений, мы получим 1, так как в каж- дое произведение любое из чисел войдет 2 раза. Тогда в 50 со- множителей войдет четное число произведений с «-1» , а поэ- тому сумма четного числа произведений с «1» и четного числа произведений с «-1» не равна 0 (25 — число нечетное, значит, одинакового числа слагаемых не будет). Вариант б 1. Преобразуем левую часть уравнения: х4 - 2х4 + Зх2 - 4х + 5 = = (х4 - 2х3 + х2) + (2х2 - 4х + 2 + 3) = (х2 - х)2 + 2(х - I)2 + 3. Полученное значение не меньше 3, поэтому х4 - 2Х3 + Зх2 - 4х + + 5 > 0. Следовательно, уравнение х2 - 2Х3 + Зх2 - 4х + 5 = О корней не имеет. 2. Преобразуем выражение: (3/4 + 3/2 +1)(3/4 -1)(ЗД - 3/2 +1) = = (3/4 + 3^2 +1)(3/2 - 1)(3/4 - 3^2 + 1)(3/2 + 1) =
Решения, указания, ответы 143 = ((W -1)((^2)3 + 1) = (2 -1)(2 + 1) = 3. 3. Так как ах1 2 + 2Ьх + с < 0 при всех х, то по свойствам квадратич- ной функции: а < О, 462 - 4<7с < 0. Из этой системы следует, что с < 0. Трехчлен а2х2 + 2Ъ2х + с2 принимает при всех х положительные значения, если выпол- няются условия: а2 > 0, 4й4 - 4я2с2 < 0. Первое неравенство очевидно. Преобразуем левую часть вто- рого неравенства: 4Z?4 - 4aV = 4(/»2 - ас)(Ь2 + ас). Так как Ь2 ~ ас < 0, Ь2 + ас > 0, то все произведение отрицатель- но. Таким образом, значения а2х2 + 2Ь2х + с2 при всех х положи- тельны. 4. Выполним дополнительное построение (рис. 56): проведем прямую СК, параллельную диагонали BD. Рассмотрим тре- угольник АСК. В нем СН — высота, АС 4. Рассмотрим два слу- чая: Z.AOD = 120° и 'АОВ = 120°. Рис. 56 1) AAOD = 120°. Так как СН - 2, аАС = 4, то Z САН = 30°, поэтому ЛАСН 60°, значит, СН — биссектриса тре- угольника АСК. А так как по условию СН — высота, то
144 Математические олимпиадные работы. 5-11 классы треугольник АСК является равнобедренным, а значит, АС = КС, то есть BD = 4. 2) Z.AOB = 120°. Тогда Z.AOD = 60°, а значит, СК совпа- дет при этом же построении с СН. Тогда вторая диагональ BD = CH-2. Ответ: 4 или 2. 5. Обозначим данные числа av а2, ... а9011. Учитывая условие, имеем: а. = —4 + б//+'1 для всех / = 2, 3, ... 2010 и а = 1 2 1 2 ..., б?2(И1 #9010 + #1 тт ---L. Из данных равенств следует: - а2011 = ~ а2 ~ а\ ~ аз ~ ^2 ~ *” ~ ^2011 ~ ^2010 ( )’ Обозначим данную разность d, Если бы было а > 0, то из ра- венств (*) следовало бы, что а2011 < < а2 < ... < а201() < а?ои, чего быть не может. Аналогично, рассматривая случай d > 0, по- лучим противоречие. Таким образом, остается, что d = 0, а это означает, что = а2 = а3 = ... = я2(Ш. Вариант 7 1. Найдем остатки от деления на 7 чисел 2010, 2011, 2012 и 2013: 2010 = 287 -7 + 1; 2011 = 287-7 + 2; 2012 = 287 • 7 + 3; 2013 = = 287-7 + 4. Произведем действия с остатками от деления каждого из этих чисел на 7: 1 • 2 • 3 + 46. Учтем, что 4G = (7 • 9 ь I)2 имеет оста- ток 1 при делении на 7, поэтому сумма всех остатков равна 6 + 1 = 7 • 1 + 0, и число 2010 • 2011 • 2012 + 20136 будет делиться на 7. 2. Подставим z = ху из первого уравнения во второе, тогда полу- чим: ху2 = х. Из данного уравнения следует, что х = 0 или у2 = 1. Рассматривая случай х = 0, получаем, что у = z = 0. Рассматривая случай у = 1, получаем следующую систему: J х = z, [Л2 = 1. Решение данной системы: х = z = ±1. Рассматривая аналогично у = -1, найдем соответствующие ре- шения другой системы: х = -z = ±1. Таким образом, решением
Решения, указания, ответы 145 исходной системы являются следующие тройки чисел: (0; 0; 0), (1; 1; !),(-!; 1;-1), (-1;-1; 1), (1;-1;-1). Ответ: (0; 0; 0), (1; 1; 1), (-1; 1;-1), (-1;-1; 1), (1;-1;-1). 3. Используя теорему Виета, имеем: Txj + Л‘2 “ ~С2, | Jtj • Xq — Cl. Сложив эти два уравнения, получим уравнение + x2 + x,-x2 = 0, которое преобразуем к уравнению ^(1 + х>) + (1 + х2) == 1. Вы- нося общий множитель за скобки, получаем: (1 + х2)-( 1 + xt) = 1. Так как корни уравнения целые, то имеем: J 1 + ~ 1, J 1 + Xj = —1, [1 + == [Д *^9 = “В Решения данных систем уравнений: = х2 =- 0 и х} = х2 = -2. Находя соответствующие значения для а, получаем а = 0 или а = 4. 4. Переложим части данного четырехугольника (рис. 57) так, что- бы их вершины Л, В, С, D оказались в одной точке и совпали отрезки АК и ВК. ВР и СР т. д. При этом получается четырех- угольник, у которого противоположные стороны попарно рав- ны (две стороны равны PN, а две — КМ). Рис. 57
146 Математические олимпиадные работы. 5-11 классы 5. Разделим квадрат на квадратики размером — х —. Так как ква- 5 5 дратиков — 25, а точек — 51, то найдется квадратик, в который попадут как минимум 3 точки. Диагональ этого квадратика >/2 2 равна — , что меньше диаметра круга —, значит, круг радиусом 5 7 1 у накроет тот квадрат, в котором находится не менее 3 точек. Варианте 1. Умножим первую скобку на последнюю, а вторую на третью. В итоге получим уравнение: (6х2 + 5х - 4)(6х2 + 5х - 6) = 35. Пусть у = 6х2 + 5х - 5. Тогда получим уравнение у2 - 1 = 35, корнями которого является у 6 или у = -6. Соответственно 5 корнями уравнения 6х2 + 5х -5 = 6 являются числа 1 и -1--, 6 1 1 а корнями уравнения 6х2 + 5х - 5 = - 6 — числа — и — . 2 3 Ответ: 1, -1 —. — , — . 6 2 3 Рис 58 2. Пусть у данного треугольника АВС величина угла В на 120° больше величины угла А, равной а. Пусть М — точка пере-
Решения, указания, ответы йи 147 сечения биссектрисы угла С со стороной АВ, а точка пересе- чения высоты, проведенной из С, с продолжением стороны АВ — N (рис. 58). Величина угла С равна 60° -2а, тогда вели- чина угла АСМ равна 30° - а, а величина угла ВМС составля- ла 1 ет 30°, поэтому--= sin 30° = —. МС 2 3. Пусть всего сделано х горизонтальных разрезов и у верти- кальных. Длина стороны квадрата равна «. Сумма периметров получившихся прямоугольников равна (2х + 2)а + (2у + 2)а. По условию (2х + 2)а + (2у + 2)а = 400я. откуда х + у = 198. Общее число получившихся прямоугольников равно: (х + 1) {у + 1) =--= (х + 1)(199 - х) = 199 + 198х - х2. Рассмотрим квадратичную функцию отх: у = ~х2 + 198х + 199, она достигает максимума при х = -198 -2 1 - 99. Тогда число пря- моугольников получается равным 10 000. 4. Ответы Бориса и Григория противоречивы. Значит, один из них сказал правду, а остальные солгали. Поскольку Александр сол- гал, то Борис не крал. Поскольку Виктор солгал, то украл он. Ответ: Кражу совершил Виктор. 5. Раскроем скобки. Произвольное слагаемое в полученной сум- 1-2...(и~1) . . 9 „ ме имеет вид:---------, где г = 1,2,1. Сгруппируем сла- i 1-2...(?/-1) гаемые в этой сумме: каждое слагаемое---------------—- сложим со i 1-2...(и-1) слагаемым------------, тогда сумма этих слагаемых будет рав- п - i на: 1 2 •... • (н -1) • | 1 + —- L 1 2 ... • (п -1) • — . i n-i j i(n - i) Данное число является целым, так как п нечетно и ? Ф п - г. Таким образом, получаем сумму целых чисел, каждое из которых делится на п.
148 Математические олимпиадные работы. 5-11 классы 6. Опишем стратегию первого игрока. Первым ходом он дол- жен взять со стола 87 монет. Если второй игрок берет х мо- нет, то каждым следующим ходом первый игрок должен взять (101 - х) монет — он всегда может это сделать, потому что если х — четное число от 2 до 100, то (101 - х) — нечетное чис- ло от 1 до 99. Так как 2007 = 101 • 19 + 87 + 1, то через 19 та- ких «ответов» после хода первого на столе останется 1 монета, и второй игрок не сможет сделать ход, то есть проиграет. Вариант 9 1. Рассмотрим возможный вариант решения (рис. 59). Рис. 59 Для этого выполним дополнительное построение: продолжим прямую ВС до пересечения с прямой EF в точке К. Рассмотрим четырехугольник EDCK ZDCK = 180° - 135° = 45°, Z СКЕ = = 360°-45°- 130° - 125° = 60°= ХАВС.Тж круглы ZCKE и Z АВС являются накрест лежащими при прямых АВ и ЕЕи се- кущей ВК, то прямые АВ и ЕЕ параллельны. 2. Так как сумма 2012 чисел — число нечетное, то число нечетных слагаемых нечетно. Тогда среди 2012 чисел есть хотя бы одно четное число, а значит, произведение 2012 чисел является чет- ным числом. 3. Проведем через точку М прямую параллельно стороне АВ, обо- значим ее пересечение с СЕ через Е(рис. 60). Треугольники АВМ и MFC равны по стороне и прилежащим к ней углам, поэтому АВ = MF. Кроме того, отрезок MD параллелен ЕЕ, a MF паралле-
Решения, указания, ответы 149 лен DE, поэтому четырехугольник MDEF— это параллелограмм, и MF= DE. Так как АВ = MF (из равенства треугольников АВМ и MFC) и MF = DE, то АВ = DE. Так как отрезки АВ и DE па- раллельны, то ABED — это параллелограмм, а значит, BE = AD. Рис. 60 4. Пусть всего в лесу растет п грибов, тогда из них — плохие. По- 4 скольку в каждом грибе может жить не более 4 толстых чер- вей, то в каждом плохом грибе живет не менее 12-4 = 8 тощих червей, а значит, во всех плохих грибах проживают не менее п 8 х — = 2?? тощих червей. Толстых же червей во всех грибах 4 не более 4/2, то есть их численность превосходит численность тощих не более чем вдвое. Значит, тощие черви составляют не менее трети от общего числа червей в лесу. 5. Разобьем доску на 16 квадратиков размером 2 х 2. В каждом из них может стоять не более одного короля, иначе они будут бить друг друга, поэтому их число при любой правильной расста-
150 и Математические олимпиадные работы. 5-11 классы новке не превосходит 16. Например, если в левом нижнем углу каждого квадрата разбиения стоит по одному королю. Мини- мальное же число королей равно 9. Оно не может быть меньше. Отметим на доске 9 полей: Ь2, Ь5, Ь8, е2, е5, е8, h2, h5, h8, в ко- торых разместим 9 королей. Поставленный на доску десятый король будет бить одного из них либо стоять на одном из них. Поэтому чтобы все эти клетки находились иод боем или были заняты, нужно не менее 9 королей. Ответ: Минимальное и максимальное число королей равно соответственно 9 и 16. Вариант 10 1. Введем прямоугольную систему координат так, чтобы начало координат совпало с вершиной Л, а вершины В и D лежали на осях координат (рис. 61). В с м У ------ А х а х Рис. 61 Примем координаты точки М за х и у, а длину стороны квад- рата — за а. Тогда в соответствии с условием задачи получим следующие три равенства: x2 + z/2 = 49(l), X2 + (у - а)2 = 289 (2), (х-л)2 + (г/-л)2 == 529 (3). Вычитая из второго равенства первое, получим а2 - 2ау = 240. Вычитая из третьего равенства второе, имеем: а2 - 2ах = 240.
Решения, указания, ответы в 151 Следовательно, х = у, поэтому точка М лежит на диагонали АС. Тогда АС = AM + МС « 30 (см), поэтому площадь квадрата рав- на 450 см2. 2. Да, можно, например: 3, -2, -2, 3, -2, -2, 3. Друюй вариант: при п натуральном и большем 2 эти числа будут п, -п - 1,0, л?, —п - 1, 0, п. 3. Так как А(А + В + С) < 0, то А Ф 0. Так как /(х) = Ах2 + Вх + С, то/(1) = А + В + С. Рассмотрим два случая: 1) А < 0 => А + В + С > 0, значит,/(1) > 0. Тогда ветви параболы направлены вниз, а вершина параболы находится выше оси абсцисс. То есть парабола пересекает ось абсцисс в двух точ- ках. Поэтому соответствующее уравнение Ах2 + Вх + С = 0 имеет два действительных корня. 2) А > 0 А +- В + С< 0, значит/(1) < 0. Тогда ветви параболы направлены вверх, а вершина параболы находится ниже оси абсцисс. То есть парабола пересекает ось абсцисс в 2 точках. Поэтому соответствующее уравнение Лх2 -t Вх -• С = 0 имеет два действительных корня. 4. Пусть в трапеции ABCD, где AD и ВС — основания, длина диа- гонали BD равна сумме длин AD и ВС (рис. 62). Отметим на BD точку О так, что ВО = ВС, тогда по условию OD = AD и треугольники ВСО и ADO являются равнобедрен- ными. Углы ADO и СВО при их вершинах равны, поэтому углы AOD и СОВ при основаниях также равны, следователь- но, отрезки АО и ОС лежат на одной прямой и образуют диа- гональ АС трапеции. Угол ВОС острый как угол при основании
152 Математические олимпиадные работы. 5-11 классы равнобедренного треугольника, следовательно, он равен углу между диагоналями трапеции и по условию равен 60°. Анало- гично, угол AOD равен 60°. Отсюда следует, что в треугольни- ках ВСО и ADO все углы равны 60°, и эти треугольники равно- сторонние. Следовательно, треугольники АОВ и DOC равны по двум сторонам и углу между ними, откуда следует, что трапе- ция равнобокая. 5. Имеем уравнение: 100я + 10/; + с = 12(cz + Ь + с), которое пре- образуется к уравнению: 11 (8а - с) = 2b, где а, Ь, с — цифры. Поэтому = 1, Ь = 0, с 8, а искомое число равно 108. 6. Победит Баба Яга с помощью следующей стратегии. Первым ходом она берет из первой кучки 1997 грибов, а затем каждым своим ходом уравнивает число грибов в кучках, имеющееся к ее ходу Таким образом, выиграет Баба Яга. 10 класс Вариант 1 1. Обозначим эти числа А и В. Пусть В — наименьшее из этих чи- сел. Тогда, учитывая условие, большее число /1 10В + с, где с — зачеркнутая цифра, 0 < с < 9. По условию задачи также имеем А + В = 11В с' = 2011. Это означает, что с является остат- ком от деления 2011 на 11, то есть с = 9, В = 182, А = 1829. Таким образом, это числа 1829 и 182. 2. Обозначим f(x) ах2 + Ьх + с. Заметим, что /(1) а + Ь + с, /(0) с. Так как уравнение ах2 + Ьх 4- с = 0 не имеет корней, то график функции /(л) = ах2 + Ьх + с не пересекает ось абсцисс. А так как а + Ь + с < 0, то /(1) < 0. Поэтому график функции f(x) = ах2 + Ьх -t- с расположен ниже оси абсцисс. А так как /(0) = с, то с < 0. 3. Предположим противное, тогда любые 3 журнала на столе вместе закрывают менее ~ его площади. Всего можно выбрать десять различных троек журналов, таких что сумма площадей закрываемых ими частей стола будет менее чем 6 площадей
Решения, указания, ответы 153 стола. В то же время, каждая точка стола покрыта хотя бы од- ним журналом и каждый журнал входит в шесть из этих троек, поэтому каждая точка стола покрыта этими тройками не менее чем 6 раз. Значит, сумма покрываемых ими частей стола не ме- нее чем шесть его площадей. Получили противоречие. 4. Если обозначить 36 камней за «зайцев», а 7 трехтонок за «кро- ликов», то по принципу Дирихле на одну из машин придется погрузить не менее 6 камней. Однако масса даже самых легких 6 камней будет 490 + 495 + ... + 515 = 3015 > 3000. Значит, увез- ти 36 камней на 7 трехтонках не удастся. 5. Четность числа бананов при любом срывании двух плодов с де- рева не меняется, поэтому, если число бананов было четным, то оставшийся плод — ананас, а если нечетным — банан. Вариант 2 1. Пусть четырехугольник имеет стороны, длины которых со- ответственно равны а, Ъ. с, J. Тогда ab ~ be = cd = da = 1. Из равенства ah = be следует, что а с; а из равенства be = cd полу- чим, что h = d. Следовательно, данный четырехугольник — па- раллелограмм. Пусть ос — угол между сторонами а и Ь. Тогда 5 = ab • sin а и, следовательно, площадь максимальна, если sin а == 1. В этом случае S = ab = 1. 2. Так как 0 < sin — < 1, 0 < cos— < 1, го etg - = cos—: sin— > cos — 5 5 5 5 5 5 , 6.6 6.6 6 71 6 Л и tg — = sm — : cos — > sm —. Поскольку — > —, то cig — < etg — = 5 5 5 5 5 3 5 3 Таким образом, порядок чисел следующий: 6 6.66 cos —,etg—, sm— ,tg—. 5 6 5 5 6 5 3. 2x2 -ь y1 = 2xy + 4x <=> x2 - 4x4 4 t- yl - 2xy + x2 =- 4 (x - 2)2 + + (x - y)2 = 4. Если сумма двух квадратов целых чисел равна 4, то один из квадратов равен 4, а другой 0. Если х - 2 = 0, то х -= 2
154 Математические олимпиадные работы. 5-11 классы и (2 - г/)2 = 4 « 2 - у = ± 2, откуда^ = 0 или у Таким обра- зом, получаем два первых ответа: (2; 0), (2; 4). Если (х - 2)2 = 4, то х - 2 = ± 2 и х - у = 0, что дает ответы: (0; 0), (4; 4). Таким образом, ответОхМ являются следующие 4 пары целых чисел: (2,0), (2,4), (0,0), (4,4). Ответ: (2; 0), (2; 4), (0; 0), (4; 4). 4. Первое из уравнений означает, что последовательность чисел х, г/, z, и, s, t является геометрической прогрессией. Из второго уравнения следует, что отношение ее первого члена к четверто- му равно 8. Значит, знаменатель прогрессии равен . В третьем уравнении указана сумма 6 членов прогрессии. Так как сумма Г rk с XI-76) 6 членов прогрессии находится по формуле: = —-----— , то 15— = 4 2 2 Т 2 . Из данного уравнения находИхМ х 8. Учи- тывая, что х, г/, г, и, s, t образуют геометрическую прогрессию 1 1 1 с q = , а х -= 8, находим у = 4, z 2, и 1,5 = —, t = — . Ответ: х = 8, у 4, z 2, и 1,5 = —, t . 2 4 5. Функцию, удовлетворяющую условию задачи и дающую ровно два корня для уравнения /(х) 1, можно задать, например, ра- венствами /(1) =/(2) = 1,/(х) = 2 прих * 1; 2. Теперь покажем, что у нашего уравнения всегда не меньше двух корней. Один из них — единица, ибо: /(I) = /UVU/(I).-.))) = /GVC-/« •))) -1 • 2009 2009 Для нахождения второго корня возьмем любое число yQ Ф 1 и положим у{ = f(y^, У2 /Gt) и т. д. Из условия следует, что не позже 2009-го шага в последовательности yv у2... появится член, равный 1. Пусть ут — такой член. Тогда число ут t являет- ся корнем уравнения /(х) = 1, не равным 1. Значит, уравнение /(х) = 1 имеет не менее 2 корней.
Решения, указания, ответы 155 Вариант 3 п + 1 1. 22011 Обозначим 22011 = п, тогда первое выражение-—-----=-------, 22012 +1 2п +1 2. 22012 + 1 2п + 1 „ а второе — 7^3----= ~----- • 1ак как разность первого и второ- го выражений положительна, то первое число больше второго. я 5 ч- 3 з п ч- 3 , п Так как -7------п -пл—--------, то п' + 3 делится нацело на п + 1 п + 1 о , , п + 3 п2 + 1, если —5-- целое. Соответственно при дробном —5---- тг + 1 п ч- 1 величина п3 + 3 делиться нацело на п2 + 1 не будет. Решая не- и5ч-3 п2 +1 равенство при п < -1, п 1 и учитывая, что 0 , найдем, что 3. »5 + 3 п2 + 1 п -Г 3 2, п Ф -3 выражение —3- является дробным. п +1 Проверяя остальные целые числа, получаем, что при п = -3; -1; л л + 3 0; 1; 2 “5-является целым, а значит, при этих целых чис- гГ +1 лах п величина п3 + 3 делится нацело на п2 + 1. Ответ: п = -3; - Г, 0; 1; 2. 7X11- 7 1 Запишем уравнение х“ ч— -ч—у = 5 в виде х" ч—- + 2 2х xz х“ 1 ( Ого ч--~ х-= 5 . Заметим, что 2х J х J 1 2д. 1 чим у = х-, тогда получаем у1 + —у х 2 1 f - х~ + - 2. Обозна- 3 -3 = 0, откуда у = - п ,л. 13 или у = -2. Таким образом, решая уравнения х-------= — и х - х 2 1 9 9 1 — = -2, получаем xt = 2, хг = — Ответ: 2; — 2 -1 + V2;-l - V2.
156 Математические олимпиадные работы. 5-11 классы 4. Допустим, что в треугольнике АВС проведены биссектри- са AL, медиана ВМ и высота СН, причем AL и СИ пересекаются в точке D, ВМ и СН — в точке £, а точки D и Е делят высоту СН на три равные части (рис. 63). Перпендикуляр MN, опу- щенный из точки М на прямую АВ, равен половине высоты СН. Значит, отрезок ЕН, который меньше, чем MN, равен трети высоты. Тогда отрезок DH должен составлять две трети высо- ты. Опустим из точки D перпендикуляр £>£на сторону АС. По свойству биссектрисы угла DF = DH. Но это невозможно, ибо в таком случае катет DF прямоугольного треугольника CDF оказывается вдвое длиннее его гипотенузы CD, составляющей треть высоты СН. Рис. 63 5. Обозначим всю траву на лугу за 1, а прирост травы за 1 день — за а. Тогда через 24 дня на лугу будет 1 + 24а травы, через 60 дней — 1 + 60а, через 96 дней — 1 + 96а. Так как 70 коров съе- ли всю траву за 24 дня, а 30 коров — за 60 дней, то количество „ , м 4 1 + 24а съеденной травы 1 коровой за 1 день определяется как у- 1 + 60а ТЛ 1 + 24а 1 + 60а 1 или------- Из уравнения-------=-------- находим а =----, 30-60 7 24-70 30-60 480 тогда через 96 дней на лугу будет 1 + 96а =1,2 (травы).
Решения, указания, ответы 157 За 1 день корова съедает ---, а за 96 дней — 0,06. Тогда 1600 1 ? 1,2 травы за 96 дней съедят —— = 20 (коров). 0,06 Вариант 4 1. Используем теорему Виета: х\ + х22 = (х1 + х2)2 - 2х}х2 = (а-2)2- - 2(-я - 3) = (я - I)2 + 9. Тогда значение выражения (а - I)2 + 9 будет наименьшим при (7=1. 2. Обозначим длины катетов прямоугольного треугольника и гипотенузы соответственно х, у, г, По теореме Пифагора х2 + + у2 = z2, где х, у, z — натуральные числа. Пусть х и у не делятся на 3. Тогда х2 и у2 имеют остаток от деления на 3, равный 1. Значит, их сумма х2 + у2 имеет остаток от деления на 3, рав- ный 2, и не может быть полным квадратом (так как квадрат натурального числа либо делится на 3, либо имеет остаток от деления на 3, равный 1). Поэтому среди катетов найдется та- кой, длина которого делится на 3. 3. Так как Z МВС = Z МКС = 60°, то через точки М, К, В, С можно провести окружность. Тогда Z.KBC = ZKMC = 60° (как впи- санные). Поэтому Л ВАС + ЛАВК = 60° + (60° + 60°) = 180°, а значит, ВК и АС параллельны (рис. 64). Рис. 64
158 » Математические олимпиадные работы. 5-11 классы 4. Обозначим данные положительные числа av а2, av ..., «201Г По условию для всех i== 2,3,4,... 2010 а 4 ’ а2 > ^оп = л/^201Г • И3 Данных равенств следует: а\ „ а2 = а3 = „ ^2011 ^2011 а1 а2 6Z20i0 Обозначим это отношение за q. Оно положительно. Пусть q < 1, тогда из условия (*) получается, что &2()11 > > а2 > ...> > <7?0)0 > чего быть не может. Аналогично доказывается, что при q > 1 тоже получается противоречие. Значит, остается 7=1, и тогда условие (*) записывается так: а, = = ...= бю..п = а.т„. J \z 12 ZU1U 2011 5. Рассмотрим все возможные 10 линий, по которым возмож- но разрезание: справа налево и сверху вниз. Предварительно докажем, что каждая из этих линий пересекает четное число костяшек домино. Допустим, что это не так и какая-то линия пересекает нечетное число костяшек. Разрсже^м доску по та- кой линии. При этом доска так разделится на две части, что в каждой будет содержаться четное число клеток. Но если линия пересекает нечетное число костяшек, то число клеток, занимаемых костяшками в каждом из этих полей, нечетно. То есть мы получили противоречие. Таким образом, каждая линия пересекает четное число костя- шек домино. Допустим теперь, что при каком-то размещении костяшек каждая из 10 линий пересекает хотя бы одну ко- стяшку. Тогда каждая из этих 10 линий пересекает хотя бы 2 костяшки, причем ясно, что 2 линии не пересекают одну ко- стяшку. Следовательно, мы получаем, что 10 линий пересека- ют по меньшей мере 20 костяшек домино, тогда как их число равно 18. Полученное противоречие и доказывает, что найдет- ся такой способ разрезания доски на 2 части, при котором ко- стяшки остаются неповрежденными. Вариант 5 1. Обозначим за х первоначальное число учащихся в лицее, тогда мальчиков из них было 0,5х. После указанных событий уча- щихся стало 0,9х, а мальчиков •— 0,9х • 0,55 = 0,495х. Так как 0,495х < 0,5х, то мальчиков стало меньше.
Решения, указания, ответы и 159 2 Вычитая из первой дроби вторую, получаем дробь с(а - Ь) b(b + с) ’ в которой выражения с, b и(Ь + с) отрицательны, а выражение (а - h) положительно. Значит, получившаяся дробь меньше нуля, поэтому второе число больше первого. 3. 2009-й член последовательности равен наименьшему натураль- ному числу п, для которого 1 + 2 + ... + п = > 2009 . По- следнее неравенство равносильно неравенству п1 + п ~ 4018 > 0. Решением данного квадратного неравенства (с учетом того, . -1 + V16073 „„ оп что п - натуральное число) является п >----------~ 62,89. 2 4. Пусть МС пересекается с AD в точке Е (рис. 65). Тогда АЕ = СЕ, как отрезки касательных, проведенных к окружности из одной точки. Выполним дополнительное построение: проведем
360 Я Математические олимпиадные работы. 5-11 классы АВ и ОС. Так как ОВ = ОС, то Z ОВС = Z ОСВ. Так как пря- мые ОС и СЕ перпендикулярны, то ^ECD = Z BCM = 90° - - Z ОСВ = 90° - Z ОВС. Рассмотрим треугольник ABD: Z. BDA = 90° - Z ОВС, значит, Z.BDA = Z.ECD, а значит, тре - угольник ECD равнобедренный, поэтому СЕ = DE. Но так как АЕ = СЕ,го АЕ = DE. 5. Обозначим НОД (х, z/) = d, отсюда х = du, у = dv. Тогда 1111, + + — + —. = 1 откуда получаем уравнение и + v + 1 + du dv duv d ' ‘ + uv = duv, которое преобразуем к уравнению (1 + v)(u + 1) = = duv. Пусть и < v. Так как (1 + v}(u + 1) duv и и и и + 1, v и v + 1 являются взаимно простыми, то и + 1 делится нацело на v. Но и + 1 < v + 1, значит, и - v будет равно 0 или -1. В первом слу- чае v = 1, во втором v = 2 или 3. Тогда соответствующими ре- шениями уравнения для данного случая являются следующие пары (х, г/): (4; 4), (3; 6), (4; 6). Рассматривая случай и > v, аналогично найдем еще два реше- ния: (6; 3), (6; 4). Ответ: (4; 4), (3; 6), (6; 3), (4; 6), (6; 4). Вариант 6 1. После выделения квадратов разностей в подкоренных вы- ражениях исходное уравнение преобразуется к уравнению: |л/х - 1 - 2| + jVx -1 - з| = 1. Обозначив Vx^d = t, найдем ре- шениЕ уравнения: |z - 2| + - 3| - 1, которое будет 2 < t < 3. Подставляя у[х~\ вмесзо t, получаем решение уравнения: 5 < х < 10. Ответ: 5 < х < 10. 2. Так как 25, 43, 70 — члены арифметической прогрессии, то 25 = -г kd', 43 = + nd', 70 = + md. Из данных трех равенств следует, что 18 = (п - k)d, 27 = (т - n)d. Из по- следних двух равенств получаем 9 = (т - 2п + k)d. Так как 2014 = 70 + 1944, а 1944 -216-9- 216(m - 2п + k)d, то 2014 - = 70 + 216(т - 2/7 + k)d = а{+ kd + 216(т - 2п 4 k)d = +
Решения, указания, ответы 161 + (216т - 432л/ + 217k)d или 2014 = + Id, где / > 0, то есть 2014 — член этой же прогрессии. 3. Обозначим соответствующие уравнения, задающие изобра- женные графики функций ух = ах2 + Ьх + с, у2 = Ьх2 + сх + а, у3 = сх2 + ах: + h (других вариантов в соответствии с изображен- ными графиками быть не может). Так как ветви двух из трех парабол направлены вверх, то две из трех букв а, Ь, с положи- тельны. В то же время, при х = 0 два из трех значений функций отрицательны. А так как значения функций при х = 0 будут совпадать с а, Ь, с, получаем противоречие. Значит, данные па- раболы графиками функций быть не могут. 4. Если справа от лжеца А сидит лжец Б, то справа от Б также сидит лжец, и т. д., то есть все сидящие за столом — лжецы, чего по условию быть не может (среди сидящих есть рыца- ри). Если справа от лжеца А сидит рыцарь Б, то справа от Б — рыцарь В, справа от В — лжец Г и далее сидят 2 рыцаря, за- тем лжец и снова 2 рыцаря. Тогда всего за столом получается 6 рыцарей и 3 лжеца. Ответ: За столом сидят 3 лжеца. 5. Рассмотрим план построения (рис. 66): 1) Соединяем точки Ап В отрезком. 2) Проводим прямую АР, перпендикулярную прямой а, и от- кладываем на луче АР любой отрезок АО. 3) Проводим окружность с центром в точке О и радиусом ЗАО. 4) Точку пересечения данной окружности и отрезка АВ обо- значаем за М. 5) Проводим прямую ВС, параллельную прямой МО. В тре- угольнике ВСА ВС = ЗАС. 6) Откладываем на луче АС СО2 = АС. 7) Проводим прямую BN, параллельную АР, и прямую О.,К, па- раллельную прямой СВ, точку их пересечения обозначаем Ov 8) Проводим две окружности: с центром в точке О1 и радиусом ВО, ис центром в точке О, и радиусом О.^А 9) Данные две окружности и являются искомыми.
162 и Математические олимпиадные работы. 5-11 классы Рис. 66 Вариант 7 1. Обозначим углы треугольника ос. р, у. Так как сумма углов треугольника равна 180°, то ос ч- р = 180° у, откуда sin( ос ч- р ) - = sin(180°y ). Так как sin (180° - у) = sin у, то sin (ос ч- Р) = sin у. 2. Рассмотрев четыре случая: а) х > 0, у > 0; б) х > 0, у < 0; в) х < 0, у > 0; I) х < 0, у < 0, получим график уравнения, пред- ставленный на рис. 67. Рис. 67
Решения, указания, ответы 163 3. Найдем значение многочлена в точке х = 1: f(\) = A + B+ C+D + E+F= 21 - 16+ 13 -47 + 46- 17 = 0 Это означает, что х = 1 является корнем любого из таких мно- гочленов, а значит, все такие многочлены имеют общий корень. 4. Обозначим стороны и углы исходного четырехугольника: AB-a,BC~h,CD-c,AD = d, Z.A= a, ZB = Р, ZC = у, ZD = = 8 (рис. 68). ~ad sin а + — be sin у = — ab sin P + — cd sin 8. 2 2 2 2 у ^AliCD $ $ A[BlA + +5C j£>, + AtD j + $ ЛИСП S*n а + 5. Все жители острова не могут быть лжецами, ибо тогда эти лжецы сказали бы правду. Значит, на острове есть рыцарь. Из его заявления вытекает, что лжецов на острове больше, чем (2009 - 1) : 2 = 1004. Теперь возьмем любого лжеца. Его заяв- ление ложно, то есть, не считая его, не более половины остро- витян — лжецы. Это значит, что если убрать одного лжеца, то оставшихся будет не больше чем 1004, то есть всего лжецов —
164 Математические олимпиадные работы. 5-11 классы не больше 1005. Единственное целое число, которое больше, чем 1004, но не больше, чем 1005, — это 1005. Значит, лжецов на острове 1005. 8 — sin2 х 6. Так как 4 + х2 > 4, а--— < 4, то уравнение будет иметь ре- 2 + х4 шение только в случае, если и левая, и правая части уравнения равны 4, а это возможно лишь тогда, когда х = 0. Варианте Пусть дана арифметическая прогрессия, содержащая четное число членов: а2, а3,... а,2п. Найдем суммы членов первой поло- с (°1+а„) с (ЯП+!+а2„) вины и второй половины: э, =--------п,$2 =------------п . Учитывая, что а = а + dn ~ d, а = а + dn, а., = а, + 2dn - d, 2^ + 3dn - d получИхМ S2 = —4— ------п. Тогда S2 - 51 = -^-(2^ + 3dn ~d - 2а{- dn + d) = d • n1. 2. Один из возможных вариантов: 20092009...2009 (2009 раз под- ряд). 3. Обозначим через w кг количество травы, нарастающее на лугу за одни сутки, через и кг — количество травы, съедаемой за день одной коровой, и через V кг — количество травы на лугу к мо- менту выгона коров. По условию V + 8 а; = 8 • 20г/, V + 6а; = 6 • 26г/, откуда w = 2и, 144г/. Поэтому за счет постоянного прироста травы на лугу могут пастись 2 коровы, а за счет первоначально имевшейся травы — еще одна (лето — 92 дня, что меньше 144). Ответ: 3 коровы. 4. Треугольники ACD и ВСЕ равны (первый из них переходит во второй при повороте на 60° против часовой стрелки). Следо- вательно, отрезки СМ и СР, как медианы этих треугольников, равны и угол между ними составляет 60°. Значит, треугольник СМР равносторонний. 5. Пусть сделано х горизонтальных и у вертикальных разрезов прямоугольника. Обозначим длину горизонтальной стороны прямоугольника а. Тогда длина вертикальной стороны бу-
Решения, указания, ответы 165 дет 2а. Сумма периметров получившихся прямоугольников равна (2х + 2)а + (2у + 2)2а. По условию 2(х + 1)а + 4(г/ + 1)а = = 101(2/2 + 4а), откуда х + 2у = 300. Всего прямоугольников (х+ 1)(г/+ 1) = (г/ + 1)(301 ~2у) = 301 + 299г/- 2г/2. Так как квад- 299 ратичная функция от у достигает максимума при у =-— = -4 = 49,75, то при у < 49,75 рассматриваемая функция возрастает, а при у > 49,75 — убывает. Тогда из симметрии графика этой функции относительно прямой у = 49,75 и того, что 50 ближе к 49,75, следует, что максимум для целых у достигается при у = 50 и равен 10 251. 6. Занумеруем столбцы и строки числами от 0 до 7 и на пересече- нии г-той строки и j-того столбца впишем в клетку остаток от деления i + j на 3 (рис. 69). 1 2 0 1 2 0 1 2 2 1 2 0 1 2 0 1 1 2 1 2 0 1 2 0 0 1 2 1 2 0 1 2 2 0 1 2 1 2 0 1 1 2 0 1 2 1 2 0 0 1 2 0 1 2 1 2 2 0 1 2 0 1 2 1 0 1 2 3 4 5 6 7 Рис. 69 Заметим, что «дельфин» всегда делает ход с 0 на 1, или с 1 на 2, или с 2 на 0. Предположим, что «дельфин» сделал требуемый обход. Тогда он последовательно побывал на клетках с числами (рис. 70). Следовательно, он побывал на 22 клетках, в которых записан 0. Но таких клеток всего 21, значит, обход невозможен. 0->1->2->0->1->... ->2->0 63 клетки Рис. 70
166 Математические олимпиадные работы. 5-11 классы Вариант 9 1. Так как sin2x + cos2x = 1, получаем уравнение: (sinx)2012 + (cosx)2012 = sin2x + cos2x. Отсюда имеем sin2x(l - sin201(,x) + cos2x(l - cos2010x) = 0. Так как |sinx| < 1, |cosx|<l, то равенство выполняется только в том случае, когда оба слагаемые равны 0. Поэтому уравнение sin2x х х (1 — sin20l0x) + cos2x (1 - cos20,()x) = 0 равносильно следующей системе уравнений: sin2 х(1 - sin2010 х) = 0, cos2 х(1 - cos2010 х) = 0. Решением данной системы является х = пп ~2 где п g Z. Ответ: х = пп —, где п g Z. 2. Пусть в верхнем левом углу таблицы стоит число х, тогда . 4 справа от него стоит 6 - х, снизу от него — число —, а в цен- х тре — 6----« -----. Продолжая заполнение таблицы далее, х 6 - х получим, что каждый элемент таблицы однозначно выража- ется через данные значения х, и для выполнения полученно- го условия достаточно положить х равным любому из корней хх 9 = 3 ± V5 полученного уравнения. Таким образом, мы полу- чаем следующие два варианта заполнения таблицы: 3 + V5 3 — 75 34- J5 [ю 1 00 3 + ’ J5 3 — У5 3+V5 3 — 75 34- У5 3-V5 3 + 75 3-75 з + Vs 3-75 3 + 75 3-75 3 + 75 3-75 3. Пусть первая из команд забила за весь матч т голов, вторая — п голов. Сумма числа голов в ходе матча изменяется с ша- гом 1 от 0 до т + п, значит, в какой-то момент она будет равна т.
Решения, указания, ответы м 167 Данный момент и является искомым в задаче, потому что при этом число голов, уже забитых второй командой, равно разно- сти т и числа голов, уже забитой первой командой, то есть чис- лу голов, которое еще предстоит забить первой команде. Ана- логично можно рассуждать и в отношении первой команды. 4. Углы AKII и ALII прямые, поэтому четырехугольник AKHL является вписанным в некоторую окружность, следователь- но, углы КАН и KLH равны, как опирающиеся на общую дугу КН (рис. 71). Значит, ZKLC- 90° + Z КАН- 90° + 90° - - ZABC = 180° - Z АВС, поэтому в четырехугольнике BKLC сумма противоположных углов при вершинах В и L равна ZKLC + ZKBC^ ZKLC^ ZABC- 180°, следовательно, вокруг него можно описать окружность, что и требовалось доказать. 5. Обозначим разность йрогрессии через d. Пусть п2 — квадрат натурального числа п, лежащий в данной прогрессии, тогда число п + d тоже натуральное и (zz + d)2 = п2 + d(2n -г d) — также элемент прогрессии. Продолжая так далее, мы постро- им бесконечно много квадратов натуральных чисел, лежащих в данной прогрессии. 6. Да, можно, например: 3, -2, - 2, 3, -2, -2, 3. Другой вариант: при натуральном п большем 1 эти числа будут п, -п - 1,0. zz, -п - 1, 0, п.
168 Математические олимпиадные работы. 5-11 классы Вариант 10 1. Пусть стороны треугольника равны b, bq, bq2, площадь тре- угольника — 5, тогда высоты треугольника соответственно рав- 25 25 25 ны: -—, —,-—-, то есть тоже образуют геометрическую про- b bq bq2 1 грессию со знаменателем —, Q 2. Возведем обе части уравнений в квадрат и сложим, тогда по- сле упрощений получим уравнение 2 + 2cos(a-p ) = а2 + 1, из которого получаем а2 = 1 + 2cos(ot-|3). Учитывая, что cos( ос - р ) < 1, получаем а1 < 3. Значит, а < . 3. Заметим, что 2013 = 11- 183. Так как rz11 - 1 = {а - 1 )(я10 + а9 + + ... + а+ 1), то 22013 - 1 = (2183)11 - 1 делится на 2183 - 1. Поэтому число 22013 - 1 является составным. 4. Если блондинов было п, то всего игроков -- 4/7. Они сыграли х о 4zz(4zz — 1) между собой ----—---- партии, из которых половина, то есть - 1) партий была выиграна рыжими, набравшими, сле- довательно, в сумме z?(4/7 - 1) очков. Каждый рыжий сыграл но 4/2 - 1 партии и мог набрать не более 4/7 - 1 очков, значит, каждый рыжий выиграл каждую партию, в которой участвовал. Отсюда следует, что рыжих не могло быть более одного и этот единственный рыжий обыграл всех остальных и победил в тур- нире, в котором участвовало 4 игрока. 5. Рассмотрим на прямой некоторую пару точек А и В одного цве- та. Если середина С отрезка АВ имеет тот же цвет, то отрезок АВ — искомый. Если нет, рассмотрим точки D и Е, такие что А — середина отрезка DB, а В — середина отрезка АЕ. Тогда либо один из отрезков DB или АЕ является искомым, либо цвет точек D и Е противоположен цвету А и В и совпадает с цветом точки С, являющейся также серединой DE. значит, DE — искомый. 6. Обозначим длины сторон большого и малого квадратов через 2х и 2у соответственно, радиус окружности — через R. Тогда расстояния от центра окружности до вершин вписанных квад- ратов, лежащих на окружности, дают выражения:
Решения, указания, ответы 169 (2х-Л)2+х2 = /?2, (2г/ + /?)2 + г/2 = Я2. 4 Отсюда получаем х - у = — h. Тогда разность длин сторон квад- 5 8 , ратов равна ~п. 11 класс Вариант 1 1. sin6x + cos6x = (sin2x + cos2x)(sin4x - cos2x • sin2x + cos4x) = 3 = 1 '((sinx + cos2x)2 - 3sin2xcos2x ) = 1-sin22x. 4 3 3 3 Так как sin22x < 1, то-sin22x > — , а значит, 1-sin22x > 4 4 4 > 0,25. Поэтому и sin6x + cos6x > 0,25. 2. 200 800 • 1003 + 2008 • 100 800 = 2008 • 100 • 1003 + 2008 • 1008 • 100 = = 2008 • 100 • (1003 + 1008) = 2008 • 100 • 2011. Данное число делится нацело на 2011. 3. Произведение среднего арифметического и среднего гармо- нического чисел а и Ъ равно ah. Таким образом, произведение чисел, записанных на доске, остается неизменным (независимо от того, стирались ли числа в выходные), и 22 октября произве- дение чисел будет таким же, каким было 1 сентября, то есть 42. 4. Проведем через точку,М прямую параллельно стороне АВ, обо- значим ее пересечение с СЕ через F(pnc. 72). Треугольники АВМ и MFC равны по стороне и прилежащим к ней углам, поэтому АВ = MF. Кроме того, отрезок MD па- раллелен FE, a MF параллелен DE, поэтому четырехугольник MDEF — это параллелограмм, и MF = DE. Так как АВ = MF (из равенства треугольников АВМ и MFC) и MF= DE, то АВ = DE. Тогда ADEB является параллелограммом, а значит, AD = BE, что и требовалось доказать.
170 Математические олимпиадные работы. 5-11 классы Рис. 72 5. Обозначим число конфет, булочек, стаканов чая и участников олимпиады соответственно k, h, с, у. Тогда, учитывая условие задачи, получаем следующую систему уравнений: J k - b + с, [с - у + k - у = Ь. Подставив k из первого уравнения системы во второе и упро- стив второе уравнение, получаем, что с = у, то есть число участ- ников олимпиады равно числу стаканов чая. А гак как каждый участник олимпиады выпил по стакану чая, то для жюри стака- нов чая не осталось. 6. Выразим х через у: И у + \ у + 1 Так как х — целое, то у + 1 — делитель 11, то есть у может быть 0; -2; 10 или -12. Находя соответствующие значения х, полу- чим следующие решения уравнения: (-6; 0), (16; -2), (4; 10), (6; -12).' Ответ: (-6; 0), (16; -2), (4; 10), (6; -12). Вариант 2 1. 11рименяя формулу для синуса двойного угла 2 раза, преобра- зуем уравнение к виду:
Решения, указания, ответы 171 — sin 4.x • cos8x = ~sin!2x. 4 4 Это уравнение после применения формулы sin а • cos (J и упро- щения преобразуется к уравнению sinl2x + sin4x = 0, решением которого является х = ke Z. 2. Числа в каждой строке образуют арифметическую прогрессию с разностью 1. Рассмотрим п-ю строку: и п + 1 п + 2 ... Зя ~ 2. Чисел в строке будет 2п - 1. Найдем их сумму: п + Зп — 2 ---------(2я -1) = (2я - 1) . 3. Обозначим массу первого слитка за х кг, а за у кг — массу вто- рого слитка. Тогда, учитывая условие задачи, в первом слитке х 2х будет ™ кг золота и — кг меди, а во втором, соответственно, 3 3 2у Зу кг золота и кг меди. Так как в первом сплаве получается 5 5 золота столько, сколько в первом слитке, то имеем уравнение: 1 х 5 2у __ 2х 3 3 + 6 Так как во 2 сплаве получается меди на 1 кг больше, чем золота во втором слитке, то получаем второе уравнение: 2 2х 1 Зу 2у __ . 3 3 2 5 5 Решая получившуюся систему уравнений: 1 х 5 2у _ 2х аТёТ""? 2 2а _1_ Зу_ _ 2у = 1 3 3 2 5 5 находим: х = 3,6 кг, у = 6 кг. Значит, золота в первом слитке было 1,2 кг, а во втором — 2,4 кг. Ответ: Золота в первом слитке было 1,2 кг, во втором — 2,4 кг. 4. Рассмотрим рис. 73. Обозначив MD х, DE ••= у, получим систе- му уравнений:
172 Математические олимпиадные работы. 5-11 классы Зх - Зу + ху = О, х2 + у2 = 16. Первое уравнение получаем, учитывая, что площадь треуголь- ника ВЕС равна сумме площадей треугольника MED и трапе- ции BMDC\ а второе уравнение получаем, применяя теорему Пифагора к треугольнику MED. Введем новые переменные: и ~ у - х, 3v = ху. В результате получаем систему уравнений: -Зи + 3v = О, и2 + 6^ - 16. Решением последней системы будут и == 2; v = 2. Тогда V7 - 1; у = 1 + V7. Так как NE = ND + DE = 3 - х + у, то NE = 5. 5. Если при наложении таблиц в некоторой строке совместились клетки разного цвета, то в этой строке найдется еще одна такая же пара клеток. Если бы это было не так, то количество синих клеток отличалось бы в совмещаемых строках двух таблиц на
Решения, указания, ответы 173 единицу, что невозможно, так как число синих клеток в каж- дой строке четное. Рассуждая аналогично, можно сказать, что в столбце, в котором накладываются клетки разного цвета, най- дется еще пара клеток разного цвета. Л в строке, содержащей эту пару клеток разного цвета, найдется еще разноцветная пара. Так как все эти три пары совмещаемых разноцветных клеток разные и не совпадают с исходной парой, то этим мы доказываем, что найдется еще три клетки, накрытые клетками другого цвета. Вариант 3 Обозначим радиусы нижнего соответственно г\ и г2, радиус (рис. 74). и верхнего оснований конуса шара — R, искомый угол — а Рис. 74 Тогда, используя условие задачи, получаем: 2rt + 2r2 = 5/?(*) Так как углы АВ О и ОВС равны (О — центр шара), то ZABO = = ZOBC = Тогда = 7?ctg~, r2 = 7?tg-~. Учитывая это, уравнение (*) примет вид: 2 /Mg j + 2 /?tgy = 5R. Последнее уравнение, в свою очередь, приводится к виду: а а 2tg27 - 5tg^ +2 = 0. 2
174 Математические олимпиадные работы. 5-11 классы ОС о 4 а 1 _ Из этого уравнения находим tg — = 2, или tg~ = —. 1ак как я а = 2arctg2 > ~, то искомый угол равен: а = 2arctg-^-. 2. Вычтем второе уравнение из первого, тогда получим уравне- ние: у2 - х2 = х3 - у3 + 3(у2 - х2) - 2(у ~ х). Оно равносильно следующему уравнению: {у - х)(у + х + х2 + + ху + у2 - Зу ~ Зх + 2) = 0. Последнее уравнение равносильно совокупности двух уравне- ний: у - х = 0 и у + х + х2 + ху + у2 - Зу - Зх + 2 = 0. Из первого уравнения выражаем у = х, подставляем вместо у = л в первое уравнение исходной системы и получаем уравне- ние х3 - 4х2 + 2х = 0, имеющее корни: х{ = 0, х>3 = 2 ±>/2 . Соот- ветственно, у{ = 0,у23 = 2 ±л/2. Второе уравнение преобразуем к виду: у2 + у(х - 2) + х2 - - 2х: + 2 = 0. 2 Его дискриминант: D(y) = (х - 2)2 - 4(х2 - 2х + 2) = -3(х-)2 - 3 Поэтому второе уравнение корней не имеет. Ответ: (0; 0), (2 - >/2 ; 2 - >/2 ), (2 + л/2 ; 2 + >/2 ). 3. Изобразим данную фигуру и прямую на координатной плоско- сти (рис. 75). Введем векторы АВ = (8; 6), Вс = (8; 6), AD = (3; -4). Так как АВ = DC и AD-AB = 0, то ABCD — прямоугольник, при этом прямая, заданная уравнением х + Ту - 67 = 0, проходит через точку С, так как координаты точки С удовлетворяют уравнению этой прямой. Кроме того, точка Е является серединой отрезка АВ (координаты середины отрезка АВ\ х = 4,у~9 удовлетворя- ют уравнению прямой х + Ту - 67 = 0). АВ =10, ВС = 5. Приме- няя формулы для вычисления площадей прямоугольного треу- гольника и прямоугольника, находим площади искомых фи!ур: Sabcd “ 50- Sвсе = 12-5- поэтому SAECD = 37,5.
Решения, указания, ответы 175 Ответ: Прямая разбивает данную фигуру на две фигуры, име- ющие площади 12,5 и 37, 5 кв. ед. 4. Введем обозначения: а = sinl° • sin2° * sin3° • ... • sin89°, b = sinl° • sin3° •... • sin87° • sin89°. Тогда -- = sin2° • sin4° -,0p6° • ... • sin88°. Применяя формулу с двойного угла, получаем: ~ = 244 • sinl° • sin2° • sin3° •... • sin44° • b • cos1°-cos2°-cos3° •... • cos44°. Отсюда находим: b = sin 1° • sin 3° •... • sin 87° • sin 89° = a 244 sin 1° • cos 1° • sin 2° • cos 2° •... • cos 44° • sin 44°
176 Математические олимпиадные работы. 5-11 классы _ sin 45° • sin 46° •... • sin 89° _ " 244 cos 1°-cos2°-... cos44° “ sin 45° sin 46° ... • sin 89° ~ 244 • sin 89° • sin 88° •... • sin 46° “ 2^ ’ 5. Неравенство —-— < — < —-— равносильно неравенству 2010 < 2011 n 2010 ‘ H n < — < 2011, которое, в свою очередь, равносильно неравенству т 2010m < п < 2011m. Если т = 1, то последнее неравенство не имеет смысла, так как п — натуральное число. Если т = 2, то получаем 4020 < п < 4022 и п = 4021. Так как при т > 2 ин- тервалы вида (2010m; 2011m) располагаются на числовой оси правее числа 4021, то найденное значение п = 4021 является наименьшим. Ответ: п = 4021. Вариант 4 1. Пусть эти четыре последовательных числа: п, п + 1, п + 2, п + 3. Тогда: п(п + l)(/z + 2)(п + 3) + 1 = (п2 + Злз)(т?2 + Зп + 2) + 1 = (п2 + + Злу)2 + 2 (п2 + Зп) + 1 = (п2 + Зп + I)2 2. Введем обозначения: 1 Л 1 1 о 2011 ( 2 3 2011J 1 f, 1 1 1 2012 ^ 2 3 2012 ) ,11 1 (1 — 1 + + + ... + 2 3 2011 а 1 ( 1 1 1огда х = , у = а + . 2011 2012^ 2012 J а 1 _ а 1 2012 20122 2011 2012 20122 > 0, так как а > 1. Поэтому: х — у = J 2011 _ 1 ( а 1_"| " 2012^2011 2012J
Решения,указания, ответы 177 3. Ответ: 1 ' 2012 ------1 Ч------1----F ... Ч------ 2011 2 3 2011 ------ 1ч------1---F ... + 2012 2 3 Выполним дополнительное построение: проведем диаметр DK (рис. 76). Тогда ZKBD = 90°, а четырехугольник АКВС будет трапецией (KBLBD, ACLBD => КВЦАС). Так как четырех- угольник вписанный, то АК = ВС. Рис. 76 Треугольник AKD прямоугольный, поэтому КА LAD и ОН± AD. Треугольники же OHD и KAD подобны (по двум ч КА KD углам), а значит,-=--- ОН OD 2R R 4. Преобразуем уравнение к виду: (1 + х2)(1 ч- х) = 2У, откуда 1 ч- х2 = 2т, где т — целое неотрицательное число, и 1 + г = 2я, где п — целое неотрицательное число. Так как х = 2" - 1, то х2 = = (2п - I)2 = 22п - 2 • 2" ч- 1. Учитывая, что 1 ч- х2 = 2W, получим 22" - 2 • 2п + 2 = 2т. Разделив обе части уравнения на 2, получим уравнение 22”'1 - 2п + 1 = 2W-1, которое равносильно уравнению 2"(2"~1 -1)ч-1== 2'”*1. Так как т — целое неотрицательное число, то при т > 1 2"'-1 — это четное число, поэтому 2Л(2”“1 - 1) — чис- ло нечетное. А это невозможно. Значит, осталось рассмотреть
178 Математические олимпиадные работы. 5-11 классы т = 0 и т = 1. При т 0 имеем 1 + х2 = 2°, откуда х = 0, у = О, которые не являются натуральными. Л при т = 1 получим, что х2 = 1, а учитывая, что х — число натуральное, получаем х = 1, У = 2- Ответ: (1; 2). 5. Пусть такой многогранник существует. Обозначим за kv k.„ ..., 7 k„ число ребер на гранях, тогда + k,2 + ...+ kfi = 21 — удвоенная сумма всех ребер многогранника, а она четная. В то же время, в левой части стоит нечетная сумма слагаемых, каждое из ко- торых нечетно. Получили противоречие. Значит, такого много- гранника не существует. Вариант 5 1. В качестве членов арифметической прогрессии рассмотрим к члены данной последовательности, имеющие вид: а ь =-------—, к 2012! где k = 1; 2;... 2012. Разность d данной прогрессии равна • 2. Перенесем в левую часть 2 sin4x • cos4x и прибавим и вычтем по cos8x. В результате полученное уравнение можно преобра- зовать к виду: (sin4x - costx)2 + cos2x( 1 - cos6) = 0. А это уравнение равносильно следующей системе уравнений: sin 4х - cos1 х -- 0, cos2 х(1 - cos6) - 0. Решая второе уравнение и подставляя его решения в первое, в результате получаСхМ решение исходного уравнения: х - ~ + тгЛ, k е Z. 2 Ответ: х - ~ + nk, k g Z. 2
Решения, указания, ответы 179 3. Функция г/(х) определена на интервалах (-«>;-1)и (-!;+«>), на каждом из которых она непрерывна. Вычисляя tg(y(x)) по формуле тангенса суммы, получим 1. Поэтому функция у(х) постоянна на каждом из промежутков области определения. При х = 0 у = —. При х == - у/З у = arctg (- >/3 ) + arctg (-2- л/з ) = 4 = - - arctg (2+л/З). Найдем arctg (2+>/3). Обозначим arctg (2 + >/3 ) = а, тогда tg а = 2 + V3 . Воспользуемся фор- л/З к мулой двойного угла и тогда tg 2 а =-. Тогда 2 ос = — + 3 6 Я Я пли а = - + 77 • УЧИТЬ1Вая» что а лежит в первой четвер- 5л _ я 5л Зя _ . ти, получим а = —. 1огда и =------------=-----. Ааким од- 12 * 3 12 4 разом, функция у(х) имеет вид: Зя . .ч --—при Хе (-оо;~ 1), Я у . - — при хе (-!;+«>). Значит, ее график выглядит так, как показано на рис. 77. Рис. 77 4. Выполним дополнительное построение: соединим точки А/ и К, В и О (рис. 78).
180 и Математические олимпиадные работы. 5-11 классы Рис. 78 Рассмотрим треугольники ОКС и АОМ. Они подобны. По свой- 2 1 2 ству медиан треугольника ОС = ~ МС, ОМ = — МС, ОА = — КЛ, ОК = ~КЛ. Гак как А К и СМ — медианы, то AM = ~^В, КС = = ~~ВС. Из подобия треугольников ОКС и ОМА следует, что ОК ОМ ОС КС 9 1? --= —. Так как ОС - -МС, ОМ = -МС, ОЛ = - КА, ОЛ AM 3 3 3 ОК = — КА и ОК • ОА = ОМ • ОС, то МС = КА, значит, треуголь- ник АВС равнобедренный. Треугольники АОВ и СОВ равны по трем сторонам. Из равенства треугольников следует, что ВО — биссектриса, а так как треугольник АВС равнобедренный, то и медиана. Рассматривая треугольник МКС, получаем по тео- МВ ВС реме синусов:---------------—. 1 ' sin 30° sin Z.CMB Из данного равенства имеем, что Z СМВ = 90°, а значит, Z СВ А = = 60°, и поэтому треугольник ЛВС является равносторонним.
Решения, указания, ответы 181 5. Минимальная площадь сечения получается, если точка М яв- ляется серединой DC (рис. 79). Найдем длины диагоналей ромба BMD^N: BDX = ау[3 , MN= аЛ. Тогда находим площадь сечения как площадь ромба. В результа- те получаем: Вариант б < п «2 1. Перенесем выражение —— в левую часть уравнения и упростим х2 получившееся уравнение. В результате получим: л 3 3 4 х + ах +а х-а х2(х2 -ах + а2) Это уравнение равносильно следующему: (х2 + а2 )(х2 -г ах - а2) 9 9 9 ’ х“ (х - ах + а' ) 1) х2 + а2 = 0, если а = 0. Но тогда х = 0, что не является корнем исходного уравнения, так как не входит в область опреде-
182 Математические олимпиадные работы. 5—11 классы ления уравнения. Поэтому при а = 0 исходное уравнение корней не имеет. 2) Пусть а * 0. Рассмотрим уравнение х2 + ах - а2 = 0. D = 5б/2 > 0 при а Ф 0. Тогда корнями уравнения являются 2 Ответ: При а = 0 уравнение не имеет корней, при а Ф 0 корни -а ± у[5 уравнения равны: х{2 ----~--• 2. Рассмотрим середины ребер AD, CD, ВС и ВА — это точки К, L, М, N (рис. 80). Так как KL и MN параллельны (KL и MN па- раллельны АС), то точки К, L, М и N лежат в одной плоскости. KLMN — это искомое сечение, так как LK = KN и KL1. KN. По- следнее вытекает из того, что АС1. BD.
Решения, указания, ответы 183 Касательная с уравнением (*) пересекает ось абсцисс в точке 1 1 (xt; 0); х{ можно определить из уравнения: —~ (х - х0) + — - 0. *0 Л0 Решая данное уравнение, получим х{ = 2х(). Точка (0; у2) пере- сечения с осью ординат определяется подстановкой в уравне- ние (*) значения х = 0. В итоге получаем у2 . Отрезки осей Л'о координат и касательной составляют прямоугольный треу- гольник, катеты которого имеют длины а = 2 !х0| и b . i*xoi Площадь данного треугольника равна 2. 4. Разобьем гирьки на пары: 18 пар гирек первого типа с суммой масс в каждой паре 38 г (это пары вида (1; 37), (2; 36), (3; 35) ... (18; 20)) и 32 пары второго типа с суммой масс в каждой такой паре по 139 г (это пары вида (38; 101), (39; 100),..., (69; 70)). На каждую чашку весов положим по 9 пар гирек первого типа и по 16 пар гирек второго типа. Тогда весы будут в равновесии. 5. Среди записанных 2011 положительных чисел имеется наи- меньшее. Обозначим его через Ь. Пусть а и с — соседние с ним j а + с . / 1 числа. 1огда b = —-— или b = Час , то есть b ~~ а с - Ь, или b'.a^c.h. Но b — это наименьшее число, поэтому b < а и b < с. Следовательно, в обоих случаях получаем: а h с, то есть «соседи» наименьшего числа также являются наименьшими. Переходя к рассмотрению числа а (и с), аналогичными рас- суждениями получаем, что «соседи^ «соседей» тоже являются наименьшими, и так далее 2011 раз. В итоге получаем, что все записанные числа равны друг другу Вариант 7 1. Воспользуемся геометрическим смыслом определенного инте- грала. Изобразим график подынтегральной функции (рис. 81). Площадь фигуры под графиком изображенной функции рав- на 5, поэтому: 3 |||х|-2|б& = 5. -3
184 Математические олимпиадные работы. 5-11 классы Рис. 81 2. Сложив уравнения системы, получаем: -2х - у + х + 2у х + у +-----г---7---~ = 0- * - yz Отсюда имеем (х + у)2 = 1, то есть х + у = 1 или х + у = -1. Рассмотрим первый случай. Учитывая его, получаем систему уравнений: Из данной системы находим: х1 = 0, х2 = 2. Им соответствуют У\ =1 1» У2 = “1- Аналогично, рассматривая случай х + у = -1, по- лучаем х3 = -1, у3 = 0; х4 = 1, z/4 = -2. Других решений нет. Ответ: (0; 1), (2; -1), (-1; 0), (1; -2). 3. Построим на стороне DC правильный треугольник DK{C, сое- диним К' с А и В (рис. 82). Треугольники ADK} и СВК{ равнобедренные (так как DK{ = DA, CKt = СВ). Тогда ZDAKi = | (180° - ZADKJ = | (180° - - 30°) = 75°. Поэтому ZK{AB = 15°. Значит, фигурирующая в условии задачи точка К совпадет с точкой Kv
Решения, указания, ответы 185 4. Чтобы представить 2010 в виде указанной разности, числа х и у должны быть одинаковой четности. Пусть х и у оба четные, тогда разностью" - уп должна делиться на 4, а 2010 на 4 не делится. Рас- смотрим случай, когда х и у оба нечетные, а п — четное. Тогда раз- ность т" - у” раскладывается на множители; одним из этих мно- жителей является х2 - У2, который делится на 4. Число же 2010 на 4 не делится. Значит, данный случай невозможен. Таким обра- зом, число 2010 представить в виде указанной разности нельзя. 5. Рассмотрим уравнение 2х2 - 2ух + (у2 - Зу) = 0. Его дискрими- нант равен 4(6у - у2). Он неотрицателен при 0 < у < 6. Заме- тим, что исходное уравнение можно привести к виду (у ~ л)2 + + х2 == Зу. Поскольку квадраты целых чисел при делении на 3 могут давать только остатки 0 и 1, то числа у - х и х, а значит, и сами числа х и у, кратны 3. Поэтому из соображений дели- мости получаем, что х делится на 3 и у делится на 3. Осталось перебрать случаи у = 0, у = 3, у = 6. В итоге получаем следую- щие решения: (0;0), (0;3), (3;3), (3;6). Ответ: (0;0), (0;3), (3;3), (3;6). Вариант 8 1. Представим все множители, кроме последнего, в следующем виде: |«з2 = Г~Ш| *°82013 ’°Й2013 1°Й2012 2011 - l°faoi3 20И 1°ё201з 2012 Тогда: а = log3 2 • log4 3 •... • log20l3 2012 =
186 в Математические олимпиадные работы. 5-11 классы ‘°§2013 . *°g20!3 . 1°&013 3 !°Й2013 Ьё2р1з20П . | 2012 = log 2. log20]32012 82013 82013 2. Пусть у ~ х2 — Зх3. Тогда у'(х) = 2х - 9х2 = х(2 -- 9х). Тогда (2 \ 12 у'(х) < 0 при х е (~о°;0) u ~;-Но I Так как у > —, то на луче ;-t-oo I ц'(х) < 0, а значит, функция у убывает на этом луче. 4 / А это означает, что: Это и требовалось доказать. 1 1 64 < 60' 3. Примем сторону треугольника АВ, лежащую против угла Z С = 120°, за х (рис. 83). Так как Z С — самый большой угол в треугольнике АВС, то сторона АВ — самая большая из сторон треугольника. Тогда длины двух других сторон треугольника равны (а - 1) и (х -2). По теореме косинусов имеем: х2 = (х - 1 )2 + (х - 2)2 - 2(х - 1) х х (х - 2)cosl20° = х2 - 2х + 1 +• х2 ~ 4х + 4 + х2 - х ~ 2х + 2 = Зх2 - - 9х + 7. Рис. 83 Уравнение х2 Зх2 - 9а + 7 будет равносильно уравнению 9± 5 2х2- 9х + 7 = 0, которое имеет корни: xJ2 « —-— = 1; 3,5. Так как х 1 не удовлетворяет условию задачи, то стороны тре- угольника равны 3,5; 2,5 и 1,5. Ответ: Стороны треугольника равны 3,5; 2,5 и 1,5.
Решения, указания, ответы 187 4. Можно, если расположить центр основания кубика в центре квадрата О так, чтобы ребро основания куба было параллельно диагонали квадрата (рис. 84). Рис. 84 Пусть отрезок MN содержит ребро куба. Так как треугольни- лиг г MN £0-0,5 AC 3>/2 ки АВС п MNB подобны, то-----=---------, ВО =----------. АС ВО 22 Тогда MN = 2(ВО - 0,5) = Зл/2 - 1 > 3. Заворачивая куб по диа- гонали квадрата, мы закроем нижнее основание и боковые гра- ни куба. При этом в каждом углу квадрата останется по треу- гольнику, которым можно закрыть больше четверти верхнего основания куба. 5. Рассмотрим остатки от деления правой и левой частей уравне- ний на 9. Перепишем исходное уравнение в виде 18(х3 - у3) + + (х3 + у3) = 51. Первое слагаемое в левой части уравнения де- лится на 9, а у второго остатки от деления на 9 могут быть 0,1,8, 2, 7, в то время как остаток от деления 51 на 9 равен 6. Поэтому исходное уравнение не имеет решений в натуральных числах. Вариант 9 1. Рассмотрим отношение: 20 И2009 _ 20112010 201 Г1 _ Г 2011 f"° , = 2O1O2010 - 2O1O2010 "(2OIOJ
188 Математические олимпиадные работы. 5-11 классы < <2010 i+-L_] 2010 •201 Г1. / 1 \а z . \2010 ~ Так как I 14- — I < 3, то I 1 ч-I • 201 Г1 <-------< 1. Зна- I. а) I. 2010) 2011 чит, 201 12009 < < 20102010. 2. Введем обозначения Vx - 1 = а и ^/-4 = Ь. Тогда х = а2 + 1, у = Ь2 +• 4. После подстановки этих выражений в исходное урав- нение получим: 24„ 2 Последнее уравнение преобразуется к виду (а - I)2 + (Ь - - 2)2 = 0, откуда а = 1,6 = 2. Тогда х = 2, у = 8. Ответ: х= 2, z/ = 8. 3. Рассмотрим симметрию относительно точки Е. Точки А и Ар В и Вх симметричны относительно точки Е. Значит, фигура АВХАХВ является параллелограммом. Площадь параллелограм- ма АВХАХВ в 4 раза больше площади треугольника АВЕ, а пло- щадь многоугольника ABCAXB{D тоже в 4 раза больше площа- ди треугольника АВЕ, то есть равна площади параллелограмма АВХАХВ. Отсюда следует, что точка D лежит на АВХ, а точка С — на ВА{. Поэтому отрезок AD параллелен ВС. 4. В соответствии с условием задачи составим систему уравнений: 100а + 106 + с = 13zz, а 4- 6 + с ~ 1 Зт. Здесь а,Ь,с — цифры, при этом а * 0; п = 1, 2,... 76; т = 1 или 2. Выразим с из второго уравнения и подставим в первое уравне- ние. После преобразований получим: 9(11а 4- 6) = 13(лг - т). Тогда 1 \а 4- 6 кратно 13. Рассматривая случаи а = 1, 2,... 9, нахо- дим соответствующие значения для 6 и с. В результате опреде- ляем все трехзначные числа, удовлетворяющие условию зада- чи: 247; 364; 481; 715; 832. zz 5. Пусть всего в лесу растет п грибов, тогда плохих из них —. По- 4 скольку в каждом грибе может жить не более 4 толстых чер-
Решения, указания, ответы 189 вей, то в каждом плохом грибе живет не менее 12-4 = 8 гощих червей, а значит, во всех плохих грибах проживают не менее 71 8 х — = 2п тощих червей. Толстых же червей во всех грибах не более 4и, то есть их численность превосходит численность тощих не более чем вдвое. Значит, тощие черви составляют не менее трети от общего числа червей в лесу. Вариант 10 1. 2. 3. Преобразуем функцию/(х) следующим образом:/(х)=х(х+ 1)х х (х + 2)(х + 3) = (х2 + Зх)(х2 + Зх + 2) = ((х2 + Зх + 1) - 1 )((х2 + + 3х + 1) + 1) = (х2 + 3х + I)2 - 1. Так как (х2 + Зх + 1 )2 > 0 для всех х и х2 + Зх + 1 =0 при х = -3±Л =-------, то наименьшее значение функции равно -1. Представим левую часть уравнения в виде: х4 - 4х - 1 = (х2 + Тогда исходное уравнение будет равносильно совокупности двух уравнений: , \ 2 /о . i Pi ex 1 - V2V2 -1 1) х2 - л/2 х + 1 - д/2 =0 имеет корни xt 2 =--у=-; 2) х2 + >/2 х + 1 4- у[2 = 0 не имеет корней, так как его дискри- минант отрицательный. _ 1 ± >/2>/2 -1 Ответ: х12 = -----=---- 2 Пусть АВС — заданный треугольник с прямым углом при вер- шине С, и пусть АС = h и ВС = а (рис. 85). Кроме того, пусть CD ± АВ и CD = /? и пусть Z ВАС = ос. Тогда расстояния от точки М (точки пересечения медиан А АВС до ч bah сторон треугольника) равны а площадь искомого тре- „ „ 1 ( a b a h . b h ) угольника 5, равна: У = —-----+----sin ос +----cos ос . 1 1 2 3 3 3 3 3 3
190 н Математические олимпиадные работы. 5-11 классы Но «sin а + 6cosa = с, где с — гипотенуза треугольника АВС. Следовательно: 1 ( 1 Л 1 (I Л 9 5 =1 :U + 1 1 fc = -5. 1 9^2) 9^2J 9 4. Так как 3« - 1 делится на 6 и 36-1 делится на а, то имеем два уравнения: За - 1 = nb и 36 - 1 = та, из которых получаем уравнение: 6(9 - тп) = т + 3. Из последнего уравнения имеем 777 + 3 тп = 9--------. Так как числа а, 6, т, п натуральные, то тп < 9. b Путем перебора, учитывая, что числа т и п натуральные и не о п + 3 , т + 3 делятся на 3, а также, что а = -, 6 =-----, получаем: 9- тп 9 - тп 1) п = 5, т = 1 ~-=> а = 2, 6 = 1; 2) п = 7, т = 1 => а = 5, 6 = 2; 3) п = 8, т = 1 «=11,6 = 4; 4) п = 2, т = 2 «=1,6=1; 5) п = 4, т = 2 « = 7, 6 = 5. При других подходящих т и п получаются те же « и 6. Ответ: а = 1, 6 = 1; а = 2, 6 = 1; а = 5, 6 = 2; а = 7, 6 = 5; а = 11, 6 = 4.
Решения, указания, ответы 191 1 5. Распилим куб на 1000 кубиков со стороной “ м. Тогда най- дется кубик, в котором сидят как минимум 3 таракана, Вычис- а - х 7з лИаМ радиус сферы, описанной вокруг такого кубика: г =-* — м. т 1 л/3 1 1ак как , то найдется сфера радиусом —, которая оу- дет содержать кубик с гремя тараканами.
Александр Фарков Математические олимпиадные работы. 5-11 классы Заведующая редакцией Литературный редактор Художник Корректор Верстка В. Малышкина А. Жданов С. Маликова С. Беляева А. Шляго Подписано в печать 16.04.10. Формат 60x90/16. Усл. л. л. 12. Тираж 5000 экз. Заказ № 155-1. ООО «Лидер». 194044, Санкт-Петербург, Б. Сампсониевский пр., д. 29а. Налоговая льгота - общероссийский классификатор продукции ОК 005-93. том 2; 95 3005 - литература учебная. Ошечатано с готовых диапозитивов в ООО «Типография Правда 1906». 195299. Санкт-Петербург, Киришская ул., 2. Тел.: (812) 531-20 00, 531-25-55